प्रयागराज शाखा पर IAS GS फाउंडेशन का नया बैच 29 जुलाई से शुरू
  संपर्क करें
ध्यान दें:

प्रारंभिक परीक्षा


प्रारंभिक परीक्षा

प्रारंभिक परीक्षा, 2024: एक विश्लेषणात्मक अध्ययन (सेट-D)

  • 16 Jun 2024
  • 241 min read

1. दिसंबर 2023 तक भारत सरकार द्वारा कितने परिसीमन आयोग गठित किये गए हैं?

(a) एक
(b) दो 
(c) तीन
(d) चार

उत्तर: (d)

व्याख्या: 

  • भारत में परिसीमन आयोग का गठन 4 बार किया जा चुका है – वर्ष 1952 में (परिसीमन आयोग अधिनियम, 1952 के तहत), वर्ष 1963 में (परिसीमन आयोग अधिनियम, 1962 के तहत), वर्ष 1973 में (परिसीमन आयोग अधिनियम, 1972 के तहत) और वर्ष 2002 में (परिसीमन आयोग अधिनियम, 2002 के तहत)। 
  • परिसीमन का तात्पर्य जनसंख्या में होने वाले परिवर्तनों के आलोक में किसी देश के क्षेत्रीय निर्वाचन क्षेत्रों की सीमाओं का निर्धारण करना है। संविधान के अनुच्छेद 82 में प्रावधान है कि प्रत्येक जनगणना के बाद संसद द्वारा परिसीमन अधिनियम बनाया जाए। 
  • भारत में परिसीमन आयोग एक उच्च शक्ति प्राप्त निकाय है जिसके आदेशों को विधिक शक्ति प्राप्त होती है तथा किसी भी न्यायालय के समक्ष इन आदेशों को प्रश्नगत नहीं किया जा सकता है।

अतः विकल्प (d) सही है।


2. संविधान (71वें संशोधन) अधिनियम, 1992 के द्वारा निम्नलिखित में से किस भाषा को शामिल करने के लिये संविधान की आठवीं अनुसूची में संशोधन किया गया है?

  1. कोंकणी
  2. मणिपुरी
  3. नेपाली
  4. मैथिली

नीचे दिये गए कूट का प्रयोग कर सही उत्तर चुनिये:

(a) 1, 2 और 3
(b) 1, 2 और 4
(c) 1, 3 और 4
(d) 2, 3 और 4

उत्तर: (a)

व्याख्या:

आठवीं अनुसूची:

  • इसमें भारत गणराज्य की आधिकारिक भाषाओं की सूची दी गई है। भारतीय संविधान के भाग XVII में अनुच्छेद 343 से 351 तक आधिकारिक भाषाओं का उल्लेख है।
  • संविधान की आठवीं अनुसूची में 22 भाषाएँ सम्मिलित है हालाँकि संविधान के प्रारंभ में 14 भाषाओं का उल्लेख किया गया था। वर्ष 1967 में सिंधी भाषा को इसमें जोड़ा गया। 
  • 71वें संविधान संशोधन अधिनियम, 1992 द्वारा कोंकणी, मणिपुरी एवं नेपाली को इसमें जोड़ा गया। इसके साथ ही आठवीं अनुसूचि में भाषाओं की कुल संख्या बढ़कर 18 हो गई।
  • बोडो, डोगरी, मैथिली एवं संथाली को 92वें संविधान संशोधन अधिनियम, 2003 द्वारा जोड़ा गया जो वर्ष 2004 में लागू हुआ। वर्तमान में संविधान की आठवीं अनुसूची में कुल 22 भाषाएँ शामिल हैं।

अतः विकल्प (a) सही है।

Source: 

Drishti IAS link: Eighth Schedule of the Indian Constitution

Other Authentic and Standard Sources: Constitutional provisions relating to the Eighth Schedule


3. निम्नलिखित युग्मों पर विचार कीजिये :

दल उसके नेता
1. भारतीय जन संघ डॉ. श्यामा प्रसाद मुखर्जी
2. सोशलिस्ट पार्टी सी. राजगोपालाचारी
3. कांग्रेस फॉर डेमोक्रेसी जगजीवन राम
4. स्वतंत्र पार्टी आचार्य नरेंद्र देव

उपर्युक्त में से कितने सही सुमेलित हैं ?

(a) केवल एक
(b) केवल दो
(c) केवल तीन
(d) सभी चार

उत्तर: (b)

व्याख्या:

  • भारतीय जनसंघ की स्थापना वर्ष 1951 में श्यामा प्रसाद मुखर्जी ने की थी। अतः युग्म 1 सही सुमेलित है। 
  • जय प्रकाश नारायण, राम मनोहर लोहिया और आचार्य नरेंद्र देव सोशलिस्ट पार्टी के प्रमुख नेता थे। अतः युग्म 2 सही सुमेलित नहीं है। 
  • कॉन्ग्रेस फॉर डेमोक्रेसी (CFD) एक भारतीय राजनीतिक पार्टी थी जिसकी स्थापना वर्ष 1977 में जगजीवन राम ने की थी। अतः युग्म 3 सही सुमेलित है। 
  • स्वतंत्र पार्टी की स्थापना वर्ष 1959 में कॉन्ग्रेस सरकार की समाजवादी नीतियों का विरोध करने वाले नेताओं द्वारा की गई थी। सी. राजगोपालाचारी, मीनू मसानी और एन. जी. रंगा सहित उस समय के कुछ प्रमुख नेताओं ने एक उदार-रूढ़िवादी पार्टी बनाने के लिये सहयोग किया। पार्टी बाजार आधारित अर्थव्यवस्था और लाइसेंस राज को समाप्त करने का उद्देश्य रखती है। अतः युग्म 4 सही सुमेलित नहीं है।

अतः विकल्प (b) सही है।

Source: 

Drishti IAS link: 

Other Authentic and Standard Sources:


4. भारत के संविधान के संबंध में निम्नलिखित कथनों में से कौन-से सही हैं?

  1. नगरपालिकाओं की शक्तियाँ संविधान के भाग 9-क में दी गई हैं।
  2. आपात उपबंध संविधान के भाग 18 में दिये गए हैं।
  3. संविधान के संशोधन से संबंधित उपबंध संविधान के भाग 20 में दिये गए हैं।

नीचे दिये गए कूट का प्रयोग कर उत्तर चुनिये :

(a) केवल 1 और 2
(b) केवल 2 और 3
(c) केवल 1 और 3
(d) 1, 2 और 3

उत्तर: (d)

व्याख्या: 

  • संविधान के भाग 9-क को संविधान (चौहत्तरवाँ संशोधन) अधिनियम, 1992 द्वारा सम्मिलित किया गया था। इसमें नगरीय स्तर पर स्थानीय स्वशासन अथवा नगर पालिकाओं संबंधी प्रावधान किये गए हैं। अतः कथन 1 सही है। 
  • संविधान के भाग 18 में राष्ट्रीय, राज्य और वित्तीय आपात स्थितियों सहित आपात उपबंधों के लिये प्रावधान हैं। अतः कथन 2 सही है। 
  • संविधान के भाग 20 में अनुच्छेद 368 संविधान और उसके उपबंधों में संशोधन करने की संसद की शक्ति से संबंधित है। इसमें कहा गया है कि संसद इस उद्देश्य के लिये निर्धारित प्रक्रिया के अनुसार किसी भी प्रावधान को जोड़ने, बदलने या निरस्त करने के माध्यम से संविधान में संशोधन कर सकती है। अतः कथन 3 सही है।

अतः विकल्प (d) सही है।

Source:

Drishti IAS link:

Other Authentic and Standard Sources:


5. भारत के संविधान के अनुसार निम्नलिखित में से कौन-सा कथन सही है?

(a) अंतर्राज्यीय व्यापार और वाणिज्य राज्य सूची के अधीन एक राज्य विषय है।
(b) अंतर्राज्यीय प्रवासन राज्य सूची के अधीन एक राज्य विषय है।
(c) अंतर्राज्यीय संगरोध संघ सूची के अधीन एक संघ विषय है।
(d) निगम कर राज्य सूची के अधीन एक राज्य विषय है।

उत्तर: (c)

व्याख्या: 

संविधान की सातवीं अनुसूची में तीन सूचियाँ हैं जो केंद्र और राज्यों के बीच शक्ति का वितरण करती हैं (अनुच्छेद 246)

  • संघ सूची में 98 विषय (मूल रूप से 97) हैं, जिन पर संसद को विधि निर्माण की विशेष शक्ति है।
  • राज्य सूची में 59 विषय (मूल रूप से 66) हैं जिन पर अकेले राज्य विधि निर्माण कर सकते हैं।
  • समवर्ती सूची में 52 विषय (मूल रूप से 47) हैं जिन पर केंद्र और राज्य दोनों विधि निर्माण कर सकते हैं।

संघ सूची के तहत विषयों का वर्गीकरणः

संघ सूची:

  • अंतर्राज्यीय व्यापार एवं वाणिज्य, संघ सूची के अधीन एक विषय है। अतः कथन (a) सही नहीं है।
  • अंतर्राज्यीय प्रवासन, संघ सूची के अधीन एक विषय है। अतः कथन (b) सही नहीं है।
  • अंतर्राज्यीय संगरोध, संघ सूची के अधीन एक विषय है। अतः कथन (c) सही है।
  • निगम कर, संघ सूची के अधीन एक विषय है। अतः कथन (d) सही नहीं है।

प्रवेश संख्या

सूची

विषय

42

संघ

अंतर्राज्यीय व्यापार एवं वाणिज्य

81

संघ

अंतर्राज्यीय प्रवासन संघ सूची  

81

संघ

अंतर्राज्यीय संगरोध

85

संघ

निगम कर 

Source: 


6. भारत के उच्चतम न्यायालय ने निजता के अधिकार को भारत के संविधान के निम्नलिखित में से किस अनुच्छेद के अंतर्गत रखा है?

(a) अनुच्छेद 15
(b) अनुच्छेद 16
(c) अनुच्छेद 19
(d) अनुच्छेद 21

उत्तर: (d)

व्याख्या: 

  • सर्वोच्च न्यायालय ने वर्ष 2017 में के. एस. पुट्टस्वामी बनाम भारत संघ के ऐतिहासिक निर्णय में गोपनीयता और इसके महत्त्व का वर्णन करते हुए उल्लिखित किया था कि निजता का अधिकार मौलिक एवं अविभाज्य है तथा यह व्यक्ति से जुड़ा हुआ है, जिसमें उस व्यक्ति एवं उसके द्वारा चुने गए विकल्पों के संदर्भ में सभी जानकारी शामिल है। अनुच्छेद 21 के अंतर्गत जीवन एवं व्यक्तिगत स्वतंत्रता के अधिकार के आंतरिक भाग के रूप में निजता के अधिकार को संरक्षित किया गया है।

 अतः विकल्प (d) सही है।

Source:


7. सैन्य कार्य विभाग के प्रमुख के रूप में चीफ ऑफ डिफेंस स्टाफ (CDS) के कर्तव्य कौन-से हैं?

  1. चीफ ऑफ स्टाफ कमेटी के स्थायी अध्यक्ष
  2. तीनों सेवा प्रधानों (सर्विस चीफ) पर सैन्य कमान का प्रयोग करना
  3. सभी तीनों सेवाओं (ट्राइ-सर्विस) के विषयों पर रक्षा मंत्री के प्रधान सैन्य सलाहकार

नीचे दिये गए कूट का प्रयोग कर सही उत्तर चुनिये :

(a) 1, 2 और 3
(b) केवल 1 और 2
(c) केवल 2 और 3
(d) केवल 1 और 3

उत्तर: (d)  

व्याख्या: 

चीफ ऑफ डिफेंस स्टाफ (CDS) 

  • इस पद का के सृजन की सिफारिश वर्ष 2001 में मंत्रियों के एक समूह (GoM) द्वारा की गई थी जिसे कारगिल समीक्षा समिति (1999) की रिपोर्ट का अध्ययन करने का काम सौंपा गया था।
  • CDS का पद भी लेफ्टिनेंट जनरल D.B. शेकटकर (retd.) की अध्यक्षता वाली शेकटकर समिति की सिफारिश पर स्थापित किया गया था। 

चीफ ऑफ डिफेंस स्टाफ (CDS) के कर्त्तव्यों एवं उत्तरदायित्वों में शामिल हैं:

  • चीफ ऑफ स्टाफ कमेटी के स्थायी अध्यक्ष के रूप में कार्य करना। अतः कथन 1 सही है। 
  • हालाँकि CDS तीनों सेवा प्रधानों पर किसी सैन्य कमान का प्रयोग नहीं करेगा लेकिन वह संयुक्त अभियानों में इनके बीच समन्वय सुनिश्चित करेगा। अतः कथन 2 सही नहीं है।
  • तीनों सेवाओं (ट्राइ-सर्विस) के मामलों पर माननीय रक्षा मंत्री के प्रधान सैन्य सलाहकार के रूप में कार्य करना। अतः कथन 3 सही है।
  • तीनों सेवाओं (ट्राइ-सर्विस) के संगठनों/एजेंसियों/कमानों का प्रशासन करना।
  • माननीय रक्षा मंत्री की अध्यक्षता वाली रक्षा अधिग्रहण परिषद का सदस्य होना।

अतः विकल्प (d) सही है। 

Source: 


8. मूलभूत आवश्यकताओं को पूरा करने सहित दूरस्थ क्षेत्रों में स्थानीय जन-समुदाय के उत्थान के लिये सेना द्वारा संचालित अभियान (ऑपरेशन) को क्या कहा जाता है?

(a) ऑपरेशन संकल्प
(b) ऑपरेशन मैत्री
(c) ऑपरेशन सद्भावना
(d) ऑपरेशन मदद

उत्तर: (c) 

व्याख्या:  

  • "ऑपरेशन सद्भावना" भारतीय सेना की एक मानवीय पहल है जिसका उद्देश्य दूरस्थ क्षेत्रों में स्थानीय जन-समुदायों की सामाजिक और आर्थिक स्थितियों में सुधार कर उनका उत्थान करना है।
  • विद्यालयों का संचालन करने, बुनियादी अवसंरचना का विकास करने एवं राष्ट्रीय एकीकरण यात्राओं को बढ़ावा देने सहित विभिन्न कल्याणकारी गतिविधियों के माध्यम से भारतीय सेना का उद्देश्य स्थानीय आबादी का सशक्तीकरण करना, रोज़गार के अवसर प्रदान करना तथा शिक्षा के क्षेत्र में सहयोग करना एवं इस प्रकार राष्ट्र निर्माण के प्रयासों में योगदान करना है। 

अतः विकल्प (c) सही है।

Source:


9. विश्व में किन्हीं दो देशों के मध्य सबसे लंबी सीमा निम्नलिखित में से किनके मध्य है?

(a) कनाडा और संयुक्त राज्य अमेरिका
(b) चिली और अर्जेंटीना
(c) चीन और भारत
(d) कजाख़स्तान और रशियन फेडरेशन

उत्तर: (a) 

व्याख्या:  

  • कनाडा और संयुक्त राज्य अमेरिका विश्व की सबसे लंबी अंतर्राष्ट्रीय सीमा साझा करते हैं, जो 8,890 किलोमीटर तक विस्तृत है। अतः विकल्प (a) सही है।
    • इसकी तुलना में, रूस-कज़ाकिस्तान सीमा 6,846 किलोमीटर लंबी है और चिली-अर्जेंटीना सीमा 5,308 किलोमीटर लंबी है। 
  • भारत, चीन के साथ 3488 किलोमीटर लंबी सीमा साझा करता है जो भारत के जम्मू और कश्मीर, हिमाचल प्रदेश, उत्तराखंड, सिक्किम एवं अरुणाचल प्रदेश राज्यों से स्पर्श होती है।

Source: 


10. लोकसभा में आचार समिति के संबंध में, निम्नलिखित कथनों में से कौन-से सही हैं?

  1. प्रारंभ में यह एक तदर्थ समिति थी।
  2. केवल कोई लोकसभा का सदस्य ही किसी लोकसभा सदस्य के अनैतिक आचरण से संबंधित शिकायत कर सकता है।
  3. यह समिति किसी ऐसे मामले पर विचार नहीं कर सकती जो न्यायाधीन है।

नीचे दिये गए कूट का प्रयोग कर उत्तर चुनिये :

(a) केवल 1 और 2
(b) केवल 2 और 3
(c) केवल 1 और 3
(d) 1, 2 और 3

उत्तर: (c) 

व्याख्या: 

  • आचार समिति का परिचय: 
    • इस समिति का गठन वर्ष 1997 में राज्यसभा में और 2000 में लोकसभा में किया गया था। यह संसद के सदस्यों के लिये आचरण संहिता को लागू करती है। यह कदाचार के मामलों की जाँच करती है एवं उचित कार्रवाई की सिफारिश करती है। इस प्रकार, यह संसद में अनुशासन व् शिष्टाचार बनाए रखने का कार्य करती है।
    • यह एकमात्र समिति है जो लोकसभा के सांसदों के खिलाफ नागरिकों की शिकायतों की जाँच करती है।
  • लोकसभा में आचार समिति का इतिहासः
    • लोकसभा के मामले में, सदन की विशेषाधिकार समिति के एक अध्ययन समूह ने वर्ष 1997 में ऑस्ट्रेलिया, ब्रिटेन और अमेरिका का दौरा करने के बाद विधिनिर्माताओं के आचरण और नैतिकता से संबंधित प्रथाओं की संवीक्षा के लिये एक आचार समिति के गठन की सिफारिश की, लेकिन इसे लोकसभा द्वारा स्वीकार नहीं किया गया।
    • विशेषाधिकार समिति ने अंततः 13वीं लोकसभा के दौरान एक आचार समिति के गठन की सिफारिश की। स्वर्गीय अध्यक्ष जी. एम. सी. बालयोगी ने वर्ष 2000 में एक तदर्थ आचार समिति का गठन किया, जो वर्ष 2015 में ही सदन का स्थायी हिस्सा बन गई। अतः कथन 1 सही है।
  • शिकायतों के लिये प्रक्रियाः 
    • शुरू में, आचार समिति के नियमों के तहत एक प्रावधान था जो किसी भी भारतीय नागरिक को एक विधिनिर्माता के खिलाफ शिकायत दर्ज़ करने की अनुमति देता था।
    • लेकिन, वर्ष 2014 में, एक उप-समिति ने सुझाव दिया कि इस तरह का प्रावधान किसी भी राजनीतिक प्रतिद्वंद्वी को कमज़ोर आधार पर एक विधिनिर्माता के खिलाफ शिकायत दर्ज़ करने की अनुमति देगा। इसके बाद नियम को संशोधित कर प्रावधान किया गया कि शिकायत एक विधिनिर्माता के माध्यम से ही प्रस्तुत की जा सकती थी।
    • तब तक, कोई भी व्यक्ति किसी अन्य लोकसभा सांसद के माध्यम से किसी सदस्य के खिलाफ शिकायत कर सकता है, साथ ही कथित कदाचार के साक्ष्य और एक हलफनामा जिसमें कहा गया है कि शिकायत "मिथ्यापूर्ण, निरर्थक या परेशान करने वाली" नहीं है। यदि सदस्य स्वयं शिकायत करता है, तो शपथ पत्र की आवश्यकता नहीं है। अतः कथन 2 सही नहीं है।
    • हालाँकि, समिति केवल मीडिया रिपोर्टों या वर्तमान में न्यायिक समीक्षा के तहत मामलों के आधार पर शिकायतों पर विचार नहीं करती है। अतः कथन 3 सही है।

अतः विकल्प (c) सही है।


11. डॉ. राजेंद्र प्रसाद के कार्यभार संभालने से पहले संविधान सभा के अस्थायी सभापति कौन थे?

(a) सी. राजगोपालाचारी
(b) डॉ. बी.आर. अंबेडकर
(c) टी.टी. कृष्णमाचारी
(d) डॉ. सच्चिदानंद सिन्हा

उत्तर: (d)  

व्याख्या: 

  • संविधान सभा की प्रथम बैठक 9 दिसंबर, 1946 को आहूत की गई, जिसमें मुस्लिम लीग ने अलग राज्य पाकिस्तान की मांग करते हुए सत्र का बहिष्कार किया। 
  • परिणामस्वरूप, केवल 211 सदस्य ही उपस्थित हुए। फ्राँसीसी परंपरा का पालन करते हुए सबसे बुजुर्ग सदस्य डॉ. सच्चिदानंद सिन्हा को संविधान सभा का अस्थायी अध्यक्ष चुना गया। अतः विकल्प (d) सही है।
  • इसके बाद संविधान सभा द्वारा डॉ. राजेंद्र प्रसाद को अध्यक्ष नियुक्त किया गया तथा एच.सी. मुखर्जी एवं वी.टी. कृष्णमाचारी को उपाध्यक्ष के रूप में चयनित किया गया।

Source: 


12. भारत सरकार अधिनियम, 1935 के संदर्भ में, निम्नलिखित कथनों पर विचार कीजिये :

  1. इसमें ब्रिटिश भारतीय प्रांतों और देशी रियासतों को मिलाकर एक अखिल भारतीय परिसंघ (फेडरेशन) की स्थापना का प्रावधान किया गया।
  2. रक्षा और विदेश संबंधी मामलों को परिसंघीय विधानमंडल के नियंत्रण के अधीन रखा गया।

उपर्युक्त कथनों में से कौन-सा/कौन-से सही है/हैं?

(a) केवल 1
(b) केवल 2
(c) 1 और 2 दोनों
(d) न तो 1, न ही 2

उत्तर: (c)

व्याख्या: 

भारत सरकार अधिनियम (1935):

  • इसने एक अखिल भारतीय परिसंघ (फेडरेशन) की स्थापना का प्रावधान किया जिसमें भारतीय प्रांतों एवं देशी रियासतों को इकाइयों के रूप में शामिल किया गया। अतः कथन 1 सही है।
    • इसने ग्यारह में से छह प्रांतों में द्विसदनात्मक व्यवस्था लागू की।
  • भारत सरकार अधिनियम (1935) ने विषयों को तीन सूचियों में वर्गीकृत किया: केंद्रीय सूची, राज्य सूची एवं समवर्ती सूची। 
    • संघीय (केंद्रीय) सूची में राष्ट्रीय महत्त्व के 59 विषय शामिल थे, जिनमें रक्षा, विदेशी मामले, वित्त, रेलवे, मुद्रा एवं प्रेस आदि संबंधी विषय शामिल थे। अतः कथन 2 सही है।
    • क्षेत्रीय महत्त्व के 54 विषयों को राज्य सूची में शामिल किया गया जिनमे  शिक्षा, चिकित्सा, कृषि, कानून और व्यवस्था तथा स्थानीय सरकारें शामिल थी।
    • समवर्ती सूची में बिजली, विवाह, तलाक, श्रम एवं आपराधिक कानून जैसे 36 विषय शामिल थे।
    • शेष विषयों को अवशिष्ट शक्तियों के अंतर्गत गवर्नर-जनरल को सौंप दिया गया था।

अतः विकल्प (c) सही है।

Source: 


13. निम्नलिखित में से कौन-सी नाटककार भास की रचना है?

(a) काव्यालंकार
(b) नाट्यशास्त्र
(c) मध्यम-व्यायोग:
(d) महाभाष्य

उत्तर: (c)

व्याख्या:

  • भास (Bhasa) पहले ज्ञात संस्कृत नाटककार थे और अब तक उनके कई पूर्ण नाटकों की खोज की जा चुकी है। उन्होंने महाभारत के प्रसंगों पर आधारित 5 नाटक लिखे जैसे मध्यम-व्यायोग, पंचरात्र, दूत वाक्यम, दूत घटोत्कचम, कर्णभरम, उरुभंगमअतः विकल्प (c) सही है।
  • भामह संभवतः 7वीं शताब्दी के विद्वान थे जिन्होंने संस्कृत काव्यशास्त्र पर काम किया। माना जाता है कि उन्होंने संस्कृत ग्रंथ काव्यलंकार की रचना की थी थी और वह दंडिन के समकालीन थे।
  • भरत द्वारा नाट्यशास्त्र में प्रदर्शन, अभिनय, भाव-भंगिमाएँ, मंच निर्देशन और अभिनय से संबंधित नियमों का वर्णन किया गया है।
  • महाभाष्य की रचना पतंजलि (दूसरी शताब्दी ईसा पूर्व) ने की थी। यह पाणिनि के ग्रंथ अष्टाध्यायी और कात्यायन की वार्तिक से संस्कृत व्याकरण के चुनिंदा नियमों पर एक टिप्पणी है।

Source:- A History of Ancient and Early Medieval India (From Stone Age to the 12th Century) Chapter 9 (Aesthetics and Empire, c. 300-600CE) Page no. 537 


14. संघभूति एक भारतीय बौद्ध भिक्षु, जिन्होंने चौथी शताब्दी ईसवी के अंत में चीन की यात्रा की, निम्नलिखित में से किस पर भाष्य के लेखक थे ?

(a) प्रज्ञापारामिता सूत्र
(b) विसुद्धिमग्गो
(c) सर्वास्तिवाद विनय
(d) ललितविस्तर

उत्तर: (c)

व्याख्या:

सर्वास्तिवाद विनय:

  • संपूर्ण सर्वास्तिवाद विनय चीनी बौद्ध धर्मग्रंथ में विद्यमान है।
  • अपने प्रारंभिक इतिहास में, सर्वास्तिवाद विनय चीन में सर्वसामान्य विनय परंपरा थी।
  • संघभूति सर्वास्तिवाद विनय पर भाष्य के लेखक थे। अतः विकल्प (c) सही है।

प्रज्ञापारामिता सूत्र:

  • प्रज्ञापारामिता सूत्र महायान सूत्रों में सबसे पुराने हैं और साथ ही महायान बौद्ध दर्शन की नींव भी हैं।
  • ये दुर्लभ ग्रंथ बौद्ध धर्मग्रंथों के साथ-साथ चीनी एवं तिब्बती धर्मग्रंथों दोनों में पाए जाते हैं।

विसुद्धिमग्गो:

  • विसुद्धिमग्गो, थेरवाद बौद्ध धर्म के महाविहार शाखा की शिक्षा का एक विश्वकोश होने के साथ ही उत्कृष्ट सारांश एवं व्याख्या है।
  • इसे महान बौद्ध टीकाकार बुद्धघोष ने 5वीं शताब्दी में श्रीलंका के राजा महानामा के शासनकाल के दौरान लिखा था।

ललितविस्तर:

  • ललितविस्तर एक बहुत ही महत्त्वपूर्ण संस्कृत बौद्ध ग्रंथ है। यह बुद्ध की जीवनी होने के साथ-साथ मूल रूप से हीनयान संप्रदाय के सर्वास्तिवाद संप्रदाय का ग्रंथ है।
  • यह ईसाई युग की प्रारंभिक शताब्दियों के दौरान भारत के सामाजिक एवं सांस्कृतिक इतिहास पर भी प्रकाश डालता है।
  • ललितविस्तर एक एकीकृत ग्रन्थ नहीं है और न ही यह किसी एक लेखक की रचना है।

Source:- मध्य एशिया तथा चीन में भारतीय संस्कृति (सत्यकेतु विद्यालंकार) Page no. 176 and 177 


15. यूनेस्को (UNESCO) द्वारा जारी विश्व धरोहर सूची में शामिल की गई निम्नलिखित संपत्तियों पर विचार कीजिये :

  1. शांतिनिकेतन
  2. रानी-की-वाव
  3. होयसला के पवित्र मंदिर समूह
  4. बोधगया स्थित महाबोधि मंदिर परिसर

उपर्युक्त में से कितनी संपत्तियों को वर्ष 2023 में शामिल किया गया?

(a) केवल एक
(b) केवल दो
(c) केवल तीन
(d) सभी चार

उत्तर: (b)

व्याख्या:

शांतिनिकेतन:पश्चिम बंगाल

  • रवींद्रनाथ टैगोर द्वारा स्थापित शांतिनिकेतन एक आवासीय विद्यालय के साथ-साथ प्राचीन भारतीय परंपराओं तथा धार्मिक एवं सांस्कृतिक सीमाओं से परे मानवता की एकता की दृष्टि पर आधारित कला का केंद्र था।
  • शांतिनिकेतन को वर्ष 2023 में यूनेस्को द्वारा भारत के 41वें विश्व धरोहर स्थल के रूप में मान्यता दी गई है। अतः कथन (1) सही है।

रानी-की-वाव:गुजरात

  • सरस्वती नदी के तट पर स्थित रानी-की-वाव का निर्माण 11वीं शताब्दी में एक राजा की स्मृति में कराया गया था।
  • रानी-की-वाव को वर्ष 2014 में यूनेस्को द्वारा भारत के 32वें विश्व धरोहर स्थल के रूप में मान्यता दी गई है। कथन (2) सही नहीं है।

होयसल पवित्र मंदिर समूह:कर्नाटक

  • इसमें दक्षिण भारत में 12वीं से 13वीं शताब्दी के होयसल शैली के मंदिर परिसरों के प्रतिनिधि उदाहरण शामिल हैं।
  • होयसल शैली के मंदिर परिसरों को यूनेस्को द्वारा वर्ष 2023 में भारत के 42वें विश्व धरोहर स्थल के रूप में मान्यता दी गई है। अतः कथन (3) सही है।

महाबोधि मंदिर परिसर:बोधगया

  • महाबोधि मंदिर परिसर भगवान बुद्ध के जीवन एवं विशेष रूप से ज्ञान प्राप्ति से संबंधित चार पवित्र स्थलों में से एक है।
  • महाबोधि मंदिर परिसर को वर्ष 2002 में यूनेस्को द्वारा भारत के 23वें विश्व धरोहर स्थल के रूप में मान्यता दी गई है। अतः कथन (4) सही नहीं है।

अतः विकल्प (b) सही है।


16. भारत के संविधान के अनुच्छेद 368 के अनुसार, संसद निम्नलिखित में से किनके द्वारा संविधान के किसी उपबंध में संशोधन कर सकेगी?

  1. परिवर्धन
  2. परिवर्तन
  3. निरसन

नीचे दिये गए कूट का प्रयोग कर सही उत्तर चुनिये :

(a) केवल 1 और 2
(b) केवल 2 और 3
(c) केवल 1 और 3
(d) 1, 2 और 3

उत्तर: (d)

व्याख्या:

  • संविधान के भाग XX में अनुच्छेद 368 संविधान एवं इसकी प्रक्रियाओं में संशोधन करने की संसद की शक्ति से संबंधित है। इसमें कहा गया है कि संसद इस उद्देश्य के लिये निर्धारित प्रक्रिया के अनुसार, किसी भी प्रावधान के परिवर्धन, परिवर्तन या निरसन के माध्यम से संविधान में संशोधन कर सकती है।
  • संसद संविधान के प्रावधानों में संशोधन कर सकती है, लेकिन इसके 'मूल ढाँचे' को नष्ट नहीं कर सकती। केशवानंद भारती वाद (1973) में सर्वोच्च न्यायालय ने यह निर्णय दिया गया था।

अतः विकल्प (d) सही है।

Drishti IAS link:- Procedure of Amendment 

Other Source: PART XX AMENDMENT OF THE CONSTITUTION


17. निम्नलिखित देशों पर विचार कीजिये :

  1. इटली
  2. जापान
  3. नाइजीरिया
  4. दक्षिण कोरिया
  5. दक्षिण अफ्रीका

उपर्युक्त में से किन देशों का उल्लेख प्राय: मीडिया में उनकी निम्न जन्म दर, अथवा वृद्धोन्मुख जनसंख्या अथवा ह्रासमान जनसंख्या के लिये किया जाता है?

(a) 1, 2 और 4
(b) 1, 3 और 5
(c) केवल 2 और 4
(d) केवल 3 और 5

उत्तर: (a)

व्याख्या:

  • हाल ही में इटली, जापान तथा दक्षिण कोरिया का उल्लेख मीडिया में उनकी निम्न जन्म दर, वृद्धोन्मुख जनसंख्या या ह्रासमान जनसंख्या के लिये किया गया है।
  • दक्षिण कोरिया की प्रजनन दर विश्व में सबसे कम है। संयुक्त राष्ट्र ने वर्ष 2021 में चेतावनी दी थी कि दक्षिण कोरिया वर्ष 2100 तक अपनी जनसंख्या को आधा करने की राह पर है। अत: बिंदु 4 सही है।
  • जापान लगातार आठवें वर्ष, रिकॉर्ड निम्न जन्म दर के साथ प्रमुख जनसांख्यिकीय चुनौती का सामना कर रहा है। वर्ष 2024 में जापान की वर्तमान प्रजनन दर प्रति महिला 1.374 जन्म है।
    • स्वास्थ्य एवं कल्याण मंत्रालय के सबसे हालिया आँकड़ों से चिंताजनक गिरावट का पता चला है। इसके अनुसार वर्ष 2023 में केवल 7,58,631 बच्चों का जन्म हुआ, जो पिछले वर्ष से 5.1 प्रतिशत की कमी को दर्शाता है और साथ ही यह वर्ष 1899 में सांख्यिकीय रिकॉर्ड प्रारंभ होने के बाद से सबसे निम्न बिंदु/स्तर को दर्शाता है। अतः बिंदु 2 सही है।
  • इटली की जन्म दर वर्ष 2023 में रिकॉर्ड निचले स्तर पर आ गई, जो लगातार 15वीं वार्षिक गिरावट है। यहाँ प्रजनन दर गिरकर प्रति महिला 1.20 बच्चे रह गई, जो स्थिर जनसंख्या के लिये आवश्यक 2.1 की दर से बहुत कम है। अतः बिंदु 1 सही है।
    • इटली में विगत वर्ष 3,79,000 बच्चों का जन्म हुआ। यह आँकड़ा वर्ष 2022 से 3.6% कम और वर्ष 2008 से 34.2% कम है। 
  • दक्षिण अफ्रीका में महिलाएँ अपने जीवनकाल में औसतन 2.33 बच्चों को जन्म देती हैं। हालाँकि यह दर कम हो रही है, फिर भी यहाँ जनसंख्या में वृद्धि जारी है।
    • दक्षिण अफ्रीका में कामकाजी उम्र की जनसंख्या में कमी आने की कोई आशंका नहीं है, जबकि यहाँ कई उच्च आय वाले देश हैं। फिर भी जनसंख्या संरचना के अनुमानों से पता चलता है कि यहाँ वरिष्ठ नागरिकों की संख्या में वृद्धि होगी।
    • वर्ष 2022 के अनुसार, दक्षिण अफ्रीका की जनसंख्या में वृद्ध नागरिकों की हिस्सेदारी 5.9% है। इस प्रकार, वहाँ की जनसंख्या अभी वृद्ध नहीं हुई है। अतः बिंदु 5 सही नहीं है।
  • नाइजीरिया, अफ्रीका का सबसे बड़ा एवं सर्वाधिक आबादी वाला देश है और साथ ही यह विश्व का सातवाँ सबसे बड़ा देश भी है। वर्तमान में इसकी जनसंख्या 215 मिलियन है।
    • इस देश की जनसंख्या वृद्धि दर लगातार उच्च रही है, जो 1960 के दशक से लगातार 2% से अधिक रही है। विश्व बैंक के अनुसार, वर्तमान में जनसंख्या वृद्धि दर 2.41% है। अतः बिंदु 3 सही नहीं है।

अतः विकल्प (a) सही है।

Source:-  


18. संसद में धन विधेयक के संबंध में निम्नलिखित कथनों में से कौन-से सही हैं ?

  1. अनुच्छेद 109 में धन विधेयक के संबंध में विशेष प्रक्रिया का उल्लेख है।
  2. धन विधेयक राज्यसभा में पुर:स्थापित नहीं किया जाएगा।
  3. राज्यसभा या तो विधेयक को अनुमोदन दे सकती है या परिवर्तन के लिये सुझाव दे सकती है किंतु इसे अस्वीकार नहीं कर सकती।
  4. राज्यसभा द्वारा धन विधेयक में सुझाए गए संशोधन को लोकसभा द्वारा स्वीकार करना होगा।

नीचे दिये गए कूट का प्रयोग कर उत्तर चुनिये :

(a) केवल 1 और 2
(b) केवल 2 और 3
(c) 1, 2 और 3
(d) 1, 3 और 4

उत्तर: (c)

व्याख्या:

  • संविधान के अनुच्छेद 109 में धन विधेयक के संबंध में एक विशेष प्रक्रिया का उल्लेख किया गया है। अतः कथन 1 सही है।
  • धन विधेयक को केवल लोकसभा में प्रस्तुत किया जाना चाहिये तथा उसे राज्यसभा (राज्य परिषद) में प्रस्तुत नहीं किया जा सकता।अतः कथन 2 सही है।
  • राज्यसभा किसी धन विधेयक पर केवल सिफारिशें कर सकती है लेकिन उसमें संशोधन अथवा अस्वीकार करने की शक्ति उसके पास नहीं है।अतः कथन 3 सही है।
  • राज्यसभा द्वारा धन विधेयक में प्रस्तावित संशोधन, जिसे लोकसभा चाहे तो अस्वीकार कर सकती है। अतः कथन 4 सही नहीं है।

अतः विकल्प (c) सही है।


19. निम्नलिखित में से कौन-सा/कौन-से, भारतीय सशस्त्र सेना की तीनों सेवाओं में समकक्ष रैंक के संदर्भ में सही सुमेलित है/हैं?

 

थल सेना

वायु सेना

नौसेना

1.

ब्रिगेडियर

एयर कमोडोर

कमांडर

2.

मेजर जनरल

एयर वाइस मार्शल

वाइस एडमिरल

3.

मेजर

स्क्वाड्रन लीडर 

लेफ्टिनेंट कमांडर

4.

लेफ्टिनेंट कर्नल

ग्रुप कैप्टन

कैप्टन

नीचे दिये गए कूट का प्रयोग कर सही उत्तर चुनिये :

(a) 1 और 4
(b) 1 और 3
(c) 2, 3 और 4
(d) केवल 3

उत्तर: (d)

व्याख्या:

थल सेना, नौसेना तथा वायु सेना भारत की प्राथमिक रक्षा सेवाएँ हैं। उन्हें उनके असाधारण संगठन, संरचना, सैन्य तैयारियों एवं अनुशासन के लिये जाना जाता है, जो खतरों के साथ-साथ प्राकृतिक या मानव निर्मित आपदाओं का कुशलतापूर्वक प्रतिउत्तर देने में सक्षम हैं। इन सशस्त्र बलों में कमीशंड अधिकारी एवं गैर-कमीशंड कार्मिक समकक्ष रैंक रखते हैं, साथ ही इन्हें अतिरिक्त मानद रैंक भी प्रदान की जाती हैं।

 

थल सेना

वायु सेना

नौसेना

1.

ब्रिगेडियर

एयर कमोडोर

कमोडोर

2.

मेजर जनरल

एयर वाइस मार्शल

रियर एडमिरल

3.

मेजर

स्क्वाड्रन लीडर 

लेफ्टिनेंट कमांडर

4.

लेफ्टिनेंट कर्नल

विंग कमांडर

कमांडर

अतः विकल्प (d) सही है।

Source: https://vishwabharatigurukul.com/equivalent-ranks-indian-armed-forces/


20. पूर्वोत्तर परिषद् (NEC) की स्थापना पूर्वोत्तर परिषद् अधिनियम, 1971 द्वारा की गई थी। वर्ष 2002 में NEC अधिनियम में संशोधन के बाद, परिषद् में निम्नलिखित में से किन-किन सदस्यों को शामिल किया गया है?

  1. संघटक राज्य का राज्यपाल
  2. संघटक राज्य का मुख्यमंत्री
  3. भारत के राष्ट्रपति द्वारा नामनिर्दिष्ट तीन सदस्य
  4. भारत का गृह मंत्री

नीचे दिये गए कूट का प्रयोग कर सही उत्तर चुनिये :

(a) केवल 1, 2 और 3
(b) केवल 1, 3 और 4
(c) केवल 2 और 4
(d) 1, 2, 3 और 4

उत्तर: A

व्याख्या:-

  • पूर्वोत्तर परिषद (संशोधन) अधिनियम, 2002 के अनुसार, परिषद की संरचना इस प्रकार होगी:
    • राज्यों के राज्यपाल का पद धारण करने वाला व्यक्ति या व्यक्तियों 
    • अरुणाचल प्रदेश, असम, मणिपुर, मेघालय, मिज़ोरम, नागालैंड सिक्किम और त्रिपुरा राज्यों के मुख्यमंत्री;
    • राष्ट्रपति द्वारा नामित किये जाने वाले तीन सदस्य
    • राष्ट्रपति परिषद के अध्यक्ष को नामित करेंगे

अतः विकल्प A सही है।

Source: 


21. ‘नारी शक्ति वंदन अधिनियम’ के संबंध में निम्नलिखित कथनों पर विचार कीजिये :

  1. ये उपबंध 18वीं लोकसभा से प्रभावी होंगे।
  2. अधिनियम बनने के बाद यह 15 वर्षों के लिये प्रवर्तन में रहेगा।
  3. इसमें अनुसूचित जातियों के लिये आरक्षित कोटे के भीतर अनुसूचित जातियों की महिलाओं के लिये स्थानों के आरक्षण के उपबंध हैं।

उपर्युक्त कथनों में से कौन-से सही हैं?

(a) 1, 2 और 3
(b) केवल 1 और 2
(c) केवल 2 और 3
(d) केवल 1 और 3

उत्तर: (c)

व्याख्या:

नारी शक्ति वंदन अधिनियम, 2023

संविधान (106वाँ संशोधन) अधिनियम, 2023 लोकसभा, राज्य विधानसभाओं तथा राष्ट्रीय राजधानी क्षेत्र दिल्ली की विधानसभा में महिलाओं के लिये एक तिहाई सीटें आरक्षित करता है, जिनमें अनुसूचित जातियों एवं अनुसूचित जनजातियों के लिये आरक्षित सीटें भी शामिल हैं। अतः कथन 3 सही है।

  • यह अधिनियम परिसीमन की प्रक्रिया शुरू होने के बाद प्रभावी होगा जो जनगणना होने के बाद ही किया जा सकता है।
  • 18वीं लोकसभा के चुनावों के पश्चात् जनगणना एवं परिसीमन किया जाएगा तथा उसके बाद ही अधिनियम लागू किया जाएगा। अतः कथन 1 सही नहीं है।
  • आरक्षण अधिनियम के लागू होने के बाद आयोजित जनगणना के प्रकाशन के बाद प्रभावी होगा और साथ ही 15 वर्ष की अवधि तक प्रवर्तन में रहेगा, जिसका संभावित विस्तार संसदीय कार्रवाई द्वारा निर्धारित किया जाएगा। अतः कथन 2 सही है।
  • यह अधिनियम लोकसभा तथा राज्य विधानसभाओं में अनुसूचित जातियों एवं अनुसूचित जनजातियों के लिये आरक्षित सीटों पर भी लागू होगा। 

अतः विकल्प (c) सही है।

Source:


22. ‘अभ्यास मित्र शक्ति-2023’ के संबंध में निम्नलिखित में से कौन-से कथन सही हैं?

  1. यह भारत और बांग्लादेश के मध्य संयुक्त सैन्य अभ्यास था।
  2. इसका प्रारंभ औंध (पुणे) में हुआ।
  3. आतंकवाद-विरोधी अभियानों के दौरान संयुक्त प्रतिक्रिया इस अभियान का एक लक्ष्य था।
  4. भारतीय वायुसेना इस अभ्यास का एक हिस्सा थी।

नीचे दिये गए कूट का प्रयोग कर उत्तर चुनिये :

(a) 1, 2 और 3
(b) 1, 2 और 4
(c) 1, 3 और 4
(d) 2, 3 और 4

उत्तर: (d) 

व्याख्या:

अभ्यास मित्र शक्ति- 2023

  • भारत एवं श्रीलंका के बीच संयुक्त सैन्य अभ्यास “अभ्यास मित्र शक्ति- 2023” का नौवाँ संस्करण आयोजित किया गया। अतः कथन 1 सही नहीं है।
  • इसकी शुरुआत औंध (पुणे) से हुई। अतः कथन 2 सही है।
  • इस अभ्यास का उद्देश्य आतंकवाद विरोधी अभियानों के दौरान संयुक्त प्रतिक्रियाओं का समन्वय करना है। अतः कथन 3 सही है।
  • इस अभ्यास में भारतीय वायु सेना के 15 सैनिकों तथा श्रीलंका वायु सेना के 5 सैनिकों ने भाग लिया। अतः कथन 4 सही है।

अतः विकल्प (d) सही है।

Source:


23. प्रतिषेध रिट उच्चतम न्यायालय या उच्च न्यायालयों द्वारा किसे और किस प्रयोजन से जारी किया गया एक आदेश है?

(a) किसी सरकारी अधिकारी को, उसे किसी विशिष्ट कार्रवाई करने से प्रतिषेध करने के लिये
(b) संसद/विधानसभा को, मद्यनिषेध पर कोई विधि पारित करने के लिये
(c) निचली अदालत को, किसी मामले में कार्यवाही जारी रखने का प्रतिषेध करने के लिये
(d) सरकार को, उसे किसी असंवैधानिक नीति का अनुपालन करने से प्रतिषेध करने के लिये

उत्तर: (c)

व्याख्या: 

रिट: सर्वोच्च न्यायालय (अनुच्छेद 32 के तहत) तथा उच्च न्यायालय (अनुच्छेद 226 के तहत) बंदी प्रत्यक्षीकरण, परमादेश, प्रतिषेध, उत्प्रेषण तथा अधिकार-पृच्छा रिट जारी कर सकते हैं। इसके अतिरिक्त, संसद (अनुच्छेद 32 के तहत) किसी अन्य न्यायालय को ये रिट जारी करने का अधिकार दे सकती है।

  • प्रतिषेध: प्रतिषेध का शाब्दिक अर्थ है ‘निषेध करना’। यह उच्च न्यायालय द्वारा निचली अदालत या न्यायाधिकरण को उसके अधिकार क्षेत्र से बाहर जाने या उसके अधिकार क्षेत्र का अतिक्रमण करने से रोकने के लिये जारी किया जाता है। इस प्रकार, गतिविधि को निर्देशित करने वाले परमादेश के विपरीत, प्रतिषेध निष्क्रियता को निर्देशित करता है।
  • प्रतिषेध केवल न्यायिक एवं अर्ध-न्यायिक प्राधिकारियों के विरुद्ध ही जारी की जा सकती है। यह प्रशासनिक अधिकारियों, विधायी निकायों एवं निजी व्यक्तियों अथवा निकायों के विरुद्ध उपलब्ध नहीं है। अतः विकल्प (c) सही है।
  • प्रतिषेध रिट एक विधिक पद है जिसका अर्थ है 'निषेध करना, रोकना, प्रतिबंधित करना’ और साथ ही  इसे 'स्थगन आदेश' के रूप में भी जाना जाता है।

Drishti IAS Link: https://www.drishtiias.com/daily-updates/daily-news-analysis/writs

Source: The Writ of Prohibition - Legal Articles - Free Law


24. निम्नलिखित कथनों पर विचार कीजिये :

  1. वह राज्य का राज्यपाल है जो उस राज्य के किसी समुदाय को अनुसूचित जनजाति के रूप में मान्यता देता है और घोषित करता है।
  2. किसी राज्य में अनुसूचित जनजाति के रूप में घोषित किसी समुदाय के लिये यह आवश्यक नहीं है कि दूसरे राज्य में भी ऐसा हो।

उपर्युक्त कथनों में से कौन-सा/कौन-से सही है/हैं ?

(a) केवल 1
(b) केवल 2
(c) 1 और 2 दोनों
(d) न तो 1, न ही 2

उत्तर: (b) 

व्याख्या:

  • अनुसूचित जनजातियों की परिभाषा:
    • भारतीय संविधान के अनुच्छेद 366 (25) में निर्धारित किया गया है कि अनुसूचित जनजातियों से तात्पर्य ऐसी जनजातियों या जनजातीय समुदायों से है जिन्हें संविधान के अनुच्छेद 342 के अंतर्गत अनुसूचित जनजाति माना गया है।
    • अनुच्छेद 342(1): किसी भी राज्य/संघ राज्य क्षेत्र के संबंध में राष्ट्रपति (राज्य के मामले में राज्यपाल के परामर्श के बाद) उस राज्य/संघ राज्य क्षेत्र में जनजातियों/आदिवासी समुदायों/जनजातियों के भाग या समूहों/आदिवासी समुदायों को अनुसूचित जनजाति के रूप में निर्दिष्ट कर सकते हैं। अतः कथन 1 सही नहीं है।
  • अनुसूचित जनजातियों की सूची राज्य/संघ राज्य क्षेत्र विशेष के लिये है और किसी राज्य में अनुसूचित जनजाति के रूप में घोषित समुदाय को दूसरे राज्य/संघ राज्य क्षेत्र में भी अनुसूचित जनजाति घोषित किया जाना आवश्यक नहीं है। अतः कथन 2 सही है।

अतः विकल्प (b) सही है।


25. केंद्रीय बजट (यूनियन बजट) के संदर्भ में, निम्नलिखित कथनों पर विचार कीजिये :

  1. प्रधानमंत्री की ओर से केंद्रीय वित्त मंत्री संसद के दोनों सदनों के समक्ष वार्षिक वित्तीय विवरण रखते हैं।
  2. केंद्रीय (यूनियन) स्तर पर, भारत के राष्ट्रपति की अनुशंसा के बिना अनुदानों की माँग नहीं की जा सकती।

उपर्युक्त कथनों में से कौन-सा/कौन-से सही है/हैं ?

(a) केवल 1
(b) केवल 2
(c) 1 और 2 दोनों
(d) न तो 1, न ही 2

उत्तर: (b)

व्याख्या:

  • राष्ट्रपति की ओर से केंद्रीय वित्त मंत्री संसद के दोनों सदनों के समक्ष वार्षिक वित्तीय विवरण प्रस्तुत करते हैं। अतः कथन 1 सही नहीं है।
  • भारतीय संविधान के अनुच्छेद 112 के अनुसार, राष्ट्रपति प्रत्येक वित्तीय वर्ष के संबंध में संसद के दोनों सदनों के समक्ष उस वर्ष के लिये भारत सरकार की प्राक्कलित प्राप्तियों और व्यय का विवरण रखवाएगा, जिसे "वार्षिक वित्तीय विवरण" कहा गया है।
  • भारतीय संविधान के अनुच्छेद 113 के अनुसार, संघ स्तर पर भारत के राष्ट्रपति की सिफारिश के बिना अनुदान की कोई मांग नहीं की जा सकती। अतः कथन 2 सही है।

Sources: 

Drishti IAS Link:

https://www.drishtiias.com/daily-updates/daily-news-analysis/union-budget-2023-24#:~:text=What%20are%20the%20Constitutional%20Provisions%20regarding%20Budget%3F 


26. निम्नलिखित में से कौन ‘‘द इंडिया वे (The India Way)’’ और ‘‘व्हाई भारत मैटर्स (Why Bharat Matters)’’ पुस्तकों के लेखक हैं?

(a) भूपेंद्र यादव
(b) नलिन मेहता
(c) शशि थरूर
(d) सुब्रह्मण्यम जयशंकर

उत्तर: (d)

व्याख्या:

सुब्रह्मण्यम जयशंकर एक भारतीय राजनयिक और राजनीतिज्ञ हैं जो वर्ष 2019 से भारत सरकार के विदेश मंत्री के रूप में कार्यरत हैं।

उन्होंने निम्नलिखित पुस्तकें लिखी हैं: 

  • द इंडिया वे: स्ट्रेटेजीज़ फॉर एन अनसर्टेन वर्ल्ड (2020)
  • व्हाई भारत मैटर्स (2024)

अतः विकल्प (d) सही है।

sources : 

Drishti IAS Link: NA


27. निम्नलिखित युग्मों पर विचार कीजिये :

   देश              समाचार में रहने का कारण

  1. अर्जेंटीना    -  सबसे खराब आर्थिक संकट
  2. सूडान       -  देश की नियमित सेना और अर्धसैनिक बलों के मध्य युद्ध
  3. तुर्की         -  NATO की अपनी सदस्यता निरस्त कर दी

उपर्युक्त युग्मों में से कितने सही सुमेलित हैं ?

(a) केवल एक युग्म
(b) केवल दो युग्म
(c) सभी तीन युग्म
(d) कोई युग्म नहीं

उत्तर: (b)

व्याख्या:

  • अर्जेंटीना: विश्व में सर्वाधिक मुद्रास्फीति दर का अनुभव करने वाले देशों में से एक है तथा एक दशक से ज़्यादा समय तक आर्थिक स्थिरता एवं बढ़ती निर्धनता के बाद, अर्जेंटीना एक बार पुनः आर्थिक पतन के कगार पर पहुँच गया है। अर्जेंटीना के आर्थिक संकट और चिरकालिक मुद्रास्फीति का मुख्य कारण अत्यधिक सार्वजनिक व्यय है जो कि मुद्रा निर्माण द्वारा वित्तपोषित है। अतः युग्म 1 सही सुमेलित है।
  • सूडान: सूडानी सेना और अर्द्धसैनिक बलों के बीच संघर्ष आरंभ होने के बाद से खार्तूम में कम-से-कम 185 लोग मारे गए और 1,800 घायल हो गए। यह संघर्ष अप्रैल 2019 से शुरू हुआ, जब देश भर में विद्रोह के बाद सैन्य जनरलों ने सूडान के लंबे समय तक राष्ट्रपति रहे उमर अल-बशीर को सत्ता से अपदस्थ कर दिया था। अतः युग्म 2 सही सुमेलित है।
  • तुर्की: तुर्की वर्ष 1952 से नाटो का सदस्य है और अपनी सामरिक स्थिति और सैन्य क्षमताओं के कारण गठबंधन में एक महत्त्वपूर्ण भूमिका निभाता है। तुर्की और अन्य नाटो सदस्यों के बीच विभिन्न मुद्दों पर तनाव तथा असहमति रही है, लेकिन तुर्की NATO से बाहर नहीं हुआ है। अतः युग्म 3 सही सुमेलित नहीं है।

Sources:

Drishti IAS Link: NA


28. निम्नलिखित कथनों पर विचार कीजिये :

कथन-I : सुमेड पाइपलाइन यूरोप को जाने वाले फारस की खाड़ी के तेल और प्राकृतिक गैस नौभार (शिपमेंट) के लिये एक रणनीतिक मार्ग है।

कथन-II: सुमेड पाइपलाइन लाल सागर को भूमध्य सागर से जोड़ती है।

उपर्युक्त कथनों के संबंध में, निम्नलिखित में से कौन-सा सही है?

(a) कथन-I और कथन-II दोनों सही हैं तथा कथन-II, कथन-I की व्याख्या करता है
(b) कथन-I और कथन-II दोनों सही हैं, किंतु कथन-II, कथन-I की व्याख्या नहीं करता है।
(c) कथन-I सही है, किंतु कथन-II सही नहीं है।
(d) कथन-I सही नहीं है, किंतु कथन-II सही है।

उत्तर: A

व्याख्या:

  • स्वेज़ नहर और सुमेड (SUMED) पाइपलाइन फारस की खाड़ी के कच्चे तेल, पेट्रोलियम उत्पादों और तरलीकृत प्राकृतिक गैस (LNG) नौभार हेतु यूरोप तथा उत्तरी अमेरिका के लिये रणनीतिक मार्ग हैं। अतः कथन I सही है।
  • स्वेज़ नहर और सुमेड पाइपलाइन मिस्र में स्थित हैं और लाल सागर को भूमध्य सागर से जोड़ते हैं। अतः कथन II सही है।
  • अतः कथन I और कथन II दोनों सही हैं तथा कथन II कथन I की व्याख्या करता है। अतः विकल्प A सही है।

Sources:


29. निम्नलिखित कथनों पर विचार कीजिये :

  1. लाल सागर में किसी भी रूप में बहुत कम वर्षण होता है।
  2. लाल सागर में नदियों से जल का प्रवेश नहीं होता है।

उपर्युक्त कथनों में से कौन-सा/कौन-से सही है/हैं?

(a) केवल 1
(b) केवल 2
(c) 1 और 2 दोनों
(d) न तो 1, न ही 2

उत्तर: (c)

व्याख्या:

  • लाल सागर, मिस्र की स्वेज नहर से दक्षिण-पूर्व की ओर लगभग 1,200 मील (1,930 किमी.) तक बाब अल-मंडेब जलडमरूमध्य तक विस्तृत जल की एक संकरा क्षेत्र है, जो अदन की खाड़ी एवं फिर अरब सागर से जुड़ता है। 
  • लाल सागर क्षेत्र में किसी भी रूप में बहुत कम वर्षण होता है, हालाँकि प्रागैतिहासिक कलाकृतियों से संकेत मिलता है कि कभी यहाँ अधिक मात्रा वर्षण भी होता था। अतः कथन 1 सही है।
  • लाल सागर में नदियों से जल का प्रवेश नहीं होता है और वर्षण बहुत कम होता है; लेकिन वाष्पीकरण की क्षति-प्रति वर्ष 80 इंच से अधिक होती है जिसका मुख्य कारण अदन की खाड़ी से बाब अल-मंडेब जलडमरूमध्य के पूर्वी चैनल के माध्यम से वाला अपवाह है। अतः कथन 2 सही है।4

Sources: 

https://www.britannica.com/place/Red-Sea

Drishti IAS Link: https://www.drishtiias.com/daily-updates/daily-news-analysis/red-sea-1 


30. पर्यावरण संरक्षण एजेंसी (EPA) के अनुसार, निम्नलिखित में से कौन-सा, सल्फर डाइऑक्साइड उत्सर्जनों का सबसे बड़ा स्रोत है?

(a) जीवाश्म ईंधनों का उपयोग करने वाले रेल इंजन (लोकोमोटिव)
(b) जीवाश्म ईंधनों का उपयोग करने वाले जहाज़
(c) अयस्कों से धातुओं का निष्कर्षण
(d) जीवाश्म ईंधनों का उपयोग करने वाले विद्युत संयंत्र

उत्तर: (d)

व्याख्या:

वायुमंडल में SO2 का सबसे बड़ा स्रोत विद्युत् संयंत्रों और अन्य औद्योगिक सुविधाओं द्वारा जीवाश्म ईंधन का दहन है। SO2 उत्सर्जन के छोटे स्रोतों में शामिल हैं: अयस्क से धातु निष्कर्षण जैसी औद्योगिक प्रक्रियाएँ; ज्वालामुखी जैसे प्राकृतिक स्रोत तथा रेल इंजन, जहाज़ व अन्य वाहन एवं भारी उपकरण जो उच्च सल्फर सामग्री वाले ईंधन का दहन करते हैं।

अतः विकल्प (d) सही है।

Sources:

https://www.epa.gov/so2-pollution/sulfur-dioxide-basics


31. निम्नलिखित कथनों पर विचार कीजिये:

कथन-I : यदि संयुक्त राज्य अमेरिका (USA) अपने ऋण पर चूक करता है, तो US ट्रेजरी बॉण्डों के धारक भुगतान प्राप्त करने के लिये अपने दावों का प्रयोग नहीं कर पाएँगे।

कथन-II : USA सरकार का ऋण कोई मूर्त (हार्ड) परिसंपत्तियों द्वारा समर्थित नहीं है, बल्कि केवल सरकार के विश्वास से समर्थित है।

उपर्युक्त कथनों के संबंध में, निम्नलिखित में से कौन-सा सही है?

(a) कथन-I और कथन-II दोनों सही हैं तथा कथन-II, कथन-I की व्याख्या करता है
(b) कथन-I और कथन-II दोनों सही हैं, किंतु कथन-II, कथन-I की व्याख्या नहीं करता है
(c) कथन-I सही है, किंतु कथन-II सही नहीं है
(d) कथन-I सही नहीं है, किंतु कथन-II सही है

उत्तर: (d)

व्याख्या:

  • यदि संयुक्त राज्य अमेरिका अपने ऋण पर चूक करता है, तो उस स्थिति में भी अमेरिकी ट्रेजरी बॉण्डों के धारकों को भुगतान प्राप्त करने के लिये अपने दावों का प्रयोग करने का विधिक अधिकार होगा। अतः कथन I सही नहीं है।
  • अमेरिकी सरकार का ऋण, जिसमें ट्रेजरी बॉण्ड, बिल और नोट शामिल हैं, किसी विशिष्ट मूर्त (हार्ड) परिसंपत्तियों द्वारा समर्थित नहीं है। इसके बजाय, यह अमेरिकी सरकार के "पूर्ण विश्वास और साख" द्वारा समर्थित है। अतः कथन II सही है।

अतः विकल्प (d) सही है।

Source:  


32. निम्नलिखित कथनों पर विचार कीजिये :

कथन-I : सामूहिक उधार बहु उधारदाताओं के मध्य उधारकर्त्ता के चूक (डिफाल्ट) का जोखिम बढ़ाता है।

कथन-II : सामूहिक ऋण एक नियत राशि/एकमुश्त निधि हो सकती है, किंतु ऋण व्यवस्था (क्रेडिट लाइन) नहीं हो सकती ।

उपर्युक्त कथनों के संबंध में, निम्नलिखित में से कौन-सा सही है?

(a) कथन-I और कथन-II दोनों सही हैं तथा कथन-II, कथन-I की व्याख्या करता है
(b) कथन-I और कथन-II दोनों सही हैं, किंतु कथन-II, कथन-I की व्याख्या नहीं करता है
(c) कथन-I सही है, किंतु कथन-II सही नहीं है
(d) कथन-I सही नहीं है, किंतु कथन-II सही है

उत्तर: (c)

व्याख्या:

  • एक सामूहिक उधार दो या दो से अधिक बैंकों द्वारा उधारकर्त्ता को दिया जाता है, जिन्हें प्रतिभागियों के रूप में जाना जाता है, जो एकल ऋण समझौते द्वारा नियंत्रित होते हैं। ऋण आमतौर पर एक बैंक, एजेंसी बैंक द्वारा समूह सदस्य की ओर से प्रशासित किया जाता है।  सामूहिक उधार उधारदाताओं के एक समूह द्वारा जारी किये जाते हैं जो एक प्रमुख उधारकर्त्ता, जैसे कि एक फर्म, एक व्यक्तिगत पहल या सरकार को ऋण देने के लिये मिलते हैं। सामूहिक उधार का मुख्य उद्देश्य उधारकर्त्ता के डिफ़ॉल्ट के जोखिम को कई उधारदाताओं, बैंकों या संस्थागत निवेशकों जैसे पेंशन फंड एवं हेज फंड में वितरित करना है। अतः कथन I सही है। ऋण में एक नियत राशि एकमुश्त निधि या ऋण व्यवस्था (क्रेडिट लाइन) हो सकती। अतः  कथन II सही नहीं है।

अतः विकल्प (c) सही है।

https://imarticus.org/blog/debt-capital-markets-and-syndicated-lending/#:~:text=Syndicated%20lending%20primarily%20aims%20to,to%20support%20significant%20business%20deals.

https://www.investopedia.com/terms/s/syndicatedloan.asp


33. डिजिटल रुपए के संबंध में निम्नलिखित कथनों विचार कीजिये :

  1. यह भारतीय रिज़र्व' बैंक (RBI) द्वारा अपनी मौद्रिक नीति के अनुरूप जारी की गई राष्ट्रिक (सॉवरेन) मुद्रा है।
  2. यह RBI के तुलन-पत्र (बैलेंस शीट) पर देयता रूप में दिखाई देता है।
  3. यह अपने डिज़ाइन से ही मुद्रास्फीति के विरुद्ध बीमाकृत है।
  4. यह वाणिज्यिक बैंक मुद्रा और नकदी के लिये स्वतंत्र रूप से परिवर्तनीय है।

उपर्युक्त कथनों में से कौन-से सही हैं?

(a) केवल 1 और 2
(b) केवल 1 और 3
(c) केवल 2 और 4
(d) 1, 2 और 4

उत्तर: (d)

व्याख्या:

डिजिटल रुपया: 

  • डिजिटल मुद्रा से तात्पर्य ऐसी किसी भी मुद्रा से है जो इलेक्ट्रॉनिक रूप में उपलब्ध हो। डिजिटल रुपया आभासी मुद्रा है, जो भौतिक मुद्रा के समान ही उद्देश्य पूरा करती है।
  • डिजिटल रुपया एक केंद्रीकृत डिजिटल मुद्रा है जिसे RBI द्वारा प्रत्यक्ष रूप से विनियमित किया जाता है, जो पारंपरिक मुद्राओं से जुड़ी स्थिरता और विश्वास को बनाए रखता है।
  • डिजिटल टोकन के रूप में डिजिटल मुद्रा (e₹-R) विधि मान्य मुद्रा का प्रतिनिधित्व करती है। 
  • इस प्रकार, डिजिटल मुद्रा भारतीय रिज़र्व बैंक द्वारा अपनी मौद्रिक नीति के अनुरूप जारी की गई एक संप्रभु मुद्रा है। अतः कथन 1 सही है।
  • यह RBI तुलन-पत्र पर एक देयता के रूप में दिखाई देता है। इसे उसी मूल्यवर्ग में जारी किया जाता है जिसमें वर्तमान में कागज़ी मुद्रा और सिक्के ज़ारी किये जाते हैं। अतः कथन 2 सही है।
  • यह अपने डिज़ाइन के कारण मुद्रास्फीति के विरुद्ध बीमाकृत नहीं है। अतः कथन 3 सही नहीं है।
    • भारतीय रिज़र्व बैंक (RBI) ने कहा है कि डिजिटल रुपया मूलतः भौतिक रुपए जैसा ही है, केवल डिजिटल रूप में है और 1:1 के अनुपात में विनिमय योग्य है। रुपए सहित भौतिक मुद्रा भी मुद्रास्फीति के प्रति संवेदनशील है। जैसे-जैसे माल और सेवाओं की कीमत बढ़ती है, रुपए (भौतिक या डिजिटल) की क्रय शक्ति घटती जाती है।
  • यह वाणिज्यिक बैंक की मुद्रा और नकदी के विरुद्ध स्वतंत्र रूप से परिवर्तनीय है। नकदी के मामले में, इस पर कोई ब्याज नहीं मिलेगा और इसे बैंकों में जमा जैसे अन्य प्रकार के धन में परिवर्तित किया जा सकता है। अतः कथन 4 सही है।

Sources:

Drishti IAS Link:

https://www.drishtiias.com/daily-updates/daily-news-analysis/central-bank-digital-currency-2


34. प्राचीन भारत के संदर्भ में, गौतम बुद्ध को सामान्यतः निम्नलिखित में से किन उपनामों से जाना जाता था?

  1. नायपुत्त
  2. शाक्यमुनि
  3. तथागत

नीचे दिये गए कूट का प्रयोग कर सही उत्तर चुनिये :

(a) केवल 1
(b) केवल 2 और 3
(c) 1, 2 और 3
(d) उपर्युक्त में से कोई भी गौतम बुद्ध के उपनाम नही हैं

उत्तर: (b)

व्याख्या:

  • प्राचीन ग्रंथों में महावीर को नायपुत्त (नयस का पुत्र) कहा गया है। यह उनके मूल वंश को संदर्भित करता है, जिसका संस्कृत में अनुवाद ज्ञात्र के रूप में किया जाता है। अतः विकल्प 1 सही नहीं है।
  • बुद्ध (जन्म लगभग 6वीं से 4वीं शताब्दी ईसा पूर्व, लुम्बिनी, कपिलवस्तु के पास, शाक्य गणराज्य, कोसल साम्राज्य [अब नेपाल में] - मृत्यु, कुशीनगर, मल्ल गणराज्य, मगध साम्राज्य [अब कासिया, भारत]) बौद्ध धर्म के संस्थापक थे, जो दक्षिणी-पूर्वी एशिया तथा विश्व के प्रमुख धर्मों एवं दार्शनिक प्रणालियों में से एक है।
  • बुद्ध (जिनके जीवन के बारे में अधिकतर किंवदंतियों के माध्यम से जाना जाता है) के रूप में संदर्भित ऐतिहासिक व्यक्ति का कुल नाम गौतम (संस्कृत में) या गोतम (पाली में) था और साथ ही उनका नाम सिद्धार्थ (संस्कृत: “वह जो अपने लक्ष्य को प्राप्त करता है”) या सिद्धात (पाली में) था। उन्हें प्राय: शाक्यमुनि, “शाक्य वंश का ऋषि” कहा जाता है। बौद्ध ग्रंथों में, उन्हें सबसे अधिक भागवत (प्राय: “भगवान” के रूप में अनुवादित) एवं तथागत के रूप में संबोधित किया जाता है, जिसका अर्थ या तो “ऐसा व्यक्ति जो इस प्रकार आया है” अथवा “ऐसा व्यक्ति जो इस प्रकार चला गया है” हो सकता है।

अतः विकल्प (b) सही है।

Sources:


35. निम्नलिखित सूचना पर विचार कीजिये:

पुरातात्विक स्थल

राज्य

विवरण

1.

चंद्रकेतुगढ़

शैलकृत गुफा मंदिर

व्यापार बंदरगाह शहर

2.

इनामगाँव

महाराष्ट्र

ताम्र-पाषाण स्थल

3.

मगडु

केरल

महापाषाण स्थल

4.

सालिहुंडम

आंध्र प्रदेश

शैलकृत गुफा मंदिर

उपर्युक्त में से कौन-सी पंक्तियों में दी गई सूचना सही सुमेलित है ?

(a) 1 और 2
(b) 2 और 3
(c) 3 और 4
(d) 1 और 4

उत्तर: (b)

व्याख्या:

  • चन्द्रकेतुगढ़ भारत के पश्चिम बंगाल के उत्तर 24 परगना ज़िले में स्थित एक पुरातात्त्विक स्थल है। चन्द्रकेतुगढ़ का नदी मार्ग के निकट स्थित होना यह दर्शाता है कि यह एक महत्त्वपूर्ण व्यापार केंद्र था, जो इस क्षेत्र को भारत के अन्य भागों और संभवतः दक्षिण पूर्व एशिया से जोड़ता था। अतः युग्म 1 सही सुमेलित नहीं है।
  • इनामगांव भारत के महाराष्ट्र के पुणे ज़िले में स्थित एक महत्त्वपूर्ण पुरातात्विक स्थल है। यह दक्कन ताम्रपाषाण (ताम्र युग) संस्कृति को समझने के लिये सबसे महत्त्वपूर्ण स्थलों में से एक है।
  • अतः युग्म 2 सही सुमेलित है।
  • केरल में मंगडू एक नया खोजा गया प्रागैतिहासिक स्थल है जहाँ बड़ी संख्या में महापाषाण पाए जाते हैं। मंगडू महापाषाण का समय लगभग 1000 ईसा पूर्व से 100 ईसा पूर्व तक है। अतः युग्म 3 सही सुमेलित है।
    • मंगडू में महापाषाण स्मारकों में 5 समुच्चय भूमि के क्षेत्र में 28 कठोर संकुल और अपरिष्कृत लेटराइट खंड शामिल थे। शैल खंड मोटे तौर पर क्रमशः 3.5 मीटर, 4.15 मीटर और 5 मीटर व्यास के तीन वृत्त का निर्माण करते हैं।
  • आंध्र प्रदेश के श्रीकाकुलम ज़िले का सालिहुंडम गांव, जो वंशधारा नदी के तट पर स्थित है, उस समय प्रसिद्ध हुआ जब यहाँ पुरातात्त्विक उत्खनन में एक प्राचीन बौद्ध बस्ती के साक्ष्य मिले। यह स्थल अब ASI द्वारा संरक्षित है, यहाँ स्तूप, एक महास्तूप, चैत्य और विहार हैं। हालाँकि, यह स्थल शैलकृत मंदिरों के लिये नहीं जाना जाता है। अतः युग्म 4 सही सुमेलित नहीं है।

Sources:

Drishti IAS Link: NA


36. मध्यकालीन भारत के निम्नलिखित शासकों में से किसने पुर्तगालियों को भटकल में एक किला बनाने की अनुमति दी थी?

(a) कृष्णदेवराय
(b) नरसिम्हा सालुव
(c) मुहम्मद शाह III
(d) यूसुफ आदिल शाह

उत्तर: (a)

व्याख्या:

  • भटकल कर्नाटक के उत्तर कन्नड़ ज़िले के दक्षिणतम छोर पर स्थित है। यह शराबी नदी के दक्षिणी तट पर स्थित है।
  • ऐसा कहा जाता है कि भटकल नाम जैन संत भट्ट अकालंका के नाम पर पड़ा है जो नौवीं शताब्दी में यहाँ निवास करते थे।
  • भारत में पुर्तगालियों के आगमन के साथ ही भटकल चर्चा में रहा। वास्को डी गामा के समय से ही भटकल के सरदार पुर्तगाली शासकों को कर देते थे।
  • सम्राट कृष्णदेवराय ने 1510 ई. में पुर्तगालियों को यहाँ एक किला बनाने की अनुमति प्रदान की थी।

अतः विकल्प (a) सही है।

Sources:

Drishti IAS Link: NA


37. कॉर्नवालिस द्वारा राजस्व संग्रहण के संदर्भ में, निम्नलिखित कथनों पर विचार कीजिये :

  1. राजस्व संग्रहण के रैयतवाड़ी बंदोबस्त के अधीन, किसानों को फसल खराब होने या प्राकृतिक आपदाओं की स्थिति में राजस्व भुगतान से छूट दी गई थी।
  2. बंगाल स्थायी बंदोबस्त के अधीन, यदि जमींदार नियत तिथि पर या उससे पहले राज्य को राजस्व का भुगतान करने में विफल रहता, तो उसे उसकी जमींदारी से हटा दिया जाता था।

उपर्युक्त कथनों में से कौन-सा/कौन-से सही है/हैं ?

(a) केवल 1
(b) केवल 2
(c) 1 और 2 दोनों
(d) न तो 1, न ही 2

उत्तर: (b)

व्याख्या:

  • राजस्व संग्रह के रैयतवाड़ी बंदोबस्त के तहत, खराब फसल या प्राकृतिक आपदाओं की स्थिति में किसानों को राजस्व भुगतान से छूट नहीं दी जाती थी। अतः कथन 1 सही नहीं है।
  • वर्ष 1790 में ज़मींदारों के साथ कर चुकाने का दस वर्षीय समझौता किया गया तथा वर्ष 1793 में इस समझौते को स्थायी बना दिया गया।
    • भूमि के स्वामी के रूप में ज़मींदार उसे बेच सकता था, गिरवी रख सकता था या हस्तांतरित कर सकता था; उसके उत्तराधिकारी अधिकारों और दायित्वों के साथ भूमि को विरासत में प्राप्त कर सकते थे।
    • लेकिन, वर्ष 1794 में शुरू किये गए 'सूर्यास्त विधि' के तहत, यदि किसी निश्चित तिथि के सूर्यास्त तक देय कर का भुगतान नहीं किया जाता है, तो ज़मींदारी को सरकार द्वारा अपने कब्ज़े में ले लिया जाएगा और नीलाम कर दिया जाएगा तथा अधिकार नए मालिक को हस्तांतरित कर दिये जाएंगे। अतः कथन 2 सही है।

अतः विकल्प (b) सही है।

Sources:

  • Spectrum: a-brief-history-of-modern-India (Page-558-562,2020 edition)

Drishti IAS link: https://www.drishtiias.com/to-the-points/paper1/land-revenue-systems-in-british-india


38. निम्नलिखित कथनों पर विचार कीजिये :

  1. उपनिषदों में कोई नीति-कथा (पैरॅबॅल) नहीं हैं।
  2. उपनिषदों की रचना पुराणों से भी पहले हुई थी।

उपर्युक्त कथनों में से कौन-सा/कौन-से सही है/हैं ?

(a) केवल 1
(b) केवल 2
(c) 1 और 2 दोनों
(d) न तो 1, न ही 2

उत्तर: (b)

व्याख्या: 

  • दो पक्षियों के नीति-कथा (पैरॅबॅल) का उल्लेख उपनिषदों से प्राप्त होता है। अतः कथन 1 सही नहीं है।
  • भारतीय धार्मिक इतिहास में दर्शन एवं रहस्यवाद की शुरुआत उपनिषदों के संकलन के काल में हुई, जो लगभग 7वीं से 5वीं ईसा पूर्व के बीच की गई थी।
  • पुराणों के प्रथम संस्करण की रचना संभवतः तीसरी से दसवीं शताब्दी ई. के बीच की गई थी। अतः कथन 2 सही है।

Sources:

Drishti IAS link: https://www.drishtiias.com/blog/vedism


39. निम्नलिखित कथनों पर विचार कीजिये :

  1. भारत इंटरनेशनल ग्रेन्स काउन्सिल का सदस्य है।
  2. चावल और गेहूँ के निर्यात या आयात के लिये किसी देश को इंटरनेशनल ग्रेन्स काउन्सिल का सदस्य होना आवश्यक है।

उपर्युक्त कथनों में से कौन-सा/कौन-से सही है/हैं ?

(a) केवल 1
(b) केवल 2
(c) 1 और 2 दोनों
(d) न तो 1, न ही 2

उत्तर: (a)

व्याख्या: 

इंटरनेशनल ग्रेन्स काउन्सिल (IGC) एक अंतर-सरकारी संगठन है, जिसके निम्नलिखित उद्देश्य हैं:

  • अनाज व्यापार में अंतर्राष्ट्रीय सहयोग को बढ़ावा देना;
  • अनाज क्षेत्र के विस्तार के साथ पारदर्शिता तथा निष्पक्षता को बढ़ावा देना;
  • अनाज बाज़ार की स्थिरता में योगदान देना तथा विश्व खाद्य सुरक्षा को बढ़ावा देना।

भारत इंटरनेशनल ग्रेन्स काउन्सिल का सदस्य है। अतः कथन 1 सही है।

इसमें प्रत्येक सदस्य को अनाज, चावल एवं तिलहन के उसके औसत व्यापार के आधार पर आयातक या निर्यातक के रूप में नामित किया जाता है।

चावल और गेहूँ के निर्यात या आयात के लिये किसी देश को इंटरनेशनल ग्रेन्स काउन्सिल का सदस्य होना आवश्यक नहीं है। अतः कथन 2 सही नहीं है।

Source: https://www.igc.int/en/about/aboutus.aspx

Drishti IAS link: no link


40. यूनेस्को (UNESCO) की अमूर्त सांस्कृतिक विरासत की सूची में निम्नलिखित में से कौन-सा नवीनतम समावेश था?

(a) छऊ (छाऊ) नृत्य
(b) दुर्गा पूजा
(c) गरबा नृत्य
(d) कुंभ मेला

उत्तर: (c)

व्याख्या: 

गरबा एक अनुष्ठानिक और भक्ति नृत्य है जो हिंदू त्योहार (नवरात्रि) के अवसर पर किया जाता है तथा यह नारी शक्ति (Feminine energy) या ऊर्जा हेतु समर्पित है। 

यूनेस्को (UNESCO) की अमूर्त सांस्कृतिक विरासत सूची में नवीनतम समावेश (वर्ष 2023 में) गुजरात का गरबा नृत्य है। अतः विकल्प (c) सही है।

छऊ नृत्य- वर्ष 2010

दुर्गा पूजा- वर्ष 2021

कुंभ मेला- वर्ष 2017

Source: https://ich.unesco.org/en/lists

Drishti IAS link: https://www.drishtiias.com/daily-updates/daily-news-analysis/unesco-recognition-to-gujarat-garba-dance


41. निम्नलिखित कथनों पर विचार कीजिये :

कथन-I: साहेल क्षेत्र में अस्थिरता और बिगड़ती सुरक्षा स्थित्ति विद्यमान है।

कथन-II: हाल ही में साहेल क्षेत्र के कई देशों में सैन्य कब्जा/तख्तापलट हुआ है।

उपर्युक्त कथनों के संबंध में, निम्नलिखित में से कौन-सा सही है?

(a) कथन-I और कथन-II दोनों सही हैं तथा कथन-II, कथन-I की व्याख्या करता है
(b) कथन-I और कथन-II दोनों सही हैं, किंतु कथन-II, कथन-I की व्याख्या नहीं करता है।
(c) कथन-I सही है, किंतु कथन-II सही नहीं है।
(d) कथन-I सही नहीं है, किंतु कथन-II सही है।

उत्तर: (a)

व्याख्या: 

  • नाइज़र, बुर्किना फासो, चाड, माली, मॉरिटानिया और सेनेगल का साहेल क्षेत्र राजनीतिक अस्थिरता और जातीय तनाव का साक्षी रहा है। अतः कथन 1 सही है।
  • नाइज़र एक राजनीतिक उथल-पुथल का सामना कर रहा है क्योंकि वहाँ की सेना ने सरकार का तख्तापलट  वर्ष 2023 में लोकतांत्रिक रूप से निर्वाचित राष्ट्रपति को हटाकर वहाँ अपना  नियंत्रण कर लिया है। अतः कथन 2 सही है।
  • इसलिये, विकल्प (a) सही है। क्योंकि कथन-I और कथन-II दोनों सही हैं तथा कथन-II कथन-I की व्याख्या करता है।

Sources: 

Drishti IAS link: https://www.drishtiias.com/daily-updates/daily-news-analysis/coup-in-niger


42. निम्नलिखित कथनों पर विचार कीजिये :

कथन-I: भारत संयुक्त राज्य अमेरिका से सेब आयात नहीं करता है।

कथन-II : भारत में, विधि के अनुसार, सक्षम प्राधिकारी के अनुमोदन के बिना आनुवंशिक रूप से रूपांतरित खाद्य (फूड) के आयात पर प्रतिषेध है।

उपर्युक्त कथनों के संबंध में, निम्नलिखित में से कौन-सा सही है?

(a) कथन-I और कथन-II दोनों सही हैं तथा कथन-II, कथन-I की व्याख्या करता है
(b) कथन-I और कथन-II दोनों सही हैं, किंतु कथन-II, कथन-I की व्याख्या नहीं करता है।
(c) कथन-I सही है, किंतु कथन-II सही नहीं है।
(d) कथन-I सही नहीं है, किंतु कथन-II सही है।

उत्तर: (d)

व्याख्या: 

  • सितंबर 2023 में भारत द्वारा अमेरिकी सेब पर "रिटेलियेटरी आयात शुल्क (Retaliatory import duty)" हटाने के बाद तीन महीनों में अमेरिकी सेबों का आयात 40 गुना बढ़ गया। अतः कथन 1 सही नहीं है। 
  • भारत में खाद्य सुरक्षा और मानक अधिनियम, 2006 द्वारा FSSAI की स्वीकृति के बिना GM खाद्य के आयात, उत्पादन, उपयोग या बिक्री पर प्रतिबंध लगाया गया है। अतः कथन 2 सही है। 

अतः विकल्प (d) सही है, क्योंकि कथन-I सही नहीं है और कथन-II सही है।

Sources:

Drishti IAS link: https://www.drishtiias.com/daily-updates/daily-news-analysis/fssai-lacks-data-on-genetically-modified-organisms#:~:text=In%20India%2C%20the%20Food%20Safety,%2C%20a%20non%2Dfood%20crop.


43. लोकसभा के अध्यक्ष के संदर्भ में, निम्नलिखित कथनों पर विचार कीजिये :

  1. जब लोकसभा के अध्यक्ष को हटाने का कोई संकल्प विचाराधीन है, तब
  2. वह पीठासीन नहीं होगा/होगी ।
  3. उसे बोलने का अधिकार नहीं होगा ।
  4. वह संकल्प पर प्रथमत: मत देने का/की हकदार नहीं होगा/होगी।

उपर्युक्त कथनों में से कौन-सा/कौन-से सही है/हैं?

(a) केवल 1
(b) केवल 1 और 2
(c) केवल 2 और 3
(d) 1, 2 और 3

उत्तर: (a)  

व्याख्या: 

  • भारत के संविधान के अनुच्छेद 96 के अनुसार, लोकसभा की किसी भी बैठक में, जब उपाध्यक्ष  को उसके पद से हटाने का संकल्प विचाराधीन है तब उपाध्यक्ष, उपस्थित रहने पर भी, पीठासीन नहीं रहेगा और अनुच्छेद 95 के खंड (2) के उपबंध ऐसे ही लागू होंगे जैसे वे उस बैठक के संबंध में लागू होते हैं जिससे, यथास्थिति, उपाध्यक्ष अनुपस्थित है। अतः कथन 1 सही है।
  • भारत के संविधान के अनुच्छेद 96 (2) के अनुसार, जब लोकसभा के अध्यक्ष को हटाने का कोई संकल्प विचाराधीन है, तब उसे लोकसभा में बोलने एवं अन्यथा उसकी कार्यवाही में भाग लेने का अधिकार होगा। अनुच्छेद 100 में किसी प्रावधान के बावजूद वह ऐसे संकल्प पर या ऐसी कार्यवाही के दौरान या किसी अन्य मामले पर केवल प्रथमत: मत देने का/की हकदार होगा/होगी, लेकिन मतों की समानता के मामले में वह मत देने का/की हकदार नहीं होगा/होगी। अतः कथन 2 और 3 सही नहीं हैं।

Source: Constitution of India 

Drishti IAS Link: 


44. भारतीय संसद के संदर्भ में, निम्नलिखित कथनों पर विचार कीजिये :

  1. लोकसभा में लंबित कोई विधेयक उसके विघटन हो जाने पर व्यपगत हो जाता है।
  2. लोकसभा द्वारा पारित और राज्यसभा में लंबित कोई विधेयक लोकसभा के विघटन हो जाने पर व्यपगत हो जाता है।
  3. कोई विधेयक, जिसके संबंध में भारत के राष्ट्रपति ने सदनों की संयुक्त बैठक आहूत करने के अपने आशय की सूचना दे दी है, लोकसभा का विघटन हो जाने पर व्यपगत हो जाता है।

उपर्युक्त कथनों में से कौन-सा/कौन-से सही है/हैं ?

(a) केवल 1
(b) 1 और 2
(c) 2 और 3
(d) केवल 3

उत्तर: (b) 

व्याख्या: 

  • अनुच्छेद 107 के अनुसार, कोई विधेयक जो लोकसभा में प्रस्तुत होकर लंबित रह जाता है, वह सदन के विघटन के साथ ही व्यपगत माना जाता है। अतः कथन 1 सही है।
  • अनुच्छेद 107(5) के अनुसार, कोई विधेयक जो लोकसभा में प्रस्तुत होकर पारित हो जाता है, लेकिन राज्यसभा में लंबित रहता है, उसे भी व्यपगत माना जाता है। अतः कथन 2 सही है।
  • अनुच्छेद 108(5) के अनुसार, इस अनुच्छेद के अंतर्गत संयुक्त बैठक आयोजित की जा सकेगी और उसमें विधेयक पारित किया जा सकेगा, भले ही राष्ट्रपति द्वारा सदनों को उसमें बैठक के लिये बुलाने के अपने इरादे की अधिसूचना दिये जाने के बाद से लोकसभा का विघटन हो गया हो। अतः कथन 3 सही नहीं है।

Source: Constitution of India

Drishti IAS Link: https://www.drishtiias.com/daily-news-analysis/lapsing-of-bills-in-parliament 


45. भारत के संसद के संदर्भ में, निम्नलिखित कथनों पर विचार कीजिये :

  1. भारत के राष्ट्रपति द्वारा किसी सदन के सत्रावसान के लिये मंत्रि-परिषद् के परामर्श की आवश्य नहीं है।
  2. सदन का सत्रावसान सामान्यत: सदन अनिश्चित काल के लिये स्थगित होने के पश्चात किया जाता है, किंतु भारत के राष्ट्रपति द्वारा उस सदन का सत्रावसान करने पर कोई वर्जन नहीं है जो सत्र में है।
  3. लोकसभा का विघटन भारत के राष्ट्रपति द्वारा असाधारण परिस्थितियों को छोड़कर, मंत्रि-परिषद के परामर्श से किया जाता है।

उपर्युक्त कथनों में से कौन-सा/कौन-से सही है/हैं?

(a) केवल 1
(b) 1 और 2
(c) 2 और 3
(d) केवल 3

उत्तर:  (c)

व्याख्या: 

  • संविधान का अनुच्छेद 85(1) राष्ट्रपति को, संसद के प्रत्येक सदन को ऐसे समय एवं स्थान पर सत्र  आहूत करने का अधिकार देता है, जिसे वह उचित समझे, किंतु एक सत्र में सदन की अंतिम बैठक और अगले सत्र में सदन की पहली बैठक के लिये नियत तिथि के बीच छह महीने का अंतर नहीं होगा। 
    • राष्ट्रपति, प्रधानमंत्री या मंत्रिमंडल की सिफारिश पर सदनों को आहूत करने की शक्ति का प्रयोग करता है। अतः कथन 1 सही नहीं है।
  • सत्रावसान सामान्यतः सदन की बैठक के अनिश्चित काल के लिये स्थगित होने के बाद होता है। सदन के अनिश्चित काल के लिये स्थगन और उसके सत्रावसान के बीच का समय अंतराल सामान्यतः दो से चार दिनों का होता है, यद्यपि ऐसे उदाहरण भी हैं जब सदन को उसी दिन स्थगित कर दिया गया जिस दिन उसे अनिश्चित काल के लिये स्थगित किया गया था। यह भी आवश्यक नहीं है कि दोनों सदनों का सत्रावसान एक ही दिन हो। ऐसे उदाहरण हैं, जब एक सदन को उसके अनिश्चित काल के लिये स्थगन पर स्थगित कर दिया गया, जबकि दूसरे सदन को उसके अनिश्चित काल के लिये स्थगन पर स्थगित नहीं किया गया। (राज्यसभा का 141वाँ सत्र जो 23 फरवरी 1987 को शुरू हुआ था, 20 मार्च 1987 को अनिश्चित काल के लिये स्थगित कर दिया गया था और 24 मार्च 1987 को सत्रावसान कर दिया गया था।) अतः कथन 2 सही है।
  • भारत में, लोकसभा का कार्यकाल पाँच वर्ष का होता है, लेकिन इसे पहले भी भंग किया जा सकता है। संविधान के अनुच्छेद 83(2) के अनुसार, इसकी बैठक के पहले दिन से पाँच वर्ष पूरे होने पर निचले सदन को भंग कर दिया जाता है। प्रधानमंत्री की सलाह पर राष्ट्रपति द्वारा निचले सदन को पहले भी भंग किया जा सकता है। इसे तब भी भंग किया जा सकता है जब राष्ट्रपति को लगता है कि किसी सरकार के त्याग-पत्र देने के बाद कोई व्यवहार्य सरकार नहीं बन सकती। अतः कथन 2 सही है।

भारतीय संविधान अनुच्छेद 85 (Article 85 in Hindi) - संसद‌ के सत्र, सत्रावसान और विघटन

(1) राष्ट्रपति समय-समय पर, संसद‌ के प्रत्येक सदन को ऐसे समय और स्थान पर, जो वह ठीक समझे, अधिवेशन के लिये आहूत करेगा, किन्तु उसके एक सत्र की अंतिम बैठक और आगामी सत्र की प्रथम बैठक के लिये नियत तारीख के बीच छह मास का अंतर नहीं होगा।

(2) राष्ट्रपति, समय-समय पर-
(क) सदनों का या किसी सदन का सत्रावसान कर सकेगा;
(ख) लोकसभा का विघटन कर सकेगा।

Sources:

Drishti IAS Link:

https://www.drishtiias.com/to-the-points/Paper2/parliament-part-ii 


46. निम्नलिखित कथनों पर विचार कीजिये :

कथन-I: हाल ही में, यूरोपीय संसद ने द नेट-जीरो इंडस्ट्री ऐव को मंजूरी दी।

कथन-II : यूरोपीय संघ वर्ष 2040 तक कार्बन तटस्थता (न्यूट्रालिटी) प्राप्त करने का आशय रखता है और इसलिये उसका लक्ष्य उस समय तक अपनी सभी स्वच्छ प्रौद्योगिकी विकसित करना है।

उपर्युक्त कथनों के संबंध में, निम्नलिखित में से कौन-सा सही है?

(a) कथन-I और कथन-II दोनों सही हैं तथा कथन-II, कथन-I की व्याख्या करता है
(b) कथन-I और कथन-II दोनों सही हैं, किंतु कथन-II, कथन-I की व्याख्या नहीं करता है।
(c) कथन-I सही है, किंतु कथन-II सही नहीं है।
(d) कथन-I सही नहीं है, किंतु कथन-II सही है।

उत्तर: (c)

व्याख्या: 

  • हाल ही में, यूरोपीय संसद ने वि-कार्बनीकरण के लिये आवश्यक प्रौद्योगिकियों में यूरोपीय संघ के उत्पादन को बढ़ावा देने हेतु नेट-ज़ीरो उद्योग अधिनियम को मंज़ूरी दी। अतः कथन I सही है।
  •  यूरोपीय संघ ने कहा है, "यह वर्ष 2050 तक "जलवायु तटस्थ" होने वाली पहली प्रमुख अर्थव्यवस्था बनने का लक्ष्य बना रहा है।" अतः कथन II सही नहीं है।

Sources:


47. निम्नलिखित कथनों पर विचार कीजिये :

कथन-I: हाल ही में, वेनेज़ुएला अपने आर्थिक संकट से तेज़ी से उबरने में सफल हुआ है और अपने लोगों को दूसरे देशों में पलायन/प्रवास करने से रोकने में सफल हुआ है।

कथन-II : वेनेज़ुएला के पास विश्व के सबसे बड़े तेल भंडार हैं। 

उपर्युक्त कथनों के संबंध में, निम्नलिखित में से कौन-सा सही है?

(a) कथन-I और कथन-II दोनों सही हैं तथा कथन-II, कथन-I की व्याख्या करता है
(b) कथन-I और कथन-II दोनों सही हैं, किंतु कथन-II, कथन-I की व्याख्या नहीं करता है।
(c) कथन-I सही है, किंतु कथन-II सही नहीं है।
(d) कथन-I सही नहीं है, किंतु कथन-II सही है।

उत्तर: (d) 

व्याख्या: 

  • वर्ष 2014 से अब तक 7.72 मिलियन से अधिक लोगों के पलायन के साथ, वेनेज़ुएला में शरणार्थी संकट लैटिन अमेरिका में सबसे बड़ा विस्थापन संकट है और विश्व में सबसे बड़े संकटों में से एक है।
    • 7.72 मिलियन लोगों में से लगभग 84% लैटिन अमेरिका और कैरिबियन में बस गए एवं उन्हें भोजन, आवास तथा आजीविका के लिये महत्त्वपूर्ण चुनौतियों का सामना करना पड़ रहा है। अतः कथन I सही नहीं है।
  • वर्ष 2021 में वेनेज़ुएला में कच्चे तेल का निक्षेप 304 बिलियन बैरल के करीब पहुँच गया, जो विगत् दशक की शुरुआत में बताए गए आँकड़े से तीन गुना से भी अधिक है। बदले में, वेनेज़ुएला ने विश्वभर में सबसे बड़ा कच्चा तेल निक्षेप होने का दावा किया। अतः कथन 2 सही है।

Sources: 


48. डिजिटल इण्डिया भूमि अभिलेख आधुनिकीकरण कार्यक्रम के संदर्भ में, निम्नलिखित कथनों पर विचार कीजिये:

  1. इस योजना को कार्यान्वित करने के लिये, केंद्र सरकार 100% वित्त-पोषण करती है।
  2. इस योजना के अंतर्गत, भूसंपत्ति (कैडस्ट्रल) मानचित्र डिजिटलीकृत किए जाते हैं।
  3. स्थानीय भाषा में उपलब्ध अधिकार अभिलेखों को भारत के संविधान द्वारा मान्यता प्राप्त किसी भी भाषा में लिप्यंतरित करने की पहल की गई है।

उपर्युक्त कथनों में से कौन-से सही हैं?

(a) केवल 1 और 2
(b) केवल 2 और 3
(c) केवल 1 और 3
(d) 1, 2 और 3

उत्तर: (d) 

व्याख्या: 

  • केंद्र सरकार द्वारा पूर्णतः वित्तपोषित डिजिटल इण्डिया भूमि अभिलेख आधुनिकीकरण कार्यक्रम (जिसे पहले राष्ट्रीय भूमि अभिलेख आधुनिकीकरण कार्यक्रम के रूप में जाना जाता था) को 1 अप्रैल, 2016 को पूर्णतः पुनः डिज़ाइन किया गया और साथ ही इसे केंद्रीय क्षेत्रक योजना बना दिया गया। अतः कथन 1 सही है।
  • भूसंपत्ति (कैडस्ट्रल) मानचित्रों का डिजिटलीकरण DILRMP के प्रमुख घटकों एवं गतिविधियों में से एक है। अतः कथन 2 सही है।

  • भूमि प्रशासन में भाषाई बाधाओं की समस्या का समाधान करने के लिये, सरकार ने सेंटर फॉर डेवलपमेंट ऑफ एडवांस्ड कंप्यूटिंग (C-DAC) पुणे के तकनीकी सहयोग से, स्थानीय भाषा में उपलब्ध अधिकारों के अभिलेखों को संविधान की अनुसूची VIII की 22 भाषाओं में से किसी एक में लिप्यंतरित करने की पहल की है। अतः कथन 3 सही है

Sources: 

Drishti IAS Link: 


49. ‘प्रधानमंत्री सुरक्षित मातृत्व अभियान’ के संदर्भ में, निम्नलिखित कथनों पर विचार कीजिये :

  1. यह योजना किसी भी सरकारी स्वास्थ्य सुविधा केंद्र में गर्भावस्था की दूसरी और तीसरी तिमाही में महिलाओं को प्रसवपूर्व चिकित्सा देखभाल सेवाओं के लिये न्यूनतम पैकेज और प्रसवोत्तर छह महीने की स्वास्थ्य चिकित्सा देखभाल सेवा की गारंटी प्रदान करती है।
  2. इस योजना के अंतर्गत, कुछ विशिष्टताओं वाले निजी क्षेत्र के स्वास्थ्य देखभाल सेवा प्रदाता स्वेच्छा से नज़दीकी सरकारी स्वास्थ्य सुविधा केंद्रों में सेवाएँ प्रदान कर सकते हैं।

उपर्युक्त कथनों में से कौन-सा/कौन-से सही है/हैं?

(a) केवल 1
(b) केवल 2
(c) 1 और 2 दोनों
(d) न तो 1, न ही 2

उत्तर: (b)

व्याख्या:

  • प्रधानमंत्री सुरक्षित मातृत्व अभियान (PMSMA) का शुभारंभ प्रत्येक माह की 9 तारीख को सभी गर्भवती महिलाओं (दूसरी और तीसरी तिमाही में) को निश्चित दिन पर सुनिश्चित, व्यापक और गुणवत्तापूर्ण प्रसवपूर्व देखभाल प्रदान करने के लिये किया गया था।
  • अभियान के एक भाग के रूप में, शहरी और ग्रामीण दोनों क्षेत्रों में सरकारी स्वास्थ्य सुविधाओं (PHC/ CHC, DH/शहरी स्वास्थ्य सुविधाएँ आदि) में गर्भवती महिलाओं को उनकी दूसरी/तीसरी तिमाही में प्रसवपूर्व देखभाल सेवाओं का एक न्यूनतम पैकेज प्रदान किया जाता है।
    • लेकिन यह योजना किसी भी सरकारी सुविधा में प्रसव के छह माह बाद स्वास्थ्य देखभाल सेवाएँ प्रदान नहीं करती है। अतः कथन 1 सही नहीं है।
  • निजी/स्वैच्छिक क्षेत्र के डॉक्टरों की सहभागिता को सुविधाजनक बनाने हेतु PMSMA के लिये एक राष्ट्रीय पोर्टल और एक मोबाइल एप्लीकेशन विकसित किया गया है।
    • निजी क्षेत्र के OBGY विशेषज्ञों/रेडियोलॉजिस्ट/चिकित्सकों को सार्वजनिक स्वास्थ्य सुविधाओं पर स्वैच्छिक सेवाएँ प्रदान करने के लिये प्रोत्साहित किया जाएगा जहाँ सरकारी क्षेत्र के चिकित्सक उपलब्ध नहीं हैं या अपर्याप्त हैं। अतः कथन 2 सही है।

अतः विकल्प (b) सही है।


50. प्रधानमंत्री श्रम योगी मान-धन (PM-SYM) योजना के संदर्भ में, निम्नलिखित कथनों पर विचार कीजिये:

  1. इस योजना में नामांकन के लिये प्रवेश आयु वर्ग 21 से 40 वर्ष है।
  2. लाभार्थी द्वारा आयु विशिष्ट अंशदान किया जाएगा।
  3. इस योजना के अंतर्गत प्रत्येक ग्राहक (सब्स्क्राइबर) को 60 वर्ष की आयु प्राप्त करने के बाद `3,000 प्रति माह की न्यूनतम पेंशन प्राप्त होगी।
  4. पारिवारिक पेंशन पति/पत्नी और अविवाहित पुत्रियों पर लागू होगी।

उपर्युक्त कथनों में से कौन-सा/कौन-से सही है/हैं ?

(a) 1, 3 और 4
(b) केवल 2 और 3
(c) केवल 2
(d) 1, 2 और 4

उत्तर: (b)

व्याख्या:

  • प्रधानमंत्री श्रम योगी मान-धन (PM-SYM) योजना 18 से 40 वर्ष की आयु के असंगठित श्रमिकों के लिये एक स्वैच्छिक एवं अंशदायी पेंशन योजना है, जिसमें अधिकतम मासिक आय 15000 रुपए या उससे कम है। अतः कथन 1 सही नहीं है। 
    • इसमें आवेदकों को 60 वर्ष की आयु प्राप्त करने तक 55 रुपए से लेकर 200 रुपए प्रति माह के बीच मासिक अंशदान करना होगा। 
    • आवेदक के 60 वर्ष की आयु प्राप्त करने के बाद, वह पेंशन राशि का दावा कर सकता है। 
  • यह 50:50 के अंशदान वाली एक स्वैच्छिक एवं अंशदायी पेंशन योजना है, जिसमें आयु-विशिष्ट आधार पर लाभार्थी द्वारा निर्धारित अंशदान किया जाएगा और उसके समान ही केंद्र सरकार द्वारा अंशदान किया जाएगा। अतः कथन 2 सही है। 
  • इसके तहत 60 वर्ष की आयु प्राप्त करने पर लाभार्थी को 3000/- रुपए की सुनिश्चित मासिक पेंशन (जैसा भी मामला हो) मिलेगी। अतः कथन 3 सही है। 
  • यदि लाभार्थी की मृत्यु हो जाती है, तो लाभार्थी का जीवनसाथी पारिवारिक पेंशन के रूप में पेंशन का 50% प्राप्त करने का हकदार होगा। इसके तहत पारिवारिक पेंशन केवल जीवनसाथी (अविवाहित बेटियों के लिये नहीं) पर लागू होती है। अतः कथन 4 सही नहीं है।

अतः विकल्प (b) सही है।

Source:

Drishti IAS Link: 

https://www.drishtiias.com/pdf/pradhan-mantri-shram-yogi-maan-dhan-pm-sym.pdf

Other Authentic and Standard Sources: 

https://maandhan.in/show_content.php?lang=1&level=1&ls_id=28&lid=28&page=6

https://himachal.nic.in/WriteReadData/l892s/14_l892s/1559374232.pdf

https://labour.gov.in/pm-sym#:~:text=4.,Government%20as%20per%20the%20chart. 


51. निम्नलिखित कथनों पर विचार कीजिये:

कथन-I : पार्थिव विकिरण की तुलना में आगमी (इनकमिंग) सौर विकिरण से वायुमंडल अधिक गर्म हो जाता है।

कथन-II: वायुमंडल में कार्बन डाइऑक्साइड और अन्य ग्रीनहाउस गैसें दीर्घ तरंग विकिरण के अच्छे अवशोषक हैं।

उपर्युक्त कथनों के संबंध में, निम्नलिखित में से कौन-सा सही है?

(a) कथन-I और कथन-II दोनों सही हैं तथा कथन-II, कथन-I की व्याख्या करता है।
(b) कथन-I और कथन-II दोनों सही हैं, किंतु कथन-II, कथन-I की व्याख्या नहीं करता है।
(c) कथन-I सही है, किंतु कथन-II सही नहीं है।
(d) कथन-I सही नहीं है, किंतु कथन-II सही है।

उत्तर: (d)

व्याख्या:

  • अन्य सभी गर्म वस्तुओं की तरह पृथ्वी भी ऊष्मा उत्सर्जित करती है। इसे पार्थिव विकिरण के नाम से जाना जाता है। लघु तरंगों के माध्यम से आने वाली सौर विकिरण को सूर्यातप कहा जाता है।
    • विकिरण वह प्रक्रिया है जिसके द्वारा सौर ऊर्जा पृथ्वी तक पहुँचती है और साथ ही पृथ्वी बाह्य अंतरिक्ष में ऊर्जा खो देती है।
  • वायुमंडल लघु तरंगों के लिये पारदर्शी एवं दीर्घ तरंगों के लिये अपारदर्शी होता है। इसलिये पृथ्वी की सतह से निकलने वाली ऊर्जा अर्थात् पार्थिव विकिरण, आगमी सौर विकिरण अर्थात् सूर्यातप की तुलना में वायुमंडल को अधिक गर्म करती है। अतः कथन 1 सही नहीं है।
  • दीर्घतरंग विकिरण वायुमंडलीय गैसों, विशेष रूप से कार्बन डाइऑक्साइड तथा अन्य ग्रीनहाउस गैसों द्वारा अवशोषित कर लिया जाता है। अतः कथन 2 सही है।

अतः विकल्प (d) सही है।

Source: 

Drishti IAS Link: 

https://www.drishtiias.com/mains-practice-question/question-360#:~:text=Terrestrial%20Radiation%3A%20The%20earth's%20surface,dioxide%20and%20other%20greenhouse%20gases.

Other Authentic and Standard Sources: https://www.nios.ac.in/media/documents/316courseE/ch10.pdf


52. निम्नलिखित कथनों पर विचार कीजिये :

कथन-I : भूमध्यरेखा (इक्वेटर) पर क्षोभमंडल की मोटाई, ध्रुवों की तुलना में बहुत अधिक है।

कथन-II: भूमध्यरेखा (इक्वेटर) पर, प्रबल संवहनी धाराओं द्वारा ऊष्मा को अधिक ऊँचाई तक ले जाया जाता है।

उपर्युक्त कथनों के संबंध में, निम्नलिखित में से कौन-सा सही है?

(a) कथन-I और कथन-II दोनों सही हैं तथा कथन-II, कथन-I की व्याख्या करता है।
(b) कथन-I और कथन-II दोनों सही हैं, किंतु कथन-II, कथन-I की व्याख्या नहीं करता है।
(c) कथन-I सही है, किंतु कथन-II सही नहीं है।
(d) कथन-I सही नहीं है, किंतु कथन-II सही है।

उत्तर: (a)

व्याख्या:

क्षोभमंडल वायुमंडल की सबसे निचली परत है। इसकी औसत ऊँचाई 13 किमी. है और ध्रुवों के पास यह लगभग 8 किमी. एवं भूमध्य रेखा पर लगभग 18 किमी. की ऊँचाई तक विस्तृत है। भूमध्य रेखा पर क्षोभमंडल सबसे मोटा है तथा उत्तरी और दक्षिणी ध्रुवों पर बहुत पतला है। अतः कथन 1 सही है।

क्षोभमंडल की मोटाई भूमध्य रेखा पर सबसे अधिक होती है क्योंकि प्रबल संवहन धाराओं द्वारा ऊष्मा को बहुत ऊँचाई तक ले जाया जाता है। इसलिये, कथन-I और कथन-II दोनों सही हैं तथा कथन-II, कथन-I की व्याख्या करता है।

अतः विकल्प (a) सही है।

Source: 

Drishti IAS Link: 

https://www.drishtiias.com/sambhav-daily-answer-writing-practice/papers/2023/the-atmosphere-divided-layers-based-temperature-significance-layer-human-life-earth-temperature-inversion-factors-controlling-temperature-distribution-gspaper1-geography/print#:~:text=Its%20average%20height%20is%2013,dust%20particles%20and%20water%20vapour.

Other Authentic and Standard Sources:

https://www.drishtiias.com/images/pdf/NCERT-Class-11-Geography-Part-1.pdf

https://www.researchgate.net/post/Why_is_troposphere_higher_at_equator_but_lower_at_poles_and_reasons_for_lowest_layer_of_atmosphere_being_deeper_at_equator_than_at_the_poles 


53. निम्नलिखित पर विचार कीजिये:

  1. ज्वलखंडाश्मी (पाइरोक्लास्टी) मलबा
  2. राख और धूल
  3. नाइट्रोजन यौगिक
  4. सल्फर यौगिक

उपर्युक्त में से कितने ज्वालामुखी उद्गारों के उत्पाद हैं?

(a) केवल एक
(b) केवल दो
(c) केवल तीन
(d) सभी चार

उत्तर: (d)

व्याख्या:

  • ज्वालामुखीय खतरा किसी भी संभावित खतरनाक ज्वालामुखीय प्रक्रिया को संदर्भित करता है जो मानव जीवन, आजीविका या बुनियादी ढाँचे को क्षति पहुँचाने के जोखिम में डालता है। 
    • ज्वालामुखी के आस-पास के क्षेत्र लावा प्रवाह, ज्वलखंडाश्मी प्रवाह, लहर और भूस्खलन या मलबे का हिमस्खलन आदि से प्रभावित होता है। 
    • ज्वालामुखीय गतिविधि से अन्य खतरे भी उत्पन्न होते हैं, जो ज्वालामुखी से दूर स्थित क्षेत्रों को प्रभावित कर सकते हैं, जैसे- भस्मपात या टेफ्रा और गैस का उत्सर्जन।
  • ज्वालामुखी उद्गार के दौरान उत्सर्जित गैस अणुओं में से 99 प्रतिशत जल वाष्प (H2O), कार्बन डाइऑक्साइड (CO2) और सल्फर डाइऑक्साइड (SO2) होते हैं।
    • शेष एक प्रतिशत हाइड्रोजन सल्फाइड, कार्बन मोनोऑक्साइड, हाइड्रोजन क्लोराइड, हाइड्रोजन फ्लोराइड और अन्य छोटी गैसों की अल्प मात्रा से बना है।
  • ज्वालामुखी प्रत्यक्ष तौर पर नाइट्रोजन के उपयोगी रूप निष्कर्षित नहीं करते हैं। उद्गार के दौरान होने वाली तीव्र गतिविधि से आकाशीय विद्युत गिरने की संभावना हो सकती है।
    • ये आकाशीय विद्युत् वायु में नाइट्रोजन अणुओं (N2) को विघटित कर देते हैं, जिससे नाइट्रोजन परमाणु अन्य तत्त्वों के साथ मिलकर उपयोगी नाइट्रोजन यौगिक बनाते हैं।
  • अतः विकल्प (d) सही है।

Source: 

Drishti IAS Link: Not Available

Other Authentic and Standard Sources:

https://www.usgs.gov/faqs/what-gases-are-emitted-kilauea-and-other-active-volcanoeshttps://www.bgs.ac.uk/discovering-geology/earth-hazards/volcanoes/volcanic-hazards/

https://eos.org/articles/volcanic-lightning-may-have-retooled-the-nitrogen-needed-for-life#:~:text=Volcanoes%20belched%20the%20gases%20that,that%20living%20things%20can%20use.


54. जनवरी माह में समतापी रेखा-मानचित्रों (आइसोथर्मल मैप) से प्राप्त कौन-सा/कौन-से निष्कर्ष सही है/हैं?

  1. समताप रेखाएँ महासागर के ऊपर उत्तर की ओर और महाद्वीप के ऊपर दक्षिण की ओर विचलित हो जाती हैं।
  2. शीत महासागरीय धाराओं, गल्फ स्ट्रीम और उत्तरी अटलांटिक अपवाह (ड्रिफ्ट) की उपस्थिति उत्तरी अटलांटिक महासागर को शीतल बनाती हैं और समताप रेखाएँ उत्तर की ओर मुड़ जाती हैं।

नीचे दिये गए कूट का प्रयोग कर उत्तर चुनिये:

(a) केवल 1
(b) केवल 2
(c) 1 और 2 दोनों
(d) न तो 1, न ही 2

उत्तर: (a)

व्याख्या:

  • समताप रेखाएँ काल्पनिक रेखाएँ हैं जो किसी दिये गए समय पर या किसी दी गई अवधि में समान तापमान वाले बिंदुओं को जोड़ती हैं। 
    • इन रेखाओं का उपयोग मौसम विज्ञान एवं जलवायु विज्ञान में भौगोलिक प्रदेशों में तापमान वितरण का आकलन करने के लिये किया जाता है। 
  • जनवरी में समताप रेखाएँ महासागरीय क्षेत्रों के ऊपर उत्तर की ओर एवं महाद्वीप क्षेत्रों के ऊपर दक्षिण की ओर विचलन करती हैं। ऐसा उत्तरी अटलांटिक महासागरीय क्षेत्र में देखा जा सकता है। अतः कथन 1 सही है।
  • उष्ण महासागरीय धाराओं, गल्फ स्ट्रीम और उत्तरी अटलांटिक अपवाह (ड्रिफ्ट) की उपस्थिति उत्तरी अटलांटिक महासागर को उष्ण बनाती हैं तथा समताप रेखाएँ उत्तर की दक्षिण की ओर विचलन करती हैं अतः कथन 2 सही नहीं है।
  • भूमि पर तापमान तेज़ी से कम हो जाता है और समताप रेखाएँ  यूरोप में दक्षिण की दक्षिण की ओर विचलन करती हैं।

अतः विकल्प (a) सही है।

Source: 

Drishti IAS Link: Not Available

Other Authentic and Standard Sources:

SOLAR RADIATION, HEAT BALANCE AND TEMPERATURE


55. निम्नलिखित में से कौन-से देश विश्व के दो सबसे बड़े कोको उत्पादक के रूप में विख्यात हैं?

(a) अल्जीरिया और मोरक्को
(b) बोत्सवाना और नामीबिया
(c) कोटे डी’आइवर और घाना
(d) मेडागास्कर और मोज़ाम्बीक

उत्तर: (c)

व्याख्या: 

  • कोटे डी’आइवर और घाना दो सबसे बड़े कोको उत्पादक देश हैं, जिनकी वैश्विक कोको उत्पादन में लगभग 60% की हिस्सेदारी है और इसके बाद 9% उत्पादन के साथ इक्वाडोर दूसरे स्थान पर है। 
    • वर्ष 2022 में 2.2 मिलियन टन कोको उत्पादन के साथ कोटे डी आइवर विश्व का सबसे बड़ा उत्पादक रहा, जिसकी कुल वैश्विक उत्पादन में लगभग एक तिहाई (30%) हिस्सेदारी रही।
      • कैडबरी और नेस्ले जैसे जाने-माने ब्रांड ज़्यादातर इसी देश से कोको प्राप्त करते हैं। 
    • घाना कोको का एक अन्य शीर्ष उत्पादक है। कोको का उत्पादन इस देश के सकल घरेलू उत्पाद में प्रमुख योगदान देता है। 
    • एशिया में इंडोनेशिया, कोको का सबसे बड़ा उत्पादक है।
  • पश्चिम अफ्रीका के तहत विश्व के अग्रणी कोको उत्पादक देश शामिल हैं, जिन्होंने वर्ष 2022 में 3.9 मिलियन टन कोको का उत्पादन किया।
  • UN FAO के डेटा के अनुसार शीर्ष कोको उत्पादक देश:

अतः विकल्प (c) सही है।

Source:

Drishti IAS Link: https://www.drishtiias.com/daily-updates/daily-news-analysis/chocolate-industry-meltdown 

Other Authentic and Standard Sources:

https://newdiplomatng.com/the-world-top-cocoa-producing/ 

https://worldpopulationreview.com/country-rankings/cocoa-producing-countries   


56. पश्चिम से पूर्व की ओर प्रयागराज के अनुप्रवाह में गंगा में मिलने वाली हिमालय की नदियों के संदर्भ में, निम्नलिखित में से कौन-सा अनुक्रम सही है?

(a) घाघरा - गोमती - गंडक - कोसी
(b) गोमती  - घाघरा - गंडक कोसी
(c) घाघरा - गोमती - कोसी - गंडक
(d) गोमती - घाघरा - कोसी - गंडक

उत्तर: (b)  

व्याख्या:

  • गंगा नदी प्रणाली भारत की सबसे बड़ी नदी प्रणाली है, जिसमें अनेक बारहमासी और गैर-बारहमासी नदियाँ हैं, जो क्रमशः उत्तर में हिमालय एवं दक्षिणी प्रायद्वीप से निकलती हैं।
  • बाईं तट की महत्त्वपूर्ण सहायक नदियाँ रामगंगा, गोमती, घाघरा, गंडक, कोसी और महानंदा हैं। 
    • अंततः यह नदी सागर द्वीप के पास बंगाल की खाड़ी में गिरती है।
  • गोमती गंगा नदी की एक सहायक नदी और गंगा मैदान की एक जलोढ़ नदी है।
    • यह नदी मैनकोट के पास गोमत ताला झील से निकलती है जिसे माधोटांडा में फुलहर झील के नाम से भी जाना जाता है। यह उत्तर प्रदेश के पीलीभीत शहर से लगभग 30 किलोमीटर की दूरी पर स्थित है। 
  • घाघरा नदी मापचाचुंगो के ग्लेशियरों से निकलती है, अपनी सहायक नदियों टीला, सेती और बेरी से जल एकत्र करती है और शीशपानी में एक गहरे महाखड्ड के माध्यम से पर्वतों से बाहर निकलती है।
  • गंडक नदी में कालीगंडक और त्रिशूलगंगा नदियाँ शामिल हैं। यह धौलागिरी और माउंट एवरेस्ट के बीच नेपाल हिमालय से निकलती है, मध्य नेपाल से होकर बहती है, बिहार के चंपारण ज़िले में गंगा के मैदान में प्रवेश करती है और पटना के पास सोनपुर में गंगा में मिल जाती है।
  • कोसी एक पूर्ववर्ती नदी है, जो तिब्बत में माउंट एवरेस्ट के उत्तर में अपनी मुख्य धारा अरुण के साथ निकलती है।
    • यह नेपाल में मध्य हिमालय को पार करती है, जहाँ यह पश्चिम से सोन कोसी और पूर्व से तमूर कोसी से मिलती है तथा अरुण नदी के साथ विलय के बाद सप्त कोसी का निर्माण करती है।
  • पश्चिम से पूर्व तक सही क्रम है: गोमती, घाघरा, गंडक, कोसी।

अतः विकल्प (b) सही है।

Source:

Drishti IAS link: https://www.drishtiias.com/pdf/1589811013-e-flows-in-river-ganga.pdf  

Other Authentic and Standard Sources:

https://ncert.nic.in/textbook/pdf/kegy103.pdf 

https://www.downtoearth.org.in/news/urbanisation/dte-exclusive-2020-order-on-floodplain-zoning-declaring-gomti-non-perennial-river-draws-flak-90655   https://ncert.nic.in/textbook/pdf/kegy103.pdf   


57. निम्नलिखित कथनों पर विचार कीजिये-

कथन-I : चट्टानों के अपक्षय के कारणों में से एक कारण वर्षा है।

कथन-II: वर्षा जल में घोल के रूप में कार्बन डाइऑक्साइड विद्यमान होता है।

कथन-III : वर्षा जल में वायुमंडलीय ऑक्सीजन विद्यमान होता है।

उपर्युक्त कथनों के संबंध में, निम्नलिखित में से कौन-सा सही है?

(a) कथन-II और कथन-III दोनों सही हैं तथा दोनों, कथन-I की व्याख्या करते हैं
(b) कथन-II और कथन-III दोनों सही हैं, किंतु उनमें से केवल एक, कथन-I की व्याख्या करता है।
(c) कथन-II और कथन-III में से केवल एक सही है और वह कथन-I की व्याख्या करता है।
(d) न तो कथन-II, न ही कथन-III सही है।

उत्तर: (a) 

व्याख्या:  

  • अपक्षय एक ऐसा पद है जो उस सामान्य प्रक्रिया का वर्णन करता है जिसके द्वारा पृथ्वी की सतह पर शैल विघटित होकर अवसाद, चिकनी मृदा, मिट्टी और जल में वियोजित पदार्थों में परिवर्तित हो जाती हैं।
    • अपक्षय जैविक, रासायनिक या भौतिक हो सकता है। 
  • वर्षा और तापमान शैलों के अपक्षय की दर को प्रभावित कर सकते हैं। अधिक वर्षा से रासायनिक अपक्षय की दर बढ़ जाती है। 
    • उष्णकटिबंधीय क्षेत्रों में शैलें प्रचुर वर्षा और गर्म तापमान के संपर्क में आते हैं जो ठंडे, शुष्क क्षेत्रों में पाए जाने वाली समान शैलों की तुलना में बहुत तेज़ी से अपक्षयित होती हैं। अतः कथन I सही है।
  • वायु से कार्बन डाइऑक्साइड वर्षा के जल में वियोजित हो जाती है, जिससे यह थोड़ा अम्लीय हो जाता है। सामान्यतः वर्षा का pH लगभग 5.6 होता है, यह थोड़ा अम्लीय होता है क्योंकि कार्बन डाइऑक्साइड (CO2) इसमें वियोजित होकर दुर्बल कार्बोनिक अम्ल बनाता है। अतः कथन II सही है।
    • जब वर्षा का जल शैल में विद्यमान खनिजों के संपर्क में आता है, तो एक अभिक्रिया हो सकती है, जिससे अपक्षय होता है। 
    • अम्लीय वर्षा का pH सामान्य तौर पर 4.2 और 4.4 के बीच होता है।
  • ऑक्सीजन अपातित वर्षा जल से प्राप्त सबसे महत्त्वपूर्ण तत्त्वों में से एक है।
    • यह वियोजित ऑक्सीजन जल के अणु में विद्यमान ऑक्सीजन के समान नहीं है। वायुमंडल के संपर्क के कारण सभी वर्षा जल और सतही आपूर्ति में वियोजित ऑक्सीजन विद्यमान होती है। अतः कथन III सही है। 
    • वर्षा जल में आमतौर पर सभी प्रकार के विलयित ठोस पदार्थ नहीं होते, लेकिन इसमें विलयित गैसें (कार्बन, नाइट्रोजन और सल्फर के ऑक्साइड) होती हैं, जिसके कारण इसका pH मान लगभग 5.5 या उससे कम होता है।
  • उपर्युक्त से, हम यह निष्कर्ष निकाल सकते हैं कि कथन II और कथन III दोनों सही हैं तथा कथन I की व्याख्या करते हैं।

अतः विकल्प (a) सही है।

Source:  

Drishti IAS Link: Not Available

Other Authentic and Standard Sources:

https://ncert.nic.in/textbook/pdf/kegy206.pdf  

https://www.bgs.ac.uk/discovering-geology/geological-processes/weathering/  

https://www.freedrinkingwater.com/blogs/water-health/710-oxygen-water  

https://www.freedrinkingwater.com/blogs/water-health/710-oxygen-water#:~:text=Oxygen%20gas%20is%20one%20of,to%20contact%20with%20the%20atmosphere.


58. निम्नलिखित देशों पर विचार कीजिये :

  1. फिनलैंड
  2. जर्मनी
  3. नॉर्वे
  4. रूस

उपर्युक्त में से कितने देशों की सीमा उत्तरी समुद्र के साथ लगती है?

(a) केवल एक
(b) केवल दो
(c) केवल तीन
(d) सभी चार

उत्तर: (b)

व्याख्या: 

  • उत्तरी समुद्र, अटलांटिक महासागर के कई समुद्रों में से एक है। यह उत्तर-पूर्वी अटलांटिक क्षेत्र का भाग है और यह इंग्लिश चैनल के मध्यम से अटलांटिक महासागर से मिलता है।  

  • यह अटलांटिक का 13वाँ सबसे बड़ा समुद्र है, जो लगभग 570,000 वर्ग किलोमीटर या महासागर के लगभग 0.5% हिस्से को कवर करता है।
    • यह लगभग 970 किमी. लंबा तथा 580 किमी. चौड़ा है और यूरोप के उत्तर-पश्चिमी महाद्वीपीय किनारों के समानांतर विस्तारित है।
  • उत्तरी सागर क्षेत्र, यूरोप की कुछ सबसे मज़बूत अर्थव्यवस्थाओं के साथ सीमा साझा करता है।
    • इस समुद्र के पश्चिम में ग्रेट ब्रिटेन (स्कॉटलैंड और इंग्लैंड) है।
    • जर्मनी, बेल्जियम, डेनमार्क, नीदरलैंड और नॉर्वे जैसे उत्तरी एवं मध्य यूरोपीय देश पूर्व तथा पश्चिम में इस समुद्र से संलग्न हैं।
  • फिनलैंड और रूस उत्तरी समुद्र के साथ सीमा साझा नहीं करते हैं।

अतः विकल्प (b) सही है।

Source: 

Drishti IAS Link: https://www.drishtiias.com/daily-updates/daily-news-analysis/uk-s-north-sea-drilling  

Other Authentic and Standard Sources:

https://www.worldatlas.com/seas/north-sea.html  


59. निम्नलिखित सूचना पर विचार कीजिये :

जलप्रपात

क्षेत्र

नदी

1.

धुआंधार

मालवा

नर्मदा

2.

हुंड्रू 

छोटा नागपुर

सुवर्णरखा

3.

गेर्सोप्पा

पश्चिमी घाट

नेत्रवती

उपर्युक्त सूचना में से कितनी पंक्तियाँ सही सुमेलित हैं?

(a) केवल एक
(b) केवल दो
(c) सभी तीन
(d) कोई नहीं

उत्तर: (a) 

व्याख्या: 

  • भारत के हृदय में शक्तिशाली नर्मदा द्वारा अपरदित, भेड़ाघाट की संगमरमर की चट्टानों तथा मध्य प्रदेश के जबलपुर ज़िले में धुआँधार जलप्रपात स्थित है। 
    • मालवा (पश्चिम-मध्य उत्तरी भारत का एक क्षेत्र) मध्य प्रदेश राज्य के पश्चिमी भाग में ज्वालामुखीय पठार पर स्थित है। 
    • इस क्षेत्र में मध्य प्रदेश के देवास, धार, इंदौर, झाबुआ, मंदसौर, नीमच, राजगढ़, रतलाम, शाजापुर, उज्जैन और गुना तथा सिहोर के कुछ भागों के साथ राजस्थान के झालावाड़, बाँसवाड़ा एवं चित्तौड़गढ़ ज़िले के कुछ भाग शामिल हैं। 
      • इसमें जबलपुर शामिल नहीं है। इस प्रकार धुआँधार जलप्रपात, हालाँकि नर्मदा नदी पर स्थित है लेकिन मालवा पठार का हिस्सा नहीं है। अतः युग्म 1 सही सुमेलित नहीं है। 
  • हुंड्रू/हुंडरू  जलप्रपात, राँची में सुवर्णरेखा नदी पर है, जहाँ यह 320 फीट की ऊँचाई से गिरता है तथा यह राज्य का सबसे ऊँचा जलप्रपात है। 
    • यह राँची के मुख्य शहर से 45 किलोमीटर की दूरी पर स्थित है।
    • यह जलप्रपात छोटा नागपुर पठार के मनोरम दृश्य को पसंद करने वाले आगंतुकों के लिये आश्चर्य का केंद्र है। अतः युग्म 2 सही सुमेलित है।
  • कर्नाटक का एक प्रमुख आकर्षण केंद्र, गेर्सोप्पा जलप्रपात (जोग जलप्रपात), कर्नाटक के शिमोगा ज़िले में स्थित है। 
    • राजा, रानी, ​​रोवर और रॉकेट के नाम से जाने जाने वाले चार झरने, शरावती नदी (नेत्रवती नदी पर नहीं) पर विशाल जलप्रपात बनाने के क्रम में विलीन हो जाते हैं। अतः युग्म 3 सही सुमेलित नहीं है।
    • शरावती नदी बेसिन पश्चिमी घाट के मध्य भाग में स्थित है।
  • केवल एक युग्म सही सुमेलित है: हुंड्रू-छोटा नागपुर-सुवर्णरेखा

अतः विकल्प (a) सही उत्तर है।

Source:

Drishti IAS link: Not Available 

Other Authentic and Standard Sources:

https://ranchi.nic.in/tourist-place/hundru-waterfall/#:~:text=The%20Hundru%20Falls%20Ranchi%20is%20created%20on%20the%20course%20of,place%20and%20a%20picnic%20spot.   

https://jabalpur.nic.in/en/tourist-place/dhuadhar-water-fall/   

https://kbb.karnataka.gov.in/storage/pdf-files/Completed%20Projects/SCR22_ETR52_Ecological%20Profile%20of%20Sharavathi%20River%20Basin.pdf 


60. निम्नलिखित सूचना पर विचार कीजिये :

क्षेत्र

पर्वत  शृंखला का नाम

पर्वत का प्रकार 

1.

मध्य एशिया

वॉसजेस

वलित पर्वत

2.

यूरोप

आल्प्स

भ्रंशोत्थ (ब्लॉक) पर्वत

3.

उत्तर अमेरिका

अप्लेशियन

वलित पर्वत

4.

दक्षिण अमेरिका

एंडीज़

वलित पर्वत

उपर्युक्त सूचना में से कितनी पंक्तियाँ सही सुमेलित हैं?

(a) केवल एक
(b) केवल दो
(c) केवल तीन
(d) सभी चार

उत्तर: (b)

व्याख्या:  

  • भ्रंशोत्थ पर्वत का निर्माण तब होता हैं जब कोई बड़ा भू-भाग विघटित होता है तथा विघटित भाग  ऊर्ध्वाधर रूप से विस्थापित हो जाते हैं।
    • ऊपर उठे हुए खंड को उत्खंड (होर्स्ट्स) कहा जाता है तथा नीचे धँसे हुए खंड को द्रोणी भ्रंश कहा जाता है।
    • यूरोप (मध्य एशिया नहीं) में राइन घाटी और वॉसजेस पर्वत ऐसी पर्वत प्रणालियों के उदाहरण हैं। अतः युग्म 1 सही सुमेलित नहीं है। 
  • हिमालय पर्वत और यूरोप में आल्प्स ऊबड़-खाबड़ उच्चावच और ऊँची शंक्वाकार चोटियों वाले युवा वलित पर्वत हैं। अतः युग्म 2 सही सुमेलित नहीं है।
  • उत्तरी अमेरिका में अप्लेशियन और रूस में यूराल पर्वत गोलाकार आकृति वाले तथा कम उन्नतांश वाले हैं। वे बहुत पुराने वलित पर्वत हैं। अतः युग्म 3 सही सुमेलित है।
  • एण्डीज़ विश्व की सबसे लंबी पर्वत शृंखला है।
    • वे दक्षिण अमेरिका के पश्चिमी तट के साथ विस्तृत हैं। 
    • यहाँ, नाज़का प्लेट दक्षिण अमेरिकी प्लेट के नीचे की ओर बढ़ रही है। एंडीज़ ज़्यादातर दक्षिण अमेरिकी प्लेट की चट्टानों से वलित हुए हैं। अतः युग्म 4 सही सुमेलित है।
  • अतः केवल दो युग्म सही सुमेलित हैं: उत्तरी अमेरिका-अप्लेशियन-वलित पर्वत और दक्षिणी अमेरिका-एंडीज़-वलित पर्वत।

अतः विकल्प (b) सही है।

Source:

Drishti IAS Link: Not Available

Other Authentic and Standard Sources:

 https://ncert.nic.in/ncerts/l/fess206.pdf 

https://education.nationalgeographic.org/resource/fold-mountain/4th-grade/   


61. जीव ‘‘सिकाडा (साइकेडा), मंडूकफुदक (फ्रॉगहॉपर) और ताल-विसर्पी (पाँड स्केटर)’’ क्या हैं?

(a) पक्षी
(b) मत्स्य
(c) कीट
(d) सरीसृप

उत्तर: (c)

व्याख्या:

  • सिकाडा ऐसे कीट हैं जो अपना अधिकांश जीवन भूमिगत रूप से निम्फ के रूप में बिताने के साथ पेड़ की जड़ों से रस चूसते हैं। नर कीट अपने शरीर के किनारों पर उपस्थित झिल्ली को हिलाकर "गुनगुनाने" की आवाज़ करते हैं।
  • फ्रॉगहॉपर, सुपरफैमिली सर्कोपोइडिया से संबंधित एक कीट है। शीर्ष पर नुकीले होने के साथ इनके शरीर का मध्य भाग संकीर्ण होता है तथा यह कूदने वाले मेंढक की तरह दिखते हैं।
  • पाँड स्केटर (जिसे वॉटर स्ट्राइडर के नाम से भी जाना जाता है) लंबे पादयुक्त वाले कीट समूह हैं जो जल की सतह पर रहते हैं। वे हेमिप्टेरा वर्ग से संबंधित हैं, जिसमें एफिड्स, बेडबग्स और सिकाडा भी शामिल हैं।

अतः विकल्प (c) सही है। 

Sources: 

Drishti IAS Link: https://www.drishtiias.com/daily-updates/daily-news-analysis/cicadas#:~:text=Cicadas%20are%20insects%20that%20belong,have%20two%20pairs%20of%20wings.

Other Authentic and Standard Sources: 

https://naturalhistory.si.edu/education/teaching-resources/life-science/periodical-cicadas

https://www.wlgf.org/froghoppers.html

https://a-z-animals.com/animals/pond-skater/ 


62. निम्नलिखित कथनों पर विचार कीजिये :  

कथन-I: बाज़ार में मिलने वाली अनेक च्यूइंग गम पर्यावरणीय प्रदूषण का स्रोत मानी जाती हैं।

कथन-II : अनेक च्यूइंग गमों में गोंद (गम बेस) के रूप में प्लास्टिक होता है।

उपर्युक्त कथनों के संबंध में, निम्नलिखित में से कौन-सा सही है?

(a) कथन-I और कथन-II दोनों सही हैं तथा कथन-II, कथन-I की व्याख्या करता है।
(b) कथन-I और कथन-II दोनों सही हैं, किंतु कथन-II, कथन-I की व्याख्या नहीं करता है।
(c) कथन-I सही है, किंतु कथन-II सही नहीं है।
(d) कथन-I सही नहीं है, किंतु कथन-II सही है।

उत्तर: (a)

व्याख्या:

  • बाज़ार में मिलने वाली कई च्युइंग गम नॉन-बायोडिग्रेडेबल होती हैं। चिपचिपाहट तथा नॉन-बायोडिग्रेडेबल प्रकृति के कारण गम को पर्यावरण प्रदूषण का स्रोत माना जाता है। अतः कथन-I सही है।
  • कई च्युइंग गम के गम बेस में प्लास्टिक होता है, जो आमतौर पर पॉलीविनाइल एसीटेट (PVA) होता है। प्रतिवर्ष च्युइंग गम से 105 टन से अधिक "प्लास्टिक" कचरा उत्पन्न होता है।
    • इसके अनुसार च्युइंग गम के अवशेष को एक खतरनाक पर्यावरण प्रदूषक माना जा सकता है। अतः कथन-II सही है।
  • कथन-I और कथन-II दोनों सही हैं तथा कथन-II कथन I की सही व्याख्या है।

अतः विकल्प (a) सही है।

Source: 

Drishti IAS Link: Not Available

Other Authentic and Standard Sources: 

https://ecofreek.com/biodegradable/is-gum-biodegradable/ 

https://www.researchgate.net/figure/Percent-of-biodegradability-of-bio-A-and-synthetic-B-chewing-gums-over-a-20-weeks_fig1_325792519#:~:text=Commercially%2C%20chewing%20gum%20is%20produced,as%20a%20dangerous%20environmental%20pollutant

https://gudgum.in/products/raspberry-gum-15-pieces-per-pack-21g#:~:text=Yes%2C%20you%20read%20that%20right,surroundings%20for%20years%20on%20end!


63. निम्नलिखित युग्मों पर विचार कीजिये :

देश अपने प्राकृतिक आवास में पाया जाने वाला जंतु
1.  ब्राज़ील इंड्री
2. इंडोनेशिया एल्क
3. मेडागास्कर बोनोबो

उपर्युक्त युग्मों में से कितने सही सुमेलित हैं?

(a) केवल एक
(b) केवल दो
(c) सभी तीन
(d) कोई नहीं

उत्तर: (d)

व्याख्या:

  • इंड्री, मेडागास्कर का स्थानिक है। ये जंतु मेडागास्कर के उत्तर-पूर्वी भाग में मिलते हैं। यह एक संकटग्रस्त प्रजाति है। अतः युग्म एक सही सुमेलित नहीं है।
  • एल्क (लाल हिरण) उत्तरी अमेरिका तथा मध्य एशिया के ऊँचे पहाड़ों में पाया जाता है।
    • पारंपरिक रूप से इनकी सीमा नॉर्वे में 65 डिग्री उत्तर से लेकर अफ्रीका के 33 डिग्री उत्तर तक है।
    • एल्क आयरलैंड, अर्जेंटीना, चिली, ऑस्ट्रेलिया और न्यूज़ीलैंड में पाया गया है। अतः युग्म दो सही सुमेलित नहीं है।
  • बोनोबो को केवल कांगो लोकतांत्रिक गणराज्य (DRC) में कांगो नदी के दक्षिण में स्थित जंगलों में देखा जा सकता है। अतः युग्म तीन सही सुमेलित नहीं है।

अतः विकल्प (d) सही है।

Source: 

Drishti IAS Link: https://www.drishtiias.com/daily-news-analysis/madagascars-indigenous-species

https://www.drishtiias.com/daily-updates/daily-news-analysis/mining-threatens-african-great-apes

Other Authentic and Standard Sources: 

https://www.britannica.com/animal/indri-lemur-species

https://www.worldwildlife.org/species/bonobo

https://www.britannica.com/animal/elk-mammal

https://animaldiversity.org/accounts/Indri_indri/

https://animaldiversity.org/accounts/Cervus_elaphus/


64. विश्व शौचालय संगठन (वर्ल्ड टॉयलेट ऑर्गनाइज़ेशन) के संबंध में निम्नलिखित कथनों पर विचार कीजिये:

  1. यह संयुक्त राष्ट्र की एजेंसियों में से एक है।
  2. वैश्विक स्वच्छता संकट के समाधान पर कार्रवाई हेतु प्रेरित करने के लिये विश्व शौचालय शिखर सम्मेलन, विश्व शौचालय दिवस और विश्व शौचालय कॉलेज इस संगठन की पहल हैं।
  3. इसके कार्यों का मुख्य लक्ष्य कम विकसित देशों और विकासशील देशों को खुले में शौच की समाप्ति के लक्ष्य की प्राप्ति के लिये निधि प्रदान करना है।

उपर्युक्त कथनों में से कौन-सा/कौन-से सही है/हैं?

(a) केवल 2
(b) केवल 3
(c) 1 और 2
(d) 2 और 3

उत्तर: (a)

व्याख्या:

  • विश्व शौचालय संगठन (WTO) एक वैश्विक गैर-लाभकारी संगठन है जो विश्व भर में शौचालय तथा स्वच्छता की स्थिति में सुधार हेतु प्रतिबद्ध है। इसे वर्ष 2013 में संयुक्त राष्ट्र आर्थिक और सामाजिक परिषद के तहत परामर्शदात्री (Consultative) का दर्ज़ा दिया गया था। अतः कथन 1 सही नहीं है।
  • WTO ने वर्ष 2001 में विश्व शौचालय दिवस तथा विश्व शौचालय शिखर सम्मेलन की शुरुआत की। इसके बाद वर्ष 2005 में विश्व शौचालय कॉलेज की स्थापना की गई। अतः कथन 2 सही है।
  • WTO शौचालय संघों, सरकारों, शैक्षणिक संस्थानों, संयुक्त राष्ट्र एजेंसियों एवं कॉर्पोरेट हितधारकों के बीच जानकारी का आदान-प्रदान करने तथा स्वच्छता और सार्वजनिक स्वास्थ्य नीतियों को बढ़ावा देने के प्रयास में मीडिया एवं कॉर्पोरेट समर्थन का लाभ उठाने हेतु एक अंतर्राष्ट्रीय मंच प्रदान करता है। अतः कथन 3 सही नहीं है।

अतः विकल्प (a) सही है।

Source: 

Drishti IAS Link: 

https://www.drishtiias.com/daily-updates/daily-news-analysis/world-toilet-day-1

Other Authentic and Standard Sources: 

https://worldtoilet.org/web-agency-gb-about-us/ 


65. निम्नलिखित कथनों पर विचार कीजिये :

  1. सिंह की कोई विशेष प्रजनन ऋतु नहीं होती है।
  2. अधिकांश अन्य बड़ी बिल्लियों से भिन्न, चीता दहाड़ता नहीं है।
  3. नर सिंह से भिन्न, नर तेंदुए गंध चिह्न द्वारा अपना क्षेत्र घोषित नहीं करते हैं।

उपर्युक्त कथनों में से कौन-से सही हैं?

(a) केवल 1 और 2
(b) केवल 2 और 3
(c) केवल 1 और 3
(d) 1, 2 और 3

उत्तर: (a)

व्याख्या:

  • सिंह की कोई विशेष प्रजनन ऋतु नहीं होती है। मादा सिंह लगभग हर दो वर्ष में गर्भ धारण करती है
    • इनका औसत गर्भधारण तीन से चार महीने का होता है और आमतौर पर एक से चार शावकों की सीमा होती है। अतः कथन 1 सही है। 
  • चीते "गुर्राने वाली बिल्लियों" के उप-समूह से संबंधित हैं, इसलिये यह दहाड़ते नहीं हैं।
    • "दहाड़ने वाली बिल्लियों" (सिंह, बाघ, जगुआर और तेंदुए) में एक अपूर्ण रूप से अस्थिकृत कंठिका (Hyoid) होती है जिससे ये दहाड़ने में सक्षम होते हैं लेकिन गुर्राने में नहीं। अतः कथन 2 सही है।
  • नर और मादा तेंदुए, दोनों ही अपने क्षेत्रों को चिह्नित करने के लिये मूत्र की गंध का उपयोग करते हैं तथा गंध चिह्न द्वारा अपना क्षेत्र घोषित करते हैं। अतः कथन 3 सही नहीं है। 
    • तेंदुए अक्सर अपने क्षेत्र में घूमते समय अपने चेहरे तथा गर्दन को वनस्पति पर रगड़ते हुए भी देखे जाते हैं। 
      • ऐसा करके तेंदुआ अपनी ग्रंथियों के स्राव के माध्यम से अन्य तेंदुओं को संदेश पहुँचाते हैं।

अतः विकल्प (a) सही है।

Source: 

Drishti IAS Link: Not Available

Other Authentic and Standard Sources: 

https://zooatlanta.org/animal/african-lion/ 

https://seaworld.org/animals/all-about/cheetah/communication/#:~:text=%22Roaring%20cats%22%20

https://www.wildlifeact.com/blog/how-do-leopards-mark-their-range#:~:text=Both%20sexes%20use%20urine%20to,carried%20during%20territorial%20boundary%20patrols


66. निम्नलिखित में से कौन-सा ‘‘100 मिलियन किसानों’’ का सही विवरण है?

(a) यह नेट-शून्य (कार्बन), प्रकृति-सकारात्मक खाद्य और जल प्रणालियों की ओर संक्रमण को तेज़ करने के लिये एक मंच है जिसका लक्ष्य किसानों की पुनरुत्थानशीलता में वृद्धि करना है।
(b) यह जैविक पशुपालन के विकास को सहायता देने एवं सशक्त करने में इच्छुक व्यक्तियों और कृषि संगठनों का एक नेटवर्क और एक अंतरराष्ट्रीय गठबंधन है।
(c) यह एक डिजिटल प्लेटफॉर्म है जो पूरी तरह से सेवा प्रदाताओं के साथ एकीकृत है और ब्लॉक-चेन पर निर्मित है, जो क्रेताओं, विक्रेताओं और तीसरे पक्षकारों को शीघ्र और सुरक्षित रूप से उर्वरकों का व्यापार करने की सुविधा प्रदान करता है।
(d) यह एक मंच है जिसका ध्येय किसानों को कृषक उत्पाद संगठनों अथवा कृषि-व्यवसाय संघ बनाने के लिये प्रोत्साहित करना है, जिससे उन्हें उनके उत्पादों को बेचने के लिये वैश्विक खुले बाज़ारों में पहुँच प्राप्त करने की सुविधा मिलती है।

उत्तर: (a)

व्याख्या:

  • “100 मिलियन किसानों’’ एक ऐसा मंच है जो निजी एवं सार्वजनिक भागीदारों को वैश्विक जलवायु तथा प्रकृति एजेंडे के संदर्भ में खाद्य पदार्थों तथा किसानों को केंद्रीय स्तंभों के रूप में स्थापित करने एवं जलवायु तथा प्रकृति के अनुकूल कृषि प्रथाओं को बढ़ाने के लिये सामूहिक कार्रवाई में तेज़ी लाने में सहायता करता है।
  • इसमें खाद्य और जल प्रणालियों की ओर संक्रमण को तेज़ करने के साहसिक उद्देश्य शामिल हैं जो शुद्ध-शून्य उत्सर्जन के साथ प्रकृति एवं कृषकों के अनुकूल हो।
  • विश्व की कृषक आबादी के पाँचवें हिस्से को लक्षित करते हुए, इस पहल का उद्देश्य एक सौ मिलियन किसानों तक पहुँचना है ताकि एक ऐसे महत्त्वपूर्ण पड़ाव तक पहुँचा जा सके जो खाद्य और जल प्रणालियों के भविष्य को आकार दे सके।

अतः विकल्प (a) सही है। 

Source: 

Drishti IAS Link: Not Available

Other Authentic and Standard Sources: 

https://initiatives.weforum.org/100-million-farmers/home 


67. निम्नलिखित पर विचार कीजिये:

  1. बैटरी भंडारण
  2. बायोमास जेनरेटर
  3. ईंधन सेल
  4. रूफटॉप सौर प्रकाश-वोल्टीय यूनिट

उपर्युक्त में से कितने ‘‘वितरित ऊर्जा संसाधन’’ माने जाते हैं?

(a) केवल एक
(b) केवल दो
(c) केवल तीन
(d) सभी चार

उत्तर: (d)

व्याख्या:

    • वितरित ऊर्जा संसाधन (DER) अक्सर छोटी उत्पादन इकाइयों को संदर्भित करते हैं जो मीटर में उपभोक्ता पक्ष की ओर स्थापित होते हैं।
    • वितरित ऊर्जा संसाधनों के उदाहरणों में  निम्नलिखित शामिल हैं:
      • बैटरी भंडारण
      • बायोमास जनरेटर, जो अपशिष्ट गैस या औद्योगिक और कृषि उप-उत्पादों से ईंधन प्राप्त करते हैं।
      • ईंधन सेल
      • रूफटॉप सौर फोटोवोल्टिक इकाइयाँ।
    • DER के अन्य उदाहरणों में पवन उत्पादन इकाइयाँ, खुले और बंद चक्र वाले गैस टर्बाइन, हाइड्रो एवं मिनी-हाइड्रो स्कीम आदि शामिल हैं।

    अतः विकल्प (d) सही है।

    Source: 

    Drishti IAS Link: 

    https://www.drishtiias.com/daily-news-analysis/draft-policy-framework-for-distributed-renewable-energy

    Other Authentic and Standard Sources: 

    https://www.aemc.gov.au/energy-system/electricity/electricity-system/distributed-energy-resources#:~:text=Distributed%20energy%20resources%20(DER)%20refers,battery%20storage 

    https://www.iea.org/reports/unlocking-the-potential-of-distributed-energy-resources#:~:text=Distributed%20energy%20resources%20(DERs)%20are,solar%20panels%20and%20battery%20storage.

    https://www.nrel.gov/docs/fy02osti/31570.pdf


    68. निम्नलिखित में से कौन-सा वृक्ष, एक ऐसे कीट के साथ एक अद्वितीय संबंध को दर्शाता है जो इस वृक्ष के साथ सह-विकसित हुआ है और वह एकमात्र कीट है जो इस पेड़ को परागित कर सकता है?

    (a) अंजीर
    (b) महुआ
    (c) चंदन
    (d) सेमल (सिल्क कॉटन)

    उत्तर: (a)

    व्याख्या:

    • अंजीर का पेड़ (फिकस माइक्रोकार्पा) अपनी वायवी मूल के लिये प्रसिद्ध है, जो शाखाओं से अंकुरित होता है और अंततः मिट्टी तक पहुँच जाता है।
    • इस वृक्ष का एक कीट के साथ भी अद्वितीय संबंध है जो इसके साथ ही विकसित होता है और एकमात्र कीट है जो इसे परागित कर सकता है।
    • कीट के शरीर का आकार और माप अंजीर के फल के समान ही होता है तथा अंजीर की प्रत्येक प्रजाति अपने विशिष्ट परागणकर्त्ता कीटों को आकर्षित करने के लिये एक विशिष्ट सुगंध उत्पन्न करती है।

    अतः विकल्प (a) सही है।

    Source: 

    Drishti IAS Link: Not Available

    Other Authentic and Standard Sources:

    https://news.illinois.edu/view/6367/1023624754#:~:text=%E2%80%94%20The%20banyan%20fig%20tree%20Ficus,insect%20that%20can%20pollinate%20it


    69. निम्नलिखित पर विचार कीजिये :

    1. तितली
    2. मत्स्य
    3. मंडूक (मेंढक)

    उपर्युक्त में से कितनों में विषाक्त जातियाँ हैं?

    (a) केवल एक
    (b) केवल दो
    (c) सभी तीन
    (d) कोई नहीं

    उत्तर: (c)

    व्याख्या:

    • कई तितलियों में ज़हरीले रसायन होते हैं। इनके चमकीले रंग शिकारियों को खतरे के प्रति आगाह करते हैं।
      • रेड लेसविंग को पराग से लंबा जीवन और विष दोनों ही मिलते हैं तथा पैशन फ्लावर को ग्रहन करने से यह कैटरपिलर की तरह विष का संग्रहण करते हैं।
      • इनके शरीर में पराग, साइनोजेनिक ग्लाइकोसाइड नामक विषाक्त पदार्थों में परिवर्तित हो जाता है। 
    • मत्स्य की विभिन्न प्रजातियों में ज़हरीले बायोटॉक्सिन होते हैं।
      • ग्रुपर्स, बाराकुडा, मोरे ईल, स्टर्जन, सी बास, रेड स्नैपर, एंबरजैक, मैकेरल, पैरट फिश, सर्जनफिश और ट्रिगरफिश जैसी कुछ मछलियाँ विषाक्तता का कारण बन सकती हैं।
    • कुछ मेंढक ज़हरीले हो सकते हैं।
      • ज़हरीले मेंढक अपनी त्वचा में एल्कलॉइड जैसे विषाक्त पदार्थों का उत्पादन एवं भंडारण करते हैं, जिससे उन्हें छूना हानिकारक हो जाता है।
      • उन्हें आमतौर पर पॉईज़न डार्ट मेंढक कहा जाता है।

    अतः विकल्प (c) सही है।

    Source: 

    Drishti IAS Link: Not Available

    Other Authentic and Standard Sources:

    https://www.discoverwildlife.com/animal-facts/insects-invertebrates/poisonous-butterflies 

    https://www.hopkinsmedicine.org/health/conditions-and-diseases/fish-poisoning#:~:text=Certain%20fish%E2%80%94groupers%2C%20barracudas%2C,eating%20moray%20eel%20or%20barracuda.

    https://www.webmd.com/a-to-z-guides/what-to-know-about-poisonous-frogs 

    https://www.discoverwildlife.com/animal-facts/insects-invertebrates/poisonous-butterflies#:~:text=Yet%20they%20seem%20untroubled%20by,warn%20predators%20of%20the%20danger.


    70. निम्नलिखित पर विचार कीजिये:

    1. काजू
    2. पपीता
    3. रक्त चंदन

    उपर्युक्त में से कितने वृक्ष वास्तव में भारत के देशीय वृक्ष हैं?

    (a) केवल एक
    (b) केवल दो
    (c) सभी तीन
    (d) कोई नहीं

    उत्तर: (a)

    व्याख्या: 

    • काजू (एनाकार्डियम ऑक्सीडेंटेल) ब्राज़ील के उष्णकटिबंधीय क्षेत्रों का स्थानिक है।
      • काजू की खोज सबसे पहले वर्ष 1558 के आस-पास ब्राज़ील में यूरोपियों ने की थी।
      • पुर्तगाली लगभग वर्ष 1560 में गोवा में काजू लाए थे। इसलिये यह भारत का स्थानिक नहीं है।
    • पपीते की खेती की उत्पत्ति दक्षिण मैक्सिको और कोस्टा रिका में हुई थी। 16वीं शताब्दी की शुरुआत में इसके उत्पादन को डोमिनिकन गणराज्य और पनामा में देखा गया था। अतः यह भारत का स्थानिक नहीं है।
    • रक्त चंदन भारत का स्थानिक है और केवल पूर्वी घाट के दक्षिणी भागों में इसे देखा जा सकता है। अतः यह भारत का स्थानिक है।

    अतः विकल्प (a) सही है।

    Source: 

    Drishti IAS Link: 

    https://www.drishtiias.com/daily-news-analysis/red-sanders 

    Other Authentic and Standard Sources:

    https://wholesalenutsanddriedfruit.com/history-of-the-cashew/

    https://nhb.gov.in/Horticulture%20Crops/Papaya/Papaya1.htm

    https://www.traffic.org/publications/reports/factsheet-on-indias-red-sanders-in-illegal-wildlife-trade/#:~:text=As%20a%20native%20species%20to,are%20regulated%20by%20the%20State.


    71. निम्नलिखित विमानपत्तनों पर विचार कीजिये :

    1. डोनी पोलो विमानपत्तन
    2. कुशीनगर अंतरराष्ट्रीय विमानपत्तन
    3. विजयवाड़ा अंतरराष्ट्रीय विमानपत्तन

    हाल ही में, उपर्युक्त में से किनका निर्माण नवीन (ग्रीनफील्ड) परियोजनाओं के रूप में किया गया है?

    (a) केवल 1 और 2
    (b) केवल 2 और 3
    (c) केवल 1 और 3
    (d) 1, 2 और 3

    उत्तर: (a)

    व्याख्या:

    • डोनी पोलो विमानपत्तन एक ग्रीनफील्ड विमानपत्तन है जो राजधानी ईटानगर में स्थित है। यह सूर्य (डोनी) और चंद्रमा (पोलो) के प्रति लोगों की श्रद्धा को दर्शाता है तथा राज्य की समृद्ध सांस्कृतिक विरासत का प्रतीक है। 
    • उत्तर प्रदेश के कुशीनगर ज़िले में कुशीनगर विमानपत्तन एक ग्रीनफील्ड विमानपत्तन है।
    • भारतीय विमानपत्तन प्राधिकरण ने आंध्र प्रदेश में विजयवाड़ा हवाई अड्डे का विकास/विस्तार किया है। यह ग्रीनफील्ड परियोजना का हिस्सा नहीं है। 

    अतः विकल्प (a) सही है।

    Source: 

    Drishti IAS Link: https://www.drishtiias.com/daily-news-analysis/kushinagar-international-airport

    Other Authentic and Standard Sources:

    https://www.aai.aero/en/airports/itanagar

    https://sansad.in/getFile/annex/259/AU2087.pdf?source=pqars

    https://pib.gov.in/PressReleasePage.aspx?PRID=1947374 


    72. ‘‘जल वाष्प’’ के संदर्भ में, निम्नलिखित में से कौन-सा/कौन-से कथन सही है/हैं?

    1. यह एक गैस है, जिसकी मात्रा ऊँचाई के साथ घटती है।
    2. ध्रुवों पर इसका प्रतिशत अधिकतम है।

    नीचे दिये गए कूट का प्रयोग कर उत्तर चुनिये :

    (a) केवल 1
    (b) केवल 2
    (c) 1 और 2 दोनों
    (d) न तो 1, न ही 2

    उत्तर: (a)

    व्याख्या:

    • जल वाष्प, जल का गैसीय रूप है। यह पृथ्वी की सबसे प्रचुर ग्रीनहाउस गैस है। 
      • वायुमंडल की जल वाष्प का 99% हिस्सा क्षोभमंडल में होता है। ऊँचाई के साथ जल वाष्प की मात्रा तेज़ी से घटती है। अतः कथन 1 सही है। 
    • जल वाष्प की सांद्रता अक्षांशीय स्थिति (उत्तर से दक्षिण) के साथ बदलती रहती है। इसकी मात्रा उष्णकटिबंधीय क्षेत्रों के ऊपर सबसे अधिक (लगभग 3% तक) होती है और ध्रुवीय क्षेत्रों की ओर जाने पर इसकी मात्रा में कमी आती है। अतः कथन 2 सही नहीं है।

    Source: 

    Drishti IAS Link: Not Available

    Other Authentic and Standard Sources:

    https://earthobservatory.nasa.gov/global-maps/MYDAL2_M_SKY_WV

    https://www.albany.edu/faculty/rgk/atm101/structur.htm 

    https://web.physics.ucsb.edu/~lgrace/chem123/troposphere.htm


    73. निम्नलिखित विवरण पर विचार कीजिये :

    1. तापमानों की वार्षिक और दैनिक सीमा (रेंज) निम्न है।
    2. वर्ष भर वर्षण होता है।
    3. वर्षण में भिन्नता 50cm – 250 cm के मध्य होती है।

    यह किस प्रकार की जलवायु है?

    (a) विषुवतीय जलवायु
    (b) चीन प्रकार जलवायु
    (c) आर्द्र उपोष्णकटिबंधीय जलवायु
    (d) समुद्री पश्चिम तटीय जलवायु

    उत्तर: (d)

    व्याख्या:  

    • समुद्री पश्चिम तटीय जलवायु महाद्वीपों के पश्चिमी तटों पर भूमध्य सागरीय जलवायु से ध्रुवों की ओर पाई जाती है। 
      • इस जलवायु के प्रमुख क्षेत्र हैं- उत्तर-पश्चिमी यूरोप, उत्तरी अमेरिका का पश्चिमी तट, उत्तरी केलिफोर्निया,दक्षिण चिली, दक्षिण-पूर्वी आस्ट्रेलिया और न्यूज़ीलैंड। यहाँ समुद्री प्रभाव के कारण तापमान मध्यम होते हैं और शीत ऋतु में अपने अक्षांशों की तुलना में कोष्ण होते हैं।
      • ग्रीष्म ऋतु में औसत तापमान 15° से 20° सेल्सियस और सर्दियों में 4° से 10° सेल्सियस के बीच रहता है। वार्षिक और दैनिक तापांतर कम पाया जाता हैं। वर्ष भर वर्षा होती है लेकिन यह सर्दियों में अधिक होती है। वर्षण 50 सेमी. से 250 सेमी. के बीच घटती बढ़ती रहती है। 

    अतः विकल्प (d) सही है।

    Source: 

    NCERT CLASS XI (PHYSICAL GEOGRAPHY) | Chapter 11: World Climate and Climate Change | Page No. 94 


    74. ‘‘कोरिऑलिस बल’’ के संदर्भ में, निम्नलिखित में कौन-सा/कौन-से कथन सही है/हैं?

    1. यह पवन वेग की वृद्धि के साथ बढ़ता है।
    2. यह ध्रुवों पर सर्वाधिक है और भूमध्यरेखा (इक्वेटर) पर विद्यमान नहीं होता है।

    नीचे दिये गए कूट का प्रयोग कर उत्तर चुनिये:

    (a) केवल 1
    (b) केवल 2
    (c) 1 और 2 दोनों
    (d) न तो 1, न ही 2

    उत्तर: (c)

    व्याख्या: 

    • पृथ्वी का अपने अक्ष पर घूर्णन पवनों की दिशा को प्रभावित करती है। इसका नाम फ्रांसीसी गणितज्ञ और इंजीनियर गैसपार्ड-गुस्ताव डी कोरिओलिस के नाम पर रखा गया है, जिन्होंने 19वीं शताब्दी की शुरुआत में पहली बार इसका गणितीय वर्णन किया था। यह उत्तरी गोलार्द्ध में पवन को दाईं ओर तथा दक्षिणी गोलार्द्ध में बाईं ओर मोड़ देता है।
      • जब पवनों का वेग अधिक होता है, तब विक्षेपण भी अधिक होता है। अतः कथन 1 सही है।
    • कोरिऑलिस बल अक्षांशों के कोण के समानुपात में बढ़ता है। यह बल ध्रुवों पर सर्वाधिक और भूमध्यरेखा (इक्वेटर) पर अनुपस्थित होता है। 
      • भूमध्यरेखा (विषुवत वृत्त) पर कोरिऑलिस बल शून्य होता है और पवनें समदाब रेखाओं के समकोण पर बहती हैं। अतः निम्न दाब क्षेत्र और अधिक गहन होने की बजाय पूरित हो जाता है। यही कारण है कि विषुवत वृत्त के निकट उष्णकटिबंधीय चक्रवात नहीं बनते। अतः कथन 2 सही है। 

    अतः विकल्प (c) सही है।

    Source: 

    NCERT CLASS XI (PHYSICAL GEOGRAPHY) | Chapter 9 : Atmospheric Circulation and Weather Systems | Page No. 79 

    Drishti IAS Link: 

    https://www.drishtiias.com/sambhav-daily-answer-writing-practice/papers/2023/tri-cellular-model-atmospheric-circulation-emphasis-forces-responsible-model-understand-term-precipitation-world-distribution-rainfall-gspaper1-geography/print#:~:text=Coriolis%20Force%3A%20The%20Coriolis%20force,polar%20cells%20to%20blow%20eastward


    75. प्रत्येक वर्ष 21 जून को, निम्नलिखित में से किस अक्षांश/किन अक्षांशों पर 12 घंटे से अधिक समय तक सूर्य का प्रकाश विद्यमान रहता है?

    1. भूमध्यरेखा (इक्वेटर)
    2. कर्क रेखा
    3. मकर रेखा
    4. उत्तर ध्रुवीय (आर्कटिक) वृत्त

    नीचे दिये गए कूट का प्रयोग कर सही उत्तर चुनिये :

    (a) केवल 1
    (b) केवल 2
    (c) 3 और 4
    (d) 2 और 4

    उत्तर: (d)

    व्याख्या: 

    • 21 जून को उत्तरी गोलार्द्ध सूर्य की तरफ झुका होता है। सूर्य की किरणें कर्क रेखा पर सीधी पड़ती हैं। इसके परिणामस्वरूप इन क्षेत्रों में ऊष्मा अधिक प्राप्त होती है। ध्रुवों के पास वाले क्षेत्रों में कम ऊष्मा प्राप्त होती है, क्योंकि वहाँ सूर्य की किरणें तिरछी पड़ती हैं। उत्तरी ध्रुव सूर्य की तरफ झुका होता है तथा उत्तरी ध्रुव रेखा के बाद वाले भागों पर लगभग 6 महीने तक लगातार दिन रहता है। 
      •  उत्तर अयनांत के दौरान भूमध्य रेखा से जितना अधिक उत्तर की ओर बढ़ा जाए, उतना ही अधिक सूर्य का प्रकाश मिलता है।
    • चूँकि उत्तरी गोलार्द्ध के बहुत बड़े भाग में सूर्य का प्रकाश प्राप्त होता है, इसलिये विषुवत वृत्त के उत्तरी भाग में गर्मी का मौसम होता है। 21 जून को इन क्षेत्रों में सबसे लंबा दिन तथा सबसे छोटी रात होती है। 
      • इस समय दक्षिणी गोलार्द्ध में ये सभी स्थितियाँ विपरीत होती हैं। वहाँ सर्दी होती है। रातें, दिन से बड़ी होती हैं। पृथ्वी की इस स्थिति को उत्तर अयनांत कहते हैं।  

    अतः विकल्प (d) सही है।

    Sources: 

    NCERT CLASS VI | Chapter 3 : Motions of the Earth | Page No. 20 

    https://statesman.com/gcdn/authoring/2019/06/20/NAAS/ghows-TX-8bc81caf-102f-3403-e053-0100007fc66d-751b5106.jpeg 

    Drishti IAS Link: 

    https://www.drishtiias.com/daily-news-analysis/summer-solstice-21st-june 


    76. निम्नलिखित क्षेत्रों में से एक में विश्व की सबसे बड़ी उष्णकटिबंधीय पीट-भूमि है, जो जीवाश्म ईंधन से होने वाले लगभग तीन वर्ष के वैश्विक कार्बन उत्सर्जन को धारण करता है; और जिसके संभाव्य विनाश से वैश्विक जलवायु पर प्रतिकूल प्रभाव पड़ सकता है।

    निम्नलिखित में से कौन-सा उस क्षेत्र को द्योतित करता है?

    (a) अमेज़न बेसिन
    (b) कांगो बेसिन
    (c) किकोरी बेसिन
    (d) रियो डे ला प्लाटा बेसिन

    उत्तर: (b) 

    व्याख्या:  

    • ब्राज़ील और इंडोनेशिया के साथ कांगो बेसिन विश्व की सबसे बड़ी उष्णकटिबंधीय पीट-भूमि का स्थल है। कांगो बेसिन के पीट दलदली वन में लगभग 29 बिलियन टन कार्बन संग्रहण रहता है- जो वैश्विक ग्रीनहाउस गैस उत्सर्जन के तीन वर्ष के बराबर है- जबकि इस पूरे बेसिन में प्रतिवर्ष लगभग 1.5 बिलियन टन कार्बन डाइऑक्साइड का अवशोषण होता है। 
    • यह बेसिन छह देशों में विस्तारित है- कैमरून, मध्य अफ्रीकी गणराज्य, कांगो लोकतांत्रिक गणराज्य, कांगो, इक्वेटोरियल गिनी और गैबॉन।

    अतः विकल्प (b) सही है। 

    Source: 

    https://www.unep.org/news-and-stories/story/critical-ecosystems-congo-basin-peatlands#:~:text=Many%20of%20the%20discussions%20will,along%20with%20Brazil%20and%20Indonesia


    77. कई उपभोक्ता उत्पादों के निर्माण के लिये प्रयुक्त होने वाले परफ्लुओरोऐल्किल और पॉलिफ्लुओरोऐल्किल पदार्थों (PFAS) के संदर्भ में, निम्नलिखित कथनों पर विचार कीजिये :

    1. PFAS पेयजल, खाद्य और खाद्य पैकेजिंग सामग्रियों में व्यापक रूप से पाए जाते हैं।
    2. PFAS पर्यावरण में आसानी से निम्नीकृत (डिग्रेडेड) नहीं होते हैं।
    3. PFAS के लगातार संपर्क के परिणामस्वरूप जंतुओं के शरीर में जैवसंचय हो सकता है।

    उपर्युक्त कथनों में से कौन-से सही हैं? 

    (a) केवल 1 और 2
    (b) केवल 2 और 3
    (c) केवल 1 और 3
    (d) 1, 2 और 3

    उत्तर: (d)

    व्याख्या:

    • परफ्लुओरोऐल्किल और पॉलिफ्लुओरोऐल्किल (PFAS) ऐसे रसायन हैं जो ग्रीस, तेल, जल और गर्मी के प्रतिरोधक हैं। इनका पहली बार उपयोग 1940 के दशक में किया गया था और अब ये दाग-धब्बे तथा जल प्रतिरोधी कपड़ों एवं कालीनों, सफाई उत्पादों, पेंट और अग्निशमन फोम सहित सैकड़ों उत्पादों के भाग हैं। कुछ PFAS को FDA द्वारा कुकवेयर, खाद्य पैकेजिंग एवं खाद्य प्रसंस्करण उपकरणों में सीमित उपयोग हेतु अधिकृत किया गया है।
      • PFAS पेयजल, खाद्य पदार्थों, खाद्य पैकेजिंग सामग्रियों एवं अन्य उपभोक्ता उत्पादों में व्यापक रूप से पाए जाते हैं। अतः कथन 1 सही है।
    • रासायनिक रूप से, PFAS बहुत भिन्न हो सकते हैं। हालाँकि सभी में कार्बन-फ्लोरीन बंध होता है, जो बहुत मज़बूत होता है जिसके कारण ये आसानी से निम्नीकृत (डिग्रेडेड) नहीं होते हैं। अतः कथन 2 सही है।
    • लोग विभिन्न तरीकों से विभिन्न PFAS रासायनिक पदार्थों के संपर्क में आ सकते हैं। समय के साथ लोगों के शरीर से जितने रासायनिक पदार्थों का निष्कासन हो रहा है, उससे ज़्यादा रासायनिक पदार्थ उनके शरीर में पहुँच सकते हैं। यह एक ऐसी प्रक्रिया है जिससे शरीर में जैवसंचय होता है। अतः कथन 3 सही है। 

    अतः विकल्प (d) सही है। 

    Sources:

    Drishti IAS Link: 


    78. निम्नलिखित पर विचार कीजिये:

    1. कैराबिड बीटल्स
    2. कांतर (सेन्टिपीड्स)
    3. मक्खियाँ
    4. दीमक
    5. बर्र (वास्प्स)

    उपर्युक्त जीवों के कितने प्रकार में परजीव्याभ जातियाँ (पैरासीटॉइड स्पीशीज़) पाई जाती हैं?

    (a) केवल दो
    (b) केवल तीन
    (c) केवल चार
    (d) सभी पाँच

    उत्तर: (b)

    व्याख्या:

    • परजीव्याभ जातियाँ (पैरासीटॉइड स्पीशीज) हमेशा ऐसे कीट एवं उनके लार्वा का समूह होते हैं जो भोजन के लिये किसी अन्य पर निर्भर रहते हैं। 
    • परजीव्याभ में बर्र (वास्प्स), मक्खियाँ (जैसे- टैचिनिड मक्खियाँ), बीटल्स (कैराबिड बीटल्स) और कीट (जैसे- गॉर्डियन कीट) की प्रजातियाँ शामिल होती हैं। 
    • कांतर (सेन्टिपीड्स) विशेष रूप से शिकारी होते हैं। ये लगभग हर छोटे और कोमल जंतु (जिसमें कीट एवं अन्य छोटे जीव शामिल होते हैं) को खाते हैं। इसलिये उन्हें परजीव्याभ नहीं माना जा सकता है। 
    • दीमक का भोजन मुख्य रूप से सेल्यूलोज़ होता है जो लकड़ी, घास, पत्तियों, ह्यूमस और वनस्पति मूल की सामग्री (जैसे- कागज़, कार्डबोर्ड, कपास) से प्राप्त होता है। इसलिये दीमक को परजीव्याभ के रूप में वर्गीकृत नहीं किया जा सकता है। 
    • दिये गए विकल्पों में से केवल तीन परजीव्याभ हैं। 

    अतः विकल्प (b) सही है।

    Sources:

    https://www.sciencedirect.com/topics/earth-and-planetary-sciences/parasitoid 

    https://www.britannica.com/science/parasitoid 

    https://australian.museum/learn/animals/insects/predators-parasites-and-parasitoids/

    https://www.britannica.com/animal/termite/Nutrition  

    https://www.sciencedirect.com/topics/agricultural-and-biological-sciences/carabidae 

    https://extension.usu.edu/planthealth/research/centipede 


    79. निम्नलिखित पौधों पर विचार कीजिये :

    1. मूँगफली
    2. कुलथी (हॉर्स-ग्राम)
    3. सोयाबीन

    उपर्युक्त में से कितने मटर कुल (फैमिली) के हैं?

    (a) केवल एक
    (b) केवल दो
    (c) सभी तीन
    (d) कोई नहीं

    उत्तर: (c)

    व्याख्या: 

    • मूँगफली या ग्राउंडनट (अरचिस हाइपोगिया) फली या "बीन" - फैबेसी कुल (आमतौर पर मटर कुल के रूप में जाना जाता है) की एक प्रजाति है। विश्व में भारत, मूँगफली का दूसरा सबसे बड़ा उत्पादक है।
      • हॉर्स ग्राम (मैक्रोटाइलोमायूनिफ्लोरम) फैबेसी कुल से संबंधित है और यह मुख्य रूप से एशियाई एवं अफ्रीकी देशों में उगाई जाने वाली छोटी या कम ज्ञात उपेक्षित फलियों में से एक है।
      • सोयाबीन मटर कुल से संबंधित है और यह खाद्य बीज है। यह आर्थिक रूप से विश्व की सबसे महत्त्वपूर्ण फलियाँ हैं, जो लाखों लोगों को वनस्पति प्रोटीन एवं सैकड़ों रासायनिक उत्पादों हेतु सामग्री प्रदान करती हैं।

    अतः विकल्प (c) सही है।

    Sources:

    https://apeda.gov.in/apedawebsite/SubHead_Products/Ground_Nut.htm 

    https://agriexchange.apeda.gov.in/product_profile/prd_profile.aspx?categorycode=0501#:~:text=The%20groundnut%20belongs%20to%20the,in%20height

    https://www.britannica.com/plant/soybean 

    https://www.researchgate.net/figure/Nutritional-composition-of-horsegram-Macrotyloma-uniflorum-Lam-Verdc_tbl1_283023597 


    80. निम्नलिखित कथनों पर विचार कीजिये:

    कथन-I: इंडियन फ्लाइंग फॉक्स को वन्य जीव (संरक्षण) अधिनियम, 1972 के अधीन ‘पीड़क जंतु’ की श्रेणी में रखा गया है।

    कथन-II : इंडियन फ्लाइंग फॉक्स अन्य जंतुओं का रक्त पीता है।

    उपर्युक्त कथनों के संबंध में, निम्नलिखित में से कौन-सा सही है?

    (a) कथन-I और कथन-II दोनों सही हैं तथा कथन-II, कथन-I की व्याख्या करता है।
    (b) कथन-I और कथन-II दोनों सही हैं, किंतु कथन-II, कथन-I की व्याख्या नहीं करता है।
    (c) कथन-I सही है, किंतु कथन-II सही नहीं है।
    (d) कथन-I सही नहीं है, किंतु कथन-II सही है।

    उत्तर: (c)

    व्याख्या: 

    • पटरोपस गिगेंटस, जिसे आमतौर पर इंडियन फ्लाइंग फॉक्स के रूप में जाना जाता है, भारतीय उपमहाद्वीप की चमगादड़ प्रजाति है।
    • पटरोपस गिगेंटस को फलों के खेतों के प्रति अपनी विनाशकारी प्रवृत्ति के कारण भारतीय वन्यजीव (संरक्षण) अधिनियम, 1972 में 'पीड़क जंतु' के रूप में सूचीबद्ध किया गया है। अतः कथन I सही है।
    • इंडियन फ्लाइंग फॉक्स दुनिया में चमगादड़ों की सबसे बड़ी प्रजातियों में से एक है। इसके पंखों की लंबाई 1.2-1.5 मीटर होती है। ये चमगादड़ दक्षिण मध्य एशिया के लिये स्थानिक हैं।
    • इंडियन फ्लाइंग फॉक्स अपने आहार में कीटों के साथ-साथ फलों को भी शामिल करती हैं, जिसमें रस और मकरंद युक्त फूल भी शामिल हैं। हालाँकि उनके पसंदीदा फल अंजीर हैं, लेकिन यह जंतु  आम, अमरूद, केले और विभिन्न खेती वाले फलों का भी सेवन करते हैं। अतः कथन II सही नहीं है।

    अतः विकल्प (c) सही है।

    Sources: 

    https://timesofindia.indiatimes.com/city/gurgaon/they-may-be-called-vermins-but-fruit-bats-are-vital-for-ecosystem/articleshow/81263203.cms

    https://www.downtoearth.org.in/coverage/bat-tracks-14620

    https://animalia.bio/indian-flying-fox#google_vignette

    https://www.drishtiias.com/daily-updates/daily-news-analysis/indian-flying-fox-bat-pteropus-giganteus


    81. किसी अर्थव्यवस्था में कुल प्रजनन दर को किस रूप है परिभाषित किया जाता है?

    (a) एक वर्ष में जनसंख्या में प्रति 1000 व्यक्तियों पर जन्म लेने वाले बच्चों की संख्या
    (b) किसी दी गई जनसंख्या में एक दंपति के जीवन-काल में उनसे जन्मे बच्चों की संख्या
    (c) जन्म दर घटा मृत्यु दर
    (d) एक महिला की गर्भधारण आयु (चाइल्ड-बेअरिंग एज) के अंत तक उनसे जन्मे जीवित बच्चों की औसत संख्या

    उत्तर: (d)

    व्याख्या: 

    • कुल प्रजनन दर (TFR): सामान्य शब्दों में कुल प्रजनन दर (TFR) का तात्पर्य उन बच्चों की कुल संख्या से है जो किसी महिला के अपने जीवनकाल में पैदा होते हैं या होने की संभावना होती है। दुसरे शब्दों में प्रति महिला पर कुल बच्चों की औसत संख्या TFR कहलाती है

    अतः विकल्प (d) सही है।

    Source: 

    Drishti IAS Link: 


    82. निम्नलिखित कथनों पर विचार कीजिये :

    1. भारत में, गैर-बैंकिंग वित्तीय कंपनियाँ भारतीय रिज़र्व' बैंक की चल-निधि समायोजन सुविधा विंडो का लाभ उठा सकती हैं।
    2. भारत में, विदेशी संस्थागत निवेशक सरकारी प्रतिभूतियों (G-Secs) के धारक बन सकते हैं।
    3. भारत में, शेयर बाज़ार (स्टॉक एक्सचेंज) ऋणों के लिये पृथक् व्यापारिक मंच (ट्रेडिंग प्लेटफॉर्म) प्रदान कर सकते हैं।

    उपर्युक्त कथनों में से कौन-सा/कौन-से सही है/हैं?

    (a) केवल 1 और 2
    (b) केवल 3
    (c) 1, 2 और 3
    (d) केवल 2 और 3

    उत्तर: (d)

    व्याख्या:

    • चलनिधि समायोजन सुविधा (LAF) भारत में RBI द्वारा प्रयुक्त एक मौद्रिक नीति उपकरण है, जिसके माध्यम से वह बैंकिंग प्रणाली में चलनिधि को प्रविष्ट या अवशोषित करता है।
      • LAF का उपयोग बैंकों को आर्थिक अस्थिरता की अवधि के दौरान या उनके नियंत्रण से परे बलों के कारण होने वाले किसी भी अन्य प्रकार के दबाब से निपटने में सहायता के लिये किया जाता है। यद्यपि NBFC बैंकिंग संस्थाएँ नहीं हैं, इसलिये वे RBI की LAF विंडो तक नहीं पहुँच सकते हैं। अतः कथन 1 सही नहीं है।
    • भारतीय रिज़र्व बैंक ने वर्ष 2018 में विदेशी पोर्टफोलियो निवेशकों या FPI को केंद्र सरकार द्वारा निर्गत किये गए राजकोष बिलों में निवेश करने की अनुमति दी थी। यद्यपि, निवेशकों को यह सुनिश्चित करना होगा कि सरकारी प्रतिभूतियों के साथ-साथ एक वर्ष से कम की परिपक्वता वाले कॉरपोरेट बॉण्ड में उनका निवेश कुल निवेश के 20% से अधिक नहीं होना चाहिये।
      • विदेशी निवेशक, चाहे वे विदेशी संस्थागत निवेशक (FII) के रूप में पंजीकृत हों या नहीं, FII मार्ग के बाहर भी भारतीय प्रतिभूतियों में निवेश कर सकते हैं। अतः कथन 2 सही है।
      • FII, अनिवासी भारतीयों (NRI) तथा भारतीय मूल के व्यक्तियों (PIO) को पोर्टफोलियो निवेश योजना (PIS) के माध्यम से भारत में प्राथमिक और द्वितीयक पूंजी बाज़ारों में निवेश करने की अनुमति है।
    • वर्ष 2018 में, नेशनल स्टॉक एक्सचेंज (NSE) ने देश का पहला समर्पित ‘क्रेडिट ट्रेडिंग प्लेटफॉर्म’ शुरू किया।
      • अलग-अलग ऋण ट्रेडिंग प्लेटफॉर्म खुदरा निवेशकों को एक चलनिधि और पारदर्शी एक्सचेंज प्लेटफॉर्म पर कॉर्पोरेट बॉण्ड में निवेश करने का अवसर प्रदान करता है। अतः कथन 3 सही है।
    • अतः विकल्प (d) सही है।

    Sources: 

    Drishti IAS Link: 


    83. भारत में, निम्नलिखित में से कौन कॉपोरेट बॉण्डों और सरकारी प्रतिभूतियों में व्यापार कर सकते हैं?

    1. बीमा कंपनियाँ
    2. पेंशन निधि
    3. खुदरा निवेशक

    नीचे दिये गए कूट का प्रयोग कर सही उत्तर चुनिये :

    (a) केवल 1 और 2
    (b) केवल 2 और 3
    (c) केवल 1 और 3
    (d) 1, 2 और 3

    उत्तर: (d)

    व्याख्या: 

    • वर्ष 2022 के आँकड़ों के अनुसार, बीमा कंपनियाँ प्रीमियम संग्रह में वृद्धि के कारण सरकारी प्रतिभूतियों (G-Secs) पर अधिक निवेश कर रही हैं, जबकि बैंक इन प्रतिभूतियों में अत्यधिक निवेश के कारण इस मोर्चे पर अपेक्षाकृत धीमी गति से आगे बढ़ रहे हैं। RBI के नवीनतम आँकड़ों से पता चलता है कि दिसंबर 2019 को समाप्त तिमाही में 24.9% से दिसंबर 2022 को समाप्त तिमाही में बीमा कंपनियों का सरकारी बॉण्ड में स्वामित्व बढ़कर 26.14% हो गया है।
      • वर्ष 2021 में, भारतीय बीमा विनियामक और विकास प्राधिकरण (IRDAI) ने बीमा कंपनियों को इंफ्रास्ट्रक्चर इन्वेस्टमेंट ट्रस्ट (InvITs) एवं रियल एस्टेट इन्वेस्टमेंट ट्रस्ट (REITS) द्वारा निर्गत ऋण प्रतिभूतियों में निवेश करने की अनुमति दी। इस कदम का उद्देश्य बीमा कंपनियों द्वारा रखे गए पोर्टफोलियो की समग्र उपज में सुधार करना है, साथ ही रियल एस्टेट क्षेत्र को अधिक दीर्घकालिक वित्तपोषण प्रदान करना है।
        •  InvITs और REITS दोनों को सेबी द्वारा विनियमित किया जाता है। 
      • इसलिये, बीमा कंपनियाँ G-Secs और कॉर्पोरेट बॉण्ड दोनों में निवेश कर सकती हैं।
    • पेंशन निधि को ऐसे कॉर्पोरेट बॉण्ड /प्रतिभूतियों में निवेश करने की अनुमति है, जिनकी लागू रेटिंग स्केल में न्यूनतम 'A' रेटिंग या समकक्ष रेटिंग हो।
      • विकास के विभिन्न उपायों के साथ, बाज़ार में सहकारी बैंकों, लघु पेंशन, भविष्य निधि और अन्य निधियों आदि जैसी लघु संस्थाओं का प्रवेश भी देखा गया है। इन संस्थाओं को संबंधित विनियमों के माध्यम से G-Secs में निवेश करना अनिवार्य है।
      • अतः पेंशन निधि G-Secs और कॉर्पोरेट बॉण्ड दोनों में व्यापार कर सकते हैं।
    • खुदरा निवेशक बैंकों और RBI के साथ गिल्ट खाते खोलकर सीधे G-Sec में निवेश कर सकते हैं। रिटेल डायरेक्ट स्कीम खुदरा निवेशकों को प्राथमिक नीलामी में G-Sec खरीदने के साथ-साथ द्वितीयक बाज़ार में G-Sec खरीदने और बेचने की अनुमति देती है।
      • हाल ही में सेबी ने निजी तौर पर रखे जाने वाले कॉरपोरेट बॉण्ड के न्यूनतम टिकट आकार को 1 लाख रुपए से घटाकर 10,000 रुपए कर दिया है। इस कदम से खुदरा निवेशकों के लिये निवेश का एक नया अवसर खुल गया है, जो पहले बाज़ार से बाहर थे।
      • अतः खुदरा निवेशक G-Secs और कॉर्पोरेट बॉण्ड दोनों में व्यापार कर सकते हैं।

    अतः विकल्प (d) सही है।

    Sources: 

    Drishti IAS Link:

    https://www.drishtiias.com/daily-updates/daily-news-analysis/government-securities-2 


    84. निम्नलिखित पर विचार कीजिये :

    1. एक्सचेंज ट्रेडेड फंड (ETF)
    2. मोटर वाहन
    3. मुद्रा की अदला-बदली

    उपर्युक्त में से किसे/किन्हें वित्तीय लिखत (इंस्ट्रूमेंट) माना जाता है?

    (a) केवल 1
    (b) केवल 2 और 3
    (c) 1, 2 और 3
    (d) केवल 1 और 3

    उत्तर: (d)

    व्याख्या: 

    • वित्तीय साधन अमूर्त परिसंपत्तियाँ हैं, जिनसे भविष्य में नकदी के दावे के रूप में लाभ मिलने की उम्मीद की जाती है।
    • इन लिखतों को दो प्रकारों में विभाजित किया जा सकता है: नकद लिखत और व्युत्पन्नी लिखत या ऋण लिखत या इक्विटी लिखत जैसे परिसंपत्ति वर्ग के आधार पर विभाजित किया जा सकता है। तीसरी विशिष्ट श्रेणी विदेशी मुद्रा लिखतों की है।
    • वित्तीय लिखतों के उदाहरणों में स्टॉक, एक्सचेंज-ट्रेडेड फंड (ETF), बॉण्ड, जमा प्रमाण-पत्र (CD), म्यूचुअल फंड, ऋण और व्युत्पन्नी अनुबंध आदि शामिल हैं। अतः 1 सही है।
      • e-IMF लाइब्रेरी के अनुसार, विगत दो दशकों में, औद्योगिक देशों में केंद्रीय बैंकों की बढ़ती संख्या ने घरेलू चलनिधि को संतुलित करने के लिखतों में विदेशी मुद्रा स्वैप को शामिल किया है, भले ही अधिकांश देशों में वास्तविक उपयोग सीमित रहा हो। अतः 3 सही है।
        • विदेशी मुद्रा लिखतों में मुद्रा स्वैप, विदेशी मुद्रा विकल्प, विदेशी मुद्रा स्वैप शामिल हैं और ये मुख्य रूप से मुद्राओं से संबंधित हैं।
    • यद्यपि मोटर वाहन एक मूर्त परिसंपत्ति है, इसलिये यह वित्तीय लिखत नहीं है। अतः 2 सही नहीं है।

    अतः विकल्प (d) सही है।

    Sources: 


    85. भारतीय अर्थव्यवस्था के सेक्टरों के संदर्भ में निम्नलिखित युग्मों पर विचार कीजिये :

    आर्थिक गतिविधि            सेक्टर

    1. कृषि उत्पाद का भंडारण - द्वितीयक
    2. डेरी फार्म                 - प्राथमिक
    3. खनिज की खोज         -  तृतीयक
    4. कपड़ा बुनाई             -  द्वितीयक

    उपर्युक्त युग्मों में से कितने सही सुमेलित हैं?

    (a) केवल एक
    (b) केवल दो
    (c) केवल तीन
    (d) सभी चार

    उत्तर: (b)

    व्याख्या:

    • भारतीय अर्थव्यवस्था में कृषि उपज के भंडारण को तृतीयक या सेवा क्षेत्र में रखा गया है। तृतीयक या सेवा क्षेत्र उत्पादन की प्रक्रिया को सहायता प्रदान करता है। इसमें उत्पादों का परिवहन, भंडारण, विपणन और बिक्री शामिल है। अतः युग्म 1 सही नहीं है।
    • डेयरी क्षेत्र प्राथमिक क्षेत्र के अंतर्गत आता है। प्राथमिक गतिविधियाँ प्रत्यक्ष रूप से पर्यावरण पर निर्भर होती हैं क्योंकि ये पृथ्वी के संसाधनों के उपयोग को संदर्भित करती हैं। अतः युग्म 2 सही है।
    • भारतीय अर्थव्यवस्था में खनिज अन्वेषण एक प्राथमिक क्षेत्र है। प्राथमिक क्षेत्र में कच्चे माल का निष्कर्षण और कृषि एवं संबद्ध गतिविधियाँ शामिल हैं। अतः युग्म 3 सही नहीं है।
    • कपड़ों की बुनाई द्वितीयक गतिविधि के अंतर्गत आती है। इस क्षेत्र में तैयार माल का उत्पादन शामिल है। अतः युग्म 4 सही है।

    अतः विकल्प (b) सही है।


    86. निम्नलिखित सामग्रियों पर विचार कीजिये:

    1. कृषि अवशिष्ट
    2. मक्का के दाने
    3. अपशिष्ट जल शोधन अवपंक
    4. काष्ठ मिल अपशिष्ट

    उपर्युक्त में से किसका उपयोग संधारणीय विमानन ईंधन के उत्पादन के लिये फीडस्टॉक के रूप में किया जा सकता है?

    (a) केवल 1 और 2
    (b) केवल 3 और 4
    (c) 1, 2, 3 और 4
    (d) केवल 1, 3 और 4

    उत्तर: (c)

    व्याख्या:

    संधारणीय विमानन ईंधन के उत्पादन के लिये संधारणीय फीडस्टॉक्स की सूची:

    • मकई अनाज
    • तिलहन
    • शैवाल
    • अन्य वसा, तेल और ग्रीस
    • कृषि अवशिष्ट
    • वन अवशिष्ट
    • काष्ठ मिल अपशिष्ट
    • नगरपालिका ठोस अपशिष्ट धाराएँ
    • आर्द्र अपशिष्ट (खाद, अपशिष्ट जल, अपशिष्ट जल शोधन अवपंक)
    • समर्पित ऊर्जा फसलें

     अतः विकल्प (c) सही है।


    87. भारतीय अर्थव्यवस्था में भौतिक पूँजी के संदर्भ में, निम्नलिखित युग्मों पर विचार कीजिये :

    वस्तु श्रेणी
    1. किसान का हल कार्यशील पूँजी
    2. कंप्यूटर स्थिर पूँजी
    3. बुनकर द्वारा प्रयोग किया जाने वाला सूत स्थिर पूँजी
    4. पेट्रोल कार्यशील पूँजी

    उपर्युक्त युग्मों में से कितने सही सुमेलित हैं?

    (a) केवल एक
    (b) केवल दो
    (c) केवल तीन
    (d) सभी चार

    उत्तर: (b)

    व्याख्या:

    भौतिक पूंजी से तात्पर्य उन परिसंपत्तियों से है,जो किसी संगठन के स्वामित्व में होती हैं तथा उसी के द्वारा उपयोग में लाई जाती हैं जैसे भवन, मशीनरी और वाहन इत्यादि।

    • उपकरण, मशीनें, इमारतें, हल, जनरेटर, टर्बाइन, कंप्यूटर आदि का उपयोग कई वर्षों तक उत्पादन में किया जा सकता है और इसलिये उन्हें स्थिर पूंजी कहा जाता है। खेती का हल स्थिर भौतिक पूंजी का एक उदाहरण है। हल एक कृषि उपकरण है जिसका उपयोग फसल बोने की तैयारी में मिट्टी को ढीला करने और जोतने के लिये किया जाता है। अतः युग्म 1 सही नहीं है।
    • कंप्यूटर को व्यवसाय के लिये एक स्थिर परिसंपत्ति माना जाता है क्योंकि यह लंबे समय तक व्यवसाय की सेवा करता है। स्थिर पूंजी एक व्यवसाय की ऐसी परिसंपत्तियाँ हैं जो प्रकृति में स्थिर होती हैं और व्यवसाय द्वारा उनका निपटान नहीं किया जाता है। इन परिसंपत्तियों में भूमि, भवन, संयंत्र, मशीनरी, स्थिर उपकरण, फर्नीचर, फिक्सचर, वाहन, पशुधन आदि शामिल हैं। अतः युग्म 2 सही है।
    • उत्पादन के लिये कई तरह के कच्चे माल की ज़रूरत होती है, जैसे- बुनकर द्वारा इस्तेमाल किया जाने वाला धागा, मशीनों और परिवहन में प्रयुक्त पेट्रोल या कुम्हार द्वारा इस्तेमाल की जाने वाली मिट्टी। साथ ही, उत्पादन के दौरान भुगतान करने और अन्य ज़रूरी सामान खरीदने के लिये हमेशा कुछ नकदी की ज़रूरत होती है। कच्चे माल और नकद को कार्यशील पूंजी कहा जाता है। अतः युग्म 3 सही नहीं है परंतु  युग्म 4 सही है।

    Source- NCERT


    88. निम्नलिखित में से कौन-सा शब्द/वाक्यांश का उपयोग “3D आभासी (वर्चुअल) दुनिया के एक इंटरऑपरेबल नेटवर्क, जिसे ऐसे लाखों प्रयोक्ताओं द्वारा एक साथ एक्सेस किया जा सकता है, जो आभासी (वर्चुअल) वस्तुओं पर संपत्ति के अधिकारों का प्रयोग कर सकते हैं’’ को द्योतित करने के लिये सर्वाधिक उपयुक्त रूप से किया जाता है?

    (a) बिग डेटा एनालिटिक्स
    (b) क्रिप्टोग्राफी
    (c) मेटावर्स
    (d) वर्चुअल मैट्रिक्स

    उत्तर: (c)

    व्याख्या:

    • मेटावर्स एक उभरता हुआ 3D सक्षम डिजिटल स्पेस है जो आभासी वास्तविकता (VR), संवर्द्धित वास्तविकता और अन्य उन्नत इंटरनेट एवं अर्द्धचालक प्रौद्योगिकी का उपयोग करता है ताकि लोगों को ऑनलाइन वास्तविक व्यक्तिगत तथा व्यावसायिक अनुभव प्राप्त हो सके।
    • मेटावर्स से तात्पर्य एक साझा वातावरण से है जो अनेक 3D आभासी क्षेत्रों में फैला हुआ है। 
    • मेटावर्स को लाखों प्रयोक्ताओं द्वारा एक साथ एक्सेस किया जा सकता है, जो आभासी (वर्चुअल) वस्तुओं पर संपत्ति के अधिकारों का प्रयोग कर सकते हैं। 

    अतः विकल्प (c) सही है।


    89. विदेशी बैंकों के साथ व्यवहार करते समय भारतीय रिज़र्व बैंक द्वारा अधिरोपित नियम/नियमों के संदर्भ निम्नलिखित कथनों पर विचार कीजिये :

    1. भारत में पूर्ण स्वामित्व वाले सहायक बैंको (बैंकिंग सब्सिडरियों) के लिये कोई न्यूनतम पूँजी की आवश्यकता नहीं है।
    2. भारत में पूर्ण स्वामित्व वाले सहायक बैंकों (बैंकिंग सब्सिडरियों) के लिये, बोर्ड सदस्यों के कम-से-कम 50% भारतीय नागरिक होने चाहिये। 

    उपर्युक्त कथनों में से कौन-सा/कौन-से सही है/हैं?

    (a) केवल 1
    (b) केवल 2
    (c) 1 और 2 दोनों
    (d) न तो 1, न ही 2

    उत्तर: (b)

    व्याख्या:

    भारत में विदेशी बैंकों द्वारा पूर्ण स्वामित्व वाली सहायक कंपनियों (WOS) के लिये नियम:

    • पूर्ण स्वामित्व वाली सहायक कंपनी के लिये आरंभिक न्यूनतम प्रदत्त मताधिकारी इक्विटी पूंजी 5 बिलियन रुपए होगी। अतः कथन 1 सही नहीं है।
    • यह सुनिश्चित करने के लिये कि भारत में स्थापित विदेशी बैंक की पूर्ण स्वामित्व वाली सहायक कंपनी के निदेशक मंडल स्थानीय संस्था के सर्वोत्तम हित में कार्य करते हैं, RBI अन्य देशों में सर्वोत्तम प्रथाओं के अनुरूप यह आदेश दे सकता है कि 50 प्रतिशत से कम निदेशक भारत में रहने वाले भारतीय नागरिक न हों। अतः कथन 2 सही है।

    अतः विकल्प (b) सही है।

    Source: RBI

     Source:RBI


    90. भारत में निगमित सामाजिक उत्तरदायित्व (CSR) नियमों के संदर्भ में, निम्नलिखित कथनों पर विचार कीजिये :

    1. CSR नियम विनिर्दिष्ट करते हैं कि सीधे कंपनी अथवा इसके कर्मचारियों को लाभ पहुँचाने वाले व्यय को CSR कार्यकलापों के रूप में नहीं माना जाएगा।
    2. CSR नियम CSR कार्यकलापों पर होने वाले न्यूनतम व्यय को विनिर्दिष्ट नहीं करते हैं।

    उपर्युक्त कथनों में से कौन-सा/कौन-से सही है/हैं?

    (a) केवल 1
    (b) केवल 2
    (c) 1 और 2 दोनों
    (d) न तो 1, न ही 2

    उत्तर: (a)

    व्याख्या: 

    • कंपनी (CSR नीति) नियम, 2014 के नियम 2(1)(d)(iv) में कहा गया है कि कंपनी के कर्मचारियों को लाभ पहुँचाने वाली कोई भी गतिविधि अर्हत CSR गतिविधि नहीं मानी जाएगी। नियम के अनुसार, कर्मचारियों के लाभ के लिये विशेष रूप से अभिकल्पित की गई कोई भी गतिविधि "कर्मचारियों को लाभ पहुँचाने वाली गतिविधि" मानी जाएगी और स्वीकार्य CSR व्यय के रूप में योग्य नहीं होगी। अतः कथन 1 सही है।
    • प्रत्येक कंपनी, जिसके लिये CSR प्रावधान लागू होते हैं, के निदेशक मंडल को यह सुनिश्चित करना होगा कि कंपनी अपनी CSR नीति के अनुसार प्रत्येक वित्तीय वर्ष में तत्काल पूर्ववर्ती तीन वित्तीय वर्षों के दौरान अर्जित औसत शुद्ध लाभ का कम-से-कम 2% व्यय करे। अतः कथन 2 सही नहीं है।

    अतः विकल्प (a) सही है।

    source-CLEARTAX

    Source- MCA


    91. विकिरण समस्थानिक ताप-वैद्युत जेनरेटरों (RTGs) के संदर्भ में, निम्नलिखित कथनों पर विचार कीजिये:

    1. RTGs लघु विखंडन रिऐक्टर हैं।
    2. RTGs का प्रयोग अंतरिक्षयानों के ऑन-बोर्ड प्रणालियों को विद्युत आपूर्ति करने के लिये होता है।
    3. RTGs में प्लूटोनियम-238 का उपयोग किया जा सकता है, जो शस्त्र विकास का एक उपोत्पाद है।

    उपर्युक्त कथनों में से कौन-से सही हैं?

    (a) केवल 1 और 2
    (b) केवल 2 और 3
    (c) केवल 1 और 3
    (d) 1, 2 और 3

    उत्तर: (a)  

    व्याख्या: 

    • विकिरण समस्थानिक ताप-वैद्युत जेनरेटर (RTG) हल्के, कॉम्पैक्ट स्पेसक्राफ्ट पावर सिस्टम हैं जो असाधारण रूप से विश्वसनीय हैं। कभी-कभी इन्हें "न्यूक्लिअर बैटरी" के रूप में भी जाना जाता है। RTG विखंडन रिएक्टर नहीं हैं, न ही प्लूटोनियम का उपयोग परमाणु आयुध के लिये किया जाता है। अतः कथन 1 सही नहीं है। 
    • RTG को अमेरिकी अंतरिक्ष यान जैसे कि वॉयेज कैसिनी और क्यूरियोसिटी द्वारा असाधारण उपलब्धियों वाले मिशनों को शक्ति प्रदान करने के लिये सफलतापूर्वक नियोजित किया गया है। अतः कथन 2 सही है।
    • RTG हल्के, कॉम्पैक्ट स्पेसक्राफ्ट पावर सिस्टम हैं जो असाधारण रूप से विश्वसनीय हैं। RTG प्लूटोनियम ऑक्साइड के रूप में प्लूटोनियम-238 के प्राकृतिक रेडियोधर्मी क्षय से ऊष्मा का उपयोग करके विद्युत शक्ति प्रदान करते हैं। अतः कथन 3 सही है।

    अतः विकल्प (b) सही है।

    Source: NASA AND DRISHTI IAS 


    92. निम्नलिखित कथनों पर विचार कीजिये :

    कथन-I : बृहत तारे वामन तारों की तुलना में अधिक समय तक अस्तित्व में बने रहते हैं।

    कथन-II : वामन तारों की तुलना में, बृहत तारों में नाभिकीय अभिक्रियाओं की उच्चतर दर होती है।

    उपर्युक्त कथनों के संबंध में, निम्नलिखित में से कौन-सा सही है?

    (a) कथन-I और कथन-II दोनों सही हैं तथा कथन-II, कथन-I की व्याख्या करता है
    (b) कथन-I और कथन-II दोनों सही हैं, किंतु कथन-II, कथन-I की व्याख्या नहीं करता है
    (c) कथन-I सही है, किंतु कथन-II सही नहीं है
    (d) कथन-I सही नहीं है, किंतु कथन-II सही है

    उत्तर: (d) 

    व्याख्या: 

    • बृहत तारों का द्रव्यमान हमारे सूर्य के द्रव्यमान से आठ गुना से लेकर 100 गुना तक होता है। इन बृहत तारों में वामन तारों की तुलना में अधिक गर्म एवं सघन क्रोड क्षेत्र होता है। इसलिये बृहत तारों में नाभिकीय अभिक्रियाओं की उच्चतर दर होती है जो तारों को दीप्तिमान बनाती है। अतः कथन 2 सही है।
    • बृहत तारे अपने क्रोड क्षेत्र में हाइड्रोजन ईंधन का भी उपयोग करते हैं, भले ही वे अपनी आरंभिक अवस्था में काफी हद तक हाइड्रोजन युक्त होते हैं, जिसका अर्थ है कि इनकी आयु वामन तारों की तुलना में बहुत कम होती है। अतः कथन 1 सही नहीं है।

    अतः विकल्प (d) सही है।

    Source: NASA 


    93. निम्नलिखित में से कौन-सा, मानव शरीर में संश्लेषित होता है जो रक्त वाहिकाओं को विस्फारित करता है और रक्त प्रवाह को बढ़ाता है?

    (a) नाइट्रिक ऑक्साइड
    (b) नाइट्रस ऑक्साइड
    (c) नाइट्रोजन डाइऑक्साइड
    (d) नाइट्रोजन पेंटऑक्साइड (पेंटॉक्साइड)

    उत्तर: (a) 

    व्याख्या: 

    • नाइट्रिक ऑक्साइड समग्र स्वास्थ्य के लिये एक आवश्यक अणु है। एक वाहिका-विस्फारक के रूप में, नाइट्रिक ऑक्साइड रक्त वाहिकाओं की शिथिलता का संकेत देता है, जिससे वह विस्फारित हो पाती हैं।
    • यह प्रभाव रक्त, पोषक तत्त्वों एवं ऑक्सीजन को शरीर के हर हिस्से में स्वतंत्र रूप से प्रवाहित होने की अनुमति देता है। हालाँकि जब नाइट्रिक ऑक्साइड का उत्पादन कम हो जाता है, तो आपका स्वास्थ्य बिगड़ सकता है।
    • इसलिये, शरीर में नाइट्रिक ऑक्साइड के इष्टतम स्तर को प्राप्त करना एवं नियंत्रित रखना महत्त्वपूर्ण है। नाइट्रिक ऑक्साइड रक्त वाहिकाओं को विस्फारित करता है, रक्त की आपूर्ति बढ़ाता है एवं रक्तचाप को कम करता है।

    अतः विकल्प (a) सही है।

    Source: Healthline 


    94. निम्नलिखित गतिविधियों पर विचार कीजिये :

    1. विमानपत्तनों अथवा विमानों में यात्रियों के पास मौजूद स्वापक पदार्थों (नार्कोटिक्स) की पहचान
    2. वर्षण का मॉनीटरन
    3. पशुओं के प्रवास पर नजर रखना (ट्रैकिंग)

    उपर्युक्त में से कितनी गतिविधियों में रेडारों का उपयोग किया जा सकता है?

    (a) केवल एक
    (b) केवल दो
    (c) सभी तीन
    (d) कोई नहीं

    उत्तर: (c) 

    व्याख्या: 

    • रडार (रेडियो डिटेक्शन और रेंजिंग) सिस्टम वस्तुओं का पता लगाने और उनकी पहचान करने के लिये रेडियो तरंगों का उपयोग करता है। ये रेडियो तरंगों का उत्सर्जन करके वस्तुओं की स्थिति, गति और विशेषताओं को निर्धारित करने के लिये परावर्तित संकेतों का विश्लेषण करता है।
    • परंपरागत रूप से, रडार का उपयोग हवाई यातायात नियंत्रण, मौसमीय निगरानी तथा नौपरिवहन के लिये किया जाता रहा है। अतः बिंदु 2 सही है। 
    • यात्री पहचान के लिये इसके उपयोग ने मौजूदा तकनीकों के अभिनव अनुप्रयोग शामिल हैं। 
      • मिलीमीटर-वेव रडार स्कैनर का उपयोग हवाई अड्डों पर शारीरिक संपर्क के संगुप्त वस्तुओं जैसे स्वापक पदार्थों का पता लगाने के लिये किया जाता है। अतः बिंदु 1 सही है।
    • 1960 के दशक से, रडार प्रवासी जीवों का अध्ययन करने के लिये व्यापक रूप से इस्तेमाल की जाने वाली तकनीक बन गई है। अतः बिंदु 3 सही है।

    अतः विकल्प (c) सही है।

    Source: Nature  and Slideshare


    95. निम्नलिखित विमानों पर विचार कीजिये :

    1. राफेल
    2. MiG-29
    3. तेजस MK-1

    उपर्युक्त में से कितने पाँचवीं पीढ़ी के लड़ाकू विमान माने जाते हैं?

    (a) केवल एक
    (b) केवल दो
    (c) सभी तीन
    (d) कोई नहीं

    उत्तर: (d)

    व्याख्या: 

    • पाँचवीं पीढ़ी (5G) के लड़ाकू विमान अत्यंत प्रतिस्पर्द्धी युद्ध क्षेत्रों, वास्तविक समय आधारित एवं प्रत्याशित सबसे उन्नत हवाई तथा थल खतरों की उपस्थिति में संचालनीय क्षमता रखते हैं।
    • 5G लड़ाकू विमान में स्टील्थ क्षमताएँ होती हैं और आफ्टरबर्नर की सहायता के बिना सुपरसोनिक गति से उड़ान भरने में सक्षम हैं।
    • वर्तमान में रूस के पास सुखोई Su-57, चीन के पास चेंगदू J-20 और अमेरिका के पास F-35 पाँचवीं पीढ़ी (5G) के लड़ाकू विमान हैं।

    अतः विकल्प (d) सही है।

    Source: Drishti IAS  


    96. निम्नलिखित में से किनमें हाइड्रोजेलों का प्रयोग होता है?

    1. रोगियों में नियंत्रित औषधि डिलीवरी
    2. चल वातानुकूलन प्रणाली
    3. औद्योगिक स्नेहकों का विरचन

    नीचे दिये गए कूट का प्रयोग कर सही उत्तर चुनिये:

    (a) केवल 1
    (b) केवल 1 और 2
    (c) केवल 2 और 3
    (d) 1, 2 और 3

    उत्तर: (B/D)

    व्याख्या: 

    • हाइड्रोजेल के अनुकूलीय गुणों, नियंत्रित निम्नं एवं परवर्ती औषध के प्रति संरक्षण क्षमताओं के कारण, हाइड्रोजेल की स्थानीय औषध वितरण प्रणाली के रूप में इसकी क्षमताओं का अन्वेषण किया जा रहा है। अतः कथन 1 सही है।
    • चल वातानुकूलन और रेफ्रिजरेशन प्रणाली में सक्रिय शीतलन तकनीक का उपयोग किया जाता है, जिसमें जीवाश्म ईंधन से बहुत अधिक ऊर्जा खपत होती है। दूसरी ओर, निष्क्रिय शीतलन को एक विकल्प के रूप में माना जाता है क्योंकि यह प्रभावी एवं वहनीय विकल्प है। हाइड्रोजेल लचीले एवं नरम त्रिआयामी नेटवर्क हैं जिनमें उच्च जल सामग्री के साथ विकिरण शीतलन गुण होते हैं जो उन्हें निष्क्रिय शीतलन तकनीक में उपयोग के लिये उपयुक्त बनाते हैं। अतः कथन 2 सही है।
    • हाइड्रोजेल का उपयोग कार्यात्मक स्नेहक के रूप में किया जा रहा है क्योंकि इसके उत्कृष्ट घर्षण-रोधी और जीर्णनरोधी गुण, अनुकूलनीय लाक्षणिक प्रयोग क्षमता तथा अर्द्ध ठोस प्रकृति के कारण यह पारंपरिक तरल स्नेहक या रिसाव के कारण होने वाली स्नेहन विफलताओं को कम करने में प्रभावी है। हालाँकि, वर्तमान में अनिर्णायक शोध के कारण, औद्योगिक स्नेहक के रूप में इसके वास्तविक अनुप्रयोग का पता लगाना कठिन है। अतः कथन 3 सही है। 

    अतः विकल्प (B/D) सही है।


    97. निम्नलिखित में से कौन-सा हाइड्रोजन द्वारा चालित फ्यूल सेल इलेक्ट्रिक वाहनों से निकलने वाला निर्वातक नली उत्सर्जन है?

    (a) हाइड्रोजन परऑक्साइड (परॉक्साइड)
    (b) हाइड्रोनियम
    (c) ऑक्सीजन
    (d) जल-वाष्प

    उत्तर: (d)

    व्याख्या:

    • ईंधन-सेल इलेक्ट्रिक वाहन केवल जल वाष्प और गर्म हवा उत्सर्जित करते हैं, जिससे हानिकारक निर्वातक नली उत्सर्जन नहीं होता है। विद्युत के समान हाइड्रोजन भी एक ऊर्जा वाहक है जिसका उत्पादन विभिन्न फीडस्टॉक्स से किया जा सकता है। हाइड्रोजन उत्सर्जन का आकलन करते समय इन फीडस्टॉक्स और उत्पादन विधियों पर विचार किया जाना चाहिये।
    •  फ्यूल सेल इलेक्ट्रिक वाहन (FCEV) इंजन पारंपरिक आंतरिक दहन इंजन के समान हैं क्योंकि वे भी ईंधन (हाइड्रोजन) और ऑक्सीजन की निरंतर आपूर्ति पर निर्भर करते हैं।
    • हालाँकि ईंधन सेल में कोई गतिशील भाग नहीं होते हैं, इसलिये वे अधिक कुशल और विश्वसनीय होते हैं।

    अतः विकल्प (d) सही है।

    Source: 


    98. हाल ही में, ‘‘पंप्ड-स्टोरेज हाइड्रोपावर’’ शब्द की वास्तव में और समुचित रूप से निम्नलिखित में से किसके संदर्भ में चर्चा की गई है?

    (a) सीढ़ीदार खेतों की सिंचाई
    (b) धान्य (अनाज) फसलों की उत्थान सिंचाई
    (c) दीर्घावधि ऊर्जा भंडारण
    (d) वर्षा-जल संचयन प्रणाली

    उत्तर: (c)

    व्याख्या:

    • ‘पंप्ड-स्टोरेज हाइड्रोपावर’ एक प्रकार का हाइड्रोइलेक्ट्रिक ऊर्जा भंडारण है जो विद्युत उत्पन्न करने के लिये अलग-अलग ऊँचाई पर दो जलाशयों में संग्रहीत जल का उपयोग करता है।
    • जब अतिरिक्त विद्युत उपलब्ध होती है अर्थात् नवीकरणीय स्रोतों जैसे सौर एवं पवन से पर्याप्त आपूर्ति की स्थिति, अतः विद्युत की कम आवश्यकता वाली स्थिति में इसका उपयोग निचले जलाशय से ऊपरी जलाशय में जल पंप्ड करने के लिये किया जाता है।
    • जब विद्युत की मांग होती है, तब जल ऊपरी जलाशय से पुनः निचले जलाशय में छोड़ा जाता है, जो विद्युत उत्पन्न करने वाले टर्बाइनों से होकर गुज़रता है।

    अतः विकल्प (c) सही है।

    Source: 


    99. ‘‘मेम्ब्रेन बायोरिऐक्टरों’’ की चर्चा प्राय: किस संदर्भ में की जाती है?

    (a) सहायताप्राप्त प्रजनन प्रौद्योगिकियाँ
    (b) औषधि डिलीवरी नैनो-प्रौद्योगिकियाँ
    (c) टीका (वैक्सीन) उत्पादन प्रौद्योगिकियाँ
    (d) अपशिष्ट-जल शोधन प्रौद्योगिकियाँ

    उत्तर: (d) 

    व्याख्या: 

    • मेम्ब्रेन बायोरिएक्टर (MBR) तकनीक पारंपरिक जैविक अपशिष्ट जल शोधन एवं आधुनिक मेम्ब्रेन प्रक्रियाओं का एक कुशल संकर उत्पाद है जिसका उपयोग नगरपालिका तथा औद्योगिक दोनों के अपशिष्ट जल शोधन में किया जाता है। पारंपरिक विद्यमान अपशिष्ट प्रक्रियाओं के समान जिनमें कार्बनिक प्रदूषकों को नष्ट करने के लिये सूक्ष्मजीवों का प्रयोग किया जाता रहा है, यह विधि निलंबित ठोस पदार्थों को हटाने के लिये उन्नत मेम्ब्रेन का उपयोग करती है, जिससे हैवी क्लेअरिफायर की आवश्यकता समाप्त हो जाती है। 

    अतः विकल्प (d) सही है।

    Source: 


    100. भारतीय अर्थव्यवस्था के संदर्भ में, ‘‘संपार्श्विकीकृत उधार लेन-देन संबंधी दायित्व’’ निम्नलिखित में से किसके लिखत (इंस्ट्रमेंट) हैं?

    (a) बॉण्ड बाज़ार
    (b) विदेशी मुद्रा बाज़ार
    (c) मुद्रा बाज़ार
    (d) शेयर (स्टॉक) बाज़ार

    उत्तर: (c)

    व्याख्या:

    • संपार्श्विकीकृत उधार लेन-देन संबंधी दायित्व (CBLO) मुद्रा बाज़ार के लिखत है जो ऋण की शर्तों और शर्तों के संबंध में उधारकर्त्ता एवं ऋणदाता के बीच दायित्व का प्रतिनिधित्व करता है। CBLO किसी भी विशिष्ट देश में अंतर-बैंक कॉल मनी मार्केट का उपयोग करने से प्रतिबंधित लोगों को अल्पकालिक मुद्रा बाज़ारों में भाग लेने की अनुमति देता है।
    • क्लियरिंग कॉरपोरेशन ऑफ इंडिया लिमिटेड (CCIL) ने 20 जनवरी, 2003 से संपार्श्विकीकृत उधार लेन-देन संबंधी दायित्व (CBLO) नामक एक मुद्रा बाज़ार लिखत प्रस्तुत किया है। 

    अतः विकल्प (c) सही है।

    Source:

    Drishti IAS Link: NA




    close
    एसएमएस अलर्ट
    Share Page
    images-2
    images-2
    × Snow